COMLEX DIT STEP 3

Réussis tes devoirs et examens dès maintenant avec Quizwiz!

What patients should receive intrapartum prophylaxis against group B strep?

+GBS during the pregnancy GBS bacteremia this pregnancy Previous infant with early onset GBS 1st week Unknown screening plus 1 of the following - intrapartum fever, PROM, preterm labor before 37 weeks

Temporal arteritis (GCA)

- A type of vasculitis that affects the branches of the external carotid and vertebral arteries - More common in older women (almost never before age 50) - Associated with PMR

A 32 year old man is brought to the ED while having a seizure. His friends report that he's been convulsing for the last 20 minutes. What is the most appropriate next step in the treatment of this patient?

- ABCs - IV Lorazepam - Phenytoin/fosphenytoin => if still has seizure, do sedation => intubate => phenobarbital or pentobarbital

What are the treatments for hypercalcemia?

- IV hydration - Calcitonin - decrease bone resorption + increase renal calcium excretion by osteoclasts - Bisphosphonates - Glucorticoids

Epidural hemotoma: Px and Tx

- Lucid interval - Damage to MMA middle meningeal artery Ct: Lens shaped lesion that can cross the midline Tx: drain hematoma if surgical Tx (medical): Elevate head, hyperventilate, mannitol

PULMONARY EMBOLISM Presentation?

- SOB, tachypnea, tachycardia, palpitations, pleuritic chest pain, cough, hemoptysis, fever, AMS - EKG may show sinus tachycardia; S1Q3T3 is an uncommon finding

How is HIV infection managed through pregnancy?

- Screen at 1st visit and then in 3rd trimester - Antiretroviral regimen - HAART -viral load as low as possible - Monitor Viral load > 1000 copies/mL => intrapartum IV zidovudine (AZT), C/S delivery <1000 copies/mL => SVD, no fetal scalp electrodue monitoring

What conditions are associated with erythema nodosum?

- Streptococal pharyngitis - Sarcoidosis - TB - Fungal infections: coccidiodomycosis, histoplasmosis, blastomycosis - Inflammatory bowel disease - Pregnancy/OCP use - Idiopathic

what are characteristic features of serotonin syndrome

- mental status changes: anxiety, agitation, delirium, restlessness, disorientation - autonomic excitation: diaphoresis, tachycardia, hyperthermia, hypertension, nausea, vomiting, diarrhea - neuromuscular hyperactivity: tremor, muscle rigidity, hyper reflexia, mild clonus, ocular clonus

Post-infectious glomerulonephritis

1-3 weeks after a group A strep infection Damage is due to immune complexes deposited into the glomerulus = ASO abs Self-limited

Transfusion reactions Nonhemolytic febrile reaction

1-6 hours after starting transfusion Fever, chills, rigor, malaise Stop transfusion (to r/o hemolytic reaction), acetaminophen

Transfusion reactions Transfusion-related acute lung injury

1-6 hours after starting transfusion Pulmonary infiltrates and hypoxemia, fever, hypotesion Stop transfusion, O2, CPAP or intubation, IV fluids

During pregnancy when should each of the following actions be performed? 1. Screening for syphilis 2. Screening for HIV 3. Triple or Quad scren 4. Screening ofr gestational diabetes 5. Administration of Rh immune globulin if Rh negative 6. Screening for gonorrhea/chlamydia 7. Screening for GBS

1. 1st prenatal, high risk - 32 weeks 2. 1st visit, 3rd trimester if high risk 3. 15-20 weeks 4. 24-28 weeks, 1 hr 50 g GTT => then 3 hr 100 mg GTT 5. 28 weeks, after delivery, anytime fetomaternal hemorrhage 6. 1st visit, 3rd trimester if high risk 7. IV PCN @ labor 35-37 weeks

What are JNC -8 treatment goals for HTN in each of the following patient populations? Gen pop age >60 Gen pop age <60 Adults with DM Adults with CKD

1. <150/90 2. <140/90 3. <140/90 4. <140/90

you suspect the patient is suffering from claudication due to peripheral vascular disease of the lower extremities. What studies can be used to confirm the diagnosis were medication can be used to help relieve this patient's symptoms

1. ABI, Doppler ultrasound <0.9 => PVD 2. exercise, angioplasty, surgical revascularization, cilostazol

What are the JNC-8 recommendations for initial pharmacotherapy in each of the following patient populations? CKD patients Black patients (w/o CKD) Non-black pts (w/o CKD)

1. ACE-i/ARB 2. Thiazide or CCB 3. ACEi/ARB or CCB or Thiazide

what is the most likely knee injury given each of the following findings? 1. Positive anterior drawer test and positive Lachman's test 2. positive posterior drawer test 3. tibial joint line tenderness with clicking or locking and positive McMurray's test

1. ACL injury 2. PCL injury 3. meniscus tear

A 9-year-old boy is brought to the physician by his mother because of poor performance at school. He makes careless mistakes on test in homework and he is easily distracted. He forgets to do his household chores and does not follow through on instructions. Physical examination reveals a well-developed child who visits during the interview. He frequently interrupts his mother and examiner, bloating out answers to the questions before the question is completed. What is the most likely diagnosis? When medications are consider 1st line treatment of this condition?

1. ADHD 2. methylphenidate, dextroamphetamine, atomoxetine

A 47 year old man comes to the physician with muscle weakness that began in the hands and has progressively worsened over the past two months. He has also noticed weakness in the legs, worsening of his balance, and slurred speech. Physical examination shows muscle atrophy and fasciculations in the hands and the calves. Handgrip strength is 1/5. Motor strength is 2/5 in the other major muscle groups of the upper and lower extremities. The sensory examination is normal. Biceps, patellar, and Achilles reflexes are all 3+. The babinski reflex is positive bilaterally. What is the most likely dx? What treatment may benefit this patient?

1. ALS 2. Riluzole

answer the following questions regarding neuroleptics malignant syndrome 1. what are the signs and symptoms of NMS? 2. What is the treatment of NMS?

1. AMS, muscle rigidity, fever, autonomic dysfunction 2. Stop meds, IVF, cooling blankets, clonidine or nitropruside for BP, BZs for agitation, Bromocriptine, Amanatidine, Dantrolene

Which antiviral agent is known for each of the following effects? Fatal hypersensitivity reaction Hyperbilirubinemia Boosting the levels of other protease inhibitors

1. Abacavir 2. Atazanavir 3. Ritonavir

An 11 year old boy is brought to the clinic for occasional periods of iimpaired consciousness while at school. These periods last no longer than 30-45 seconds. His teacher believes the boy is daydreaming and finds difficulty getting his attention 1. What is the most likely cause of the patient's presentation? 2. What is the tx?

1. Absence seizures 2. Ethosuximide

A 55 year old man is diagnosed with essential htn and begins taking an antihypertensive medication. two weeks later, he is brought to the ED for swelling of the lips, cheeks, and tongue. He cannot speak clearly, but his airway is not compromised. What class of antihypertensive medication is most likely responsible? Accumulation of what substance is most likely responsible? How should this patient be managed?

1. Ace-inhibitor 2. Bradykinin 3. DC meds, close monitoring for airway compromise, intubate if indurated, antihistamines may lead to quicker resolution, epinephrine + glucocorticoids do not appear helpful.

Identify the hep B status for each of the following patients based on the serologies: - HBsAg+ , HBsAb- , HBcAb+ , HbeAg+ - HBsAg- , HBsAb+ , HBcAb- , HBeAg- - HBsAg- , HBsAb- , HBcAb+ (IgM)

1. Acute Hep B 2. Hep vaccine 3. Window period

What is the most likely cause of vertigo in each of the following scenarios? 1. Acute, self-limited vertigo with hearing loss and tinnitus 2. Vertigo is independent of head movement; no hearing loss or tinnitus; may result from viral infection 3. vertigo + ataxia, hearing loss, and tinnitus; MRI shows a tumor near the internal auditory canal 4. Vertigo with change of head position; no hearing loss, tinnitus, or ataxia; positive Dix-halpike maneuver 5. chronic vertigo + hearing loss and tinnitus; treat with salt restrictions and diuretics

1. Acute Labrynitis 2. Vestibular neuritis 3. Acoustic neuroma 4. BPPV 5. Meniere's

Hep B scenarios 1. HbsAg(+) , HBsAb(-) , HBcAb(+IgM), HBeAg(+) 2. HbsAg(-) , HBsAb(-) , HBcAb(+IgM), HBeAg(+/-) 3. HbsAg(+) , HBsAb(-) , HBcAb(+IgG), HBeAg(+) 4. HbsAg(+) , HBsAb(-) , HBcAb(+IgG), HBeAg(-) 5. HbsAg(-) , HBsAb(+) , HBcAb(+IgG), HBeAg(-) 6. HbsAg(-) , HBsAb(+) , HBcAb(-), HBeAg(-)

1. Acute infection 2. Window period 3. Chronic infection with high transmission 4. Chronic infection with low transmission 5. Past infection 6. Vaccination

A 49 year old man is brought to the emergency department with epigastric pain, nausea, and vomiting for 2 days. He has not eaten during that time. He has a history of alcohol abuse. Physical examination shows epigastric tenderness without guarding. There are ecchymoses on the flanks bilaterally. 1. What is the most appropriate initial lab test for this patient? 2. What is the most appropriate initial mgnt of this condition?

1. Acute pancreatitis for alcohol. Grey-Turner sign Serum Lipase 2. Admission Monitoring HDS + organ failure Aggressive hydration NPO Morphine for pain Establish underlying cause (gallstones #1 most common cause)

A 34 year old man with a hx of primary adrenal insufficiency is brought to the emergency department because of AMS. His medications includ ehis usual dose of hydrocortisone and fludrocortisone. He is admitted to the hospital for treatment of shock and pneumonia. VS are temperature 38.6C, heart rate 120/min, bp 80/50 and resp 24/min. Lab studies show hyponatremia, hyperkalemia, and hypoglycemia 1. What is the most likely cause of this patient's shock? 2. What is the most appropriate mngt of this patient's condition?

1. Adrenal crisis triggered by pneumonia 2. IV Dexamethasone IV hydration IV glucose Correct electrolyte imbalances Hemodynamic monitoring Treat undelrying cause Vial of Dexamethasone @ home

You are asked to evaluate a 45 year old man on the inpatient ward for elevated liver function tests. He c/o epigastric and ruq pain that began 8 hours ago. He also had fever, nausea, and vomiting. 1. What type of hepatitis would most likely cause the AST to be twice as high as the ALT? 2. In a patient with acute hepatitis, what type of bilirubin would you expect to be elevated?

1. Alcoholic hepatitis 2. Increase direct bilirubin = hepatobiliary disease Increase indirect bilirubin = hemolysis

A 47 year old man comes to the ER with headache, diaphoresis, and tachycardia. His BP is 176/99 mm Hg. HE is found to have markedly elevated levels of norepinephrine, dopamine, and metanephrine in the urine. A Ct scan of the abdomen shows a 3 cm mass in the right adrenal gland. 1. What preoperative treatment is necessary in this patient? 2. What diagnoses should be considered if the CT showed bilateral adrenal masses? 3. What other disorders of the adrenal glands can cause secondary hypertension?

1. Alpha blocker - phenoxybenzamine, phentolamine, labetalol (alpha/beta) 2. MEN type 2 - VHL diseae 3. Cushing syndrome Hyperaldosteronism: HTN, Hypokalemia, Met alkalosis

Adult immunizations 1. Influenza 2. Pneumococcal 3. Herpes zoster 4. Td booster 5. Meningococcal vaccine 6. Hep A vaccine 7. Hep B vaccine

1. Annually in everyone over age 6 months 2. Once after age 65, may be given to younger high-risk individuals 3. All patients over age 60 4. Booster every 10 years - Tdap given once in place of the usual Td booster 5. Asplenic patients 6. Patients with chronic liver disease (chronic hep C) 7. Patients with chronic liver disease, Patients receiving treatment for STIs or HIV, patients with multiple sexual partners, IV drug users

A 73 year old man with a history of stable angina presents with substernal chest pain that began at rest. His serum troponin is elevated and an initial ECG shows ST elevation in leads V2, V3, V4, and V5. What is the most likely diagnosis? Which artery is most likely involved?

1. Anterior Wall STEMI 2. LAD - most common

A 53 year old man with a hx of atherosclerosis and Aortic aneurysm repair comes to the clinic with loss of sensation and motor paralysis of both lower extremities. Physical exam reveals loss of pain and temperature sensation and motor function at level of T10 and below. Vibration sense and proprioception are intact in the feet. 1. What condition most likely explains this presentation? 2. Why are vibration sense and propioception preserved?

1. Anterior spinal artery syndrome 2. Dorsal column spared

A 46 year old woman presents with pain and swelling in her fingers that began about 3 months ago and has been gradually getting worse. The pain and stiffness are generally worse when she first gets up in the morning but improve after she "loosens up" She also complains of stiffness in her knees and generalized fatigue. Physical examination reveals slight tenderness and swelling of multiple MCP and PIP joints bilaterally and the right wrist. What is the most specific lab test to confirm the most likely dx? How is this diagnosis treated?

1. Anti-citrullinated protein Antibody 2. DMARDs - Hydroxychloroquine, Sulfasalazine, MTX, TNF-alpha inhibitor like etanercept, infliximab - START EARLY

A 72 year old man comes to the clinic for exertional dyspnea for the past week. He was treated for infective endocarditis 6 weeks ago. Vital signs are temperature 37C, respirations 16/min, pulse 100/min, and bp 148/62 mmHg. There are boudning pulses. A diastrolic descrendo murmur is heard at the left sternal border. Echo shows early mitral valve closure and reverse blood flow across the aortic valve. WBC and ESR are normal. What is the most likely dx? What two classes of medication would most likely improve this patient's sx?

1. Aortic regurgitation from Infective Endocarditis 2. ACE-inhibitor or CCB to decrease afterload

Match the pneumoconiosis to the corresponding findings 1. Associated with shipbuilding, roofing, plumbing, and brake mechanics 2. Associated with foudnries (casting metal), sandblasting, and mining 3. Associated with coal mines, large urban centers, and tobacco smoke 4. Associated with exposure in the nuclear and aerospace industry

1. Asbestosis 2. Silicosis 3. Coal worker's 4. berylliosis

A 42 year old woman is diagnosed with sympomatic cholelithiasis and is being evaluated for an elective cholecystectomy. She has a hx of bipolar disorder with psychotic feature. 1. Who should determine whether this patient has the capacity to make health care decisions? 2. What is required to determine if a patient has decision making capacity?

1. Attending physician. Competence: Judge 2. At least 18 yo (legally emancipated minor) Capable of making + communicating a treatment decision Informed of risks, benefits, and alternatives Decision consistent with patient's values Stable overtone (decision) Cannot be influenced by delusions, hallucinations, and mood disorders

Seizure types? 1. absence 2. myoclonic 3. tonic-clonic 4. tonic 5. atonic 6. status epilepticus

1. Blank stare, smacking the lips 2. Brief involuntary twitching of muscles 3. Stiffing and shaking 4. Stiffening 5. Loss of muscle tone 6. Seizure lasting longer than 5-10 minutes

APGAR SCORE Appearance Pulse Grimace Activity Resp

1. Blue all over, acrocyanosis, no cyanosis 2. No pulse, <100, >100 3. No response, grimace/feeble cry, strong cry 4. No muscle tone, some flexion, flexed arms + legs 5. Respiration: Absent, weak + irregular, regular + strong cry

Which psychiatric disorder matches each of the following descriptions? 1. Patient with normal anatomy is convinced a part of his/her anatomy is abnormal 2. Unexplained loss of sensory or motor function without any basis in a medical or neurological condition 3. Patient is preoccupied with the possibility of contracting a serious illness and is constantly checking his/her body for signs of illness 4. Patient produces false signs/symptoms of an illness in order to receive sympathy or medical attention

1. Body dysmoprhic disorder 2. Conversion disorder 3. Illness anxiety disorder 4. Factitious disorder

Answer the following questions regarding the diagnosis of brain death. 1. What dx tests can assist in declaring a patient brain dead? 2. What clinical scenarios can mimic brain death?

1. Brain blood flow - cerebral angiography - Transcranial doppler EEG 2. Locked-in syndrome, GBS, Hypothermia, Drug intoxification, Neuromuscular paralysis

What pediatric immunodeficiency syndrome matches each of the following statements? 1. Recurrent bacterial infections after age of six months 2. Recurrent sinopulmonary infections + anaphylactic reaction to blood transfusion 3. Thymic aplasia + T cell deficiency + hypocalcemia 4. Severe recurrent infections by a variety of organisms 5. Thrombocytopenia + eczema + recurrent infections

1. Bruton (X-linked) agammaglobulinemia 2. IgA deficiency 3. DiGeorge Syndrome 4. SCID 5. Wiskott-Aldrich Syndrome

A 67 year old is admitted to the hospital for community-acquired pneumonia. He is treated with levofloxacin and his sxs improve. Six days after d/c, he develops watery diarrhea, crampy lower abdominal pain, and fever. 1. What pathogen is the most common cause of antibiotic-associated diarrhea? 2. What is the most appropriate initial therapy for this patient? 3. What treatments should be considered if initial therapy fails or the diarrhea recurs?

1. C diff 2. Metronidazole 3. Oral vancomycin, fidoxomin, rifaximin, fecal bacteriotherapy

Which hematologic malignancy matches each of the following descriptions? 1. Always positive for t(9;22) 2. 10 year old boy with trisomy 21 3. Treated with bortezomib plus melphalan 4. Treated with ATRA 5. Treated with thalidomide 6. Treated with rituximab 7. Treated with imatinib

1. CML 2. ALL 3. Multiple Myeloma 4. AML (M3) 5. Muliple Myeloma 6. NHL (CD20+) 7. CML

What are hemodynamic changes seen in these shock? 1. cardiogenic 2. Hypovolemic 3. Neurogenic 4. Anaphylactic 5. Septic

1. CO down, PCWP up, SVR up - dobutamine/norepi/dopamine 2. CO down, PCWP down, SVR up - IVF 3. CO down, PCWP down, SVR down - IVF; pressors; atropine or external pacing for HR 4. CO up, PCWP down, SVR down - Epinephrine 5. CO up, PCWP down, SVR down - IVF, Abx, Norepi

What is the most likely cause of secondary htn given the following findings? 1. High BP in the arm but low bp in the legs 2. Hypokalemia 3. Hyperkalemia 4. Episodic sweat, tachycardia

1. COA 2. Primary aldosterone, RAS 3. Renal Failure 4. Pheochromocytoma

answered the following questions about esophageal perforation what is the 1st study of choice in the suspect esophageal perforation? What is the most common cause of esophageal perforation?

1. CT chest, Gastrografin with contrast, esophagram, no barium 2. iatrogenic most common, Boerhaave syndrome, trauma to the chest

Answer the following questions regarding neutropenic fever 1. What patient population is most susceptible to acquiring a neutropenic fever 2. What is clinical definition of neutropenic fever

1. Cancer pts with chemo 2. Fever: 38.4 celsius on single reading or >38C for >1 hr Neutropenia: ANC < 500

A 31 year old woman comes to the clinic for amenorrhea for the last 6 months. She is not pregnant and does not desire pregnancy presently. Lab evaluation shows a serum prolactin level of 126. An MRI of the brian shows a 3 mm mass in the sella turcica. 1. What is the most appropriate medical mngt of this patient? 2. What are the indications for surgical tx of this condition?

1. Carbegoline, Bromocriptine 2. Failure of medical therapy Adenoma > 3 cm who desires pregnancy

A 65 year old man with a hx of ischemic cardiomyopathy due to chronic CAD is admitted with pneumonia. Appropriate abx are given. During the course of his hospital stay he develops hypotension and shock. A swan-ganz pulmonary artery catheter is inserted and reveals the following: decreased CO, increased SVR, and increased pulmonary capillary wedge pressure. What is the most likely cause of this patient's shock? What is the most appropriate treatment to raise his bp acutely?

1. Cardiogenic because SVR increase and wedge presssure increase 2. Dobutamine or Dopamine

What type of research study? Compares a group of people with a given disease to a group without the disease Compares a group with a given risk factor or exposure to a group without that risk factor Participants are randomly assigned to either the experimental group or the control group

1. Case control 2. Cohort 3. RCT

A 21 year old woman comes to the clinic with six months of chronic diarrhea. She c/o occasional nausea/vomiting and abdominal pain. Analysis of the stool shows steatorrhea. Serum anti-endomysial antibody and anti-tissue transglutaminase antibodies are positive. 1. What test should be performed to confirm the dx? 2. How should this condition be managed?

1. Celiac disease: confirmed by small bowel biopsy 2. Avoid gluten-contaminated foods Barley Rye Oats Wheat

A 71 year old man comes to the clinic with a persistent cough. There is no fever or sputum production. He has minimal sob. He smoked 1 pack of cigarettes daily for 46 years, but for the last 5 years he has cut down to about 2 cig/day. An x-ray of the chest reveals 2 small nodules in the right upper lung. CT scan shows enlargement of several lymph nodes in the right lung and mediastinum. A needle biopsy confirms the dx of small cell lung cancer 1. What treatment modalities would be most appropriate for this patient? 2. What paraneoplastic syndromes are associated with small cell lung cancer? 3. What paraneoplastic syndromes are associated with squamous cell lung cancer?

1. Chemo + radiation. Not surgically resectable 2. SIADH ACTH => CUSHING LAMBERT-EATON MYASTHENIA SYDNROME - muscle weakness that increase with use 3. Hypercalcemia PTHrP => increase bone turnover, hypercalcemia Pancoast Tumor => horner: ptosis, miosis, anhidrosis

What is the mechanism of action of each of the following drugs Amiodarone atenolol Disopyramide Diltiazem Procainamide Propafenone

1. Class III 2. Class II 3. Class IA 4. Class IV 5. Class IA 6. Class IC

A 63 year old man is intubated and admitted to the ICU for an acute exacerbation of COPD 1. In what situation is ppx against stress ulcers in the ICU indicated? 2. What stress ulcer ppx treatment or treatments are linked with an increased risk of C. difficile infection?

1. Coagulopathy, Intubation/ventilation > 48 hrs, GI ulceration/bleeding within past year, Head trauma, Spinal cord trauma, Brain injury >2 of the following: sepsis, ICU > 1 week, occult GI bleeding for over 6 days, Glucorticoid treatment 2. PPI + H2 blocker

Fungi causing pneumonia Desert southwest Ohio or Mississippi River valleys: Cave with bat droppings:

1. Coccidiodiomycosis 2. Histoplasmosis 3. Histoplasmosis

Which microorganism is associated with each of the following clinical or epidemiological clues? Travel in the desert southwest Rust-colored sputum Cave exploring Pneumonia in HIV patient with CD4 count < 200

1. Coccidiodoes Immitis 2. S. pneumoniae 3. Histoplasma Capsulatum 4. PCP

A 20 year prospective observational study looking at the correlation between smoking and the development of lung cancer 1. What type of study is this 2. Based upon what is the relative risk of lung cancer associated with smoking?

1. Cohort study 2. A/A+B / C/C+D

A 19 year old woman is brought to the emergency department because of severe right eye pain. She says that the pain started after she accidentally poked herself in the eye while applying make up. She now complains of photophobia and a foreign body sensation. The patient is reluctant to open her right eye completely. Physical examination shows mild conjunctival injection but no evidence of penetrating trauma. What is the most likely dx? How can the dx be confirmed? What is the most appropriate management?

1. Corneal abrasion 2. Fluorescein dye-yellow on cornea 3. Topical erythromycin 3-5 days, analgesia

A 65 year old woman has diagnosed with Inflammatory bowel disease 5 years ago. She complains of colicky right lower quadrant abdominal pain and occasional low-grade fevers. A colonoscopy reveals cobblestoning and skip lesions. 1. What would the colon biopsy most likely reveal? 2. Under what clinical circumstances would you refer this patient for a curative colectomy?

1. Crohn's 2. Cannot be cured by colectomy. There is a high risk of recurrence Tx: 5-ASA, steroids, immunosuppressants, Anti-TNF alpha drugs

What is the most likely cause of diarrhea in each of the following patients? 1. HIV-positive patients with <100 CD4 cells 2. Vomiting and diarrhea after eating reheated Chinese fried rice 3. Vomiting and diarrhea after eating raw oysters 4. Diarrhea began after backpacking in the mountains 5. Patient recently treated for UTI

1. Cryptosporidium 2. Bacillus Cereus 3. Vibrio parahaemylyticus 4. Giardia Lambria 5. C. diff

A 61 year old man is admitted to the hospital for an acute exacerbation of COPD. He plays golf regularly and the chairman of the cardiology department. At grand rounds, the cardiologist asks the treating pulmonologist how his friend is doing. 1. Under what circumstances may a physician share a patient's confidential information? 2. What is the most appropriate response to the cardiologist's request for information?

1. Danger or harm to others Evidence of child/elder abuse Reportable disease (HIV, TB) Pt grants permission Providers/Facilities involved in Pt's care 2. Refuse to Share

What is the management of preeclampsia with severe features?

1. Decrease BP: IV labetalol, Hydralazine, and Nifedipine 2. Seizure ppx: IV magsulfate 3. Delivery

Cellulitis depth of infection causative organisms exam findings treatments

1. Deeper dermis, subQ fat 2. S. aureus, S. pyogenes, others 3. Spreading warmth, edema + redness 4. Oral dicloxacillin/cephalexin IV cefazolin or Clindamycin

What is antidote or reversal agent for each of the following Digoxin Warfarin Heparin tPA Anticholinergics Organophosphates Carbon Monoxide Lead poisoning Mercury poisoning Copper poisoning Iron poisoning

1. Digoxin Immune Fab 2. IV Vitamin K, FFP 3. Protamine Sulfate 4. Aminocaproic acid 5. Physostigmine 6. Atropine, Pralidoxime 7. Hyperbaric Oxygen, 100% Oxygen 8. EDTA, Succimer, Dimercaprol 9. Dimercaprol, Penicillamine 10. Penicillamine 11. Deferoxamine

A 86 year old woman is brought to the ED for AMS. She is found to have a serum sodium of 122. 1. How rapidly can hyponatremia be safely corrected? 2. What is the consequence of correcting hyponatremia too rapidly?

1. Do not raise more than 12 mEq per 24 hours 2. Risk of central pontine myelinolysis w rapid correction - Paralysis, Dysphagia, Dysarthria

Class IA Class IB Class IC

1. Double quarter pounder - Disopyramide - Quinidine - Procainamide Lengthens action potential (shifts right) 2. Mayo, Lettuce Tomato - Mexilitine - Lidocaine - Tocainide shortens action potential (shifts left) 3. Fries Please - Fleicanide - Propafenone No chnage to duration of action potential

Answer the following questions regarding Rocky Mountain spotted fever 1. What abx treatment of choice for RMSF? 2. Whatis the abx treatment of choice for RMSF in pregnancy?

1. Doxycycyline 2. Chlroamphenicol

Answer the following questions regarding malaria prophylaxis 1. Why might chloroquine be avoided in a patient for malaria ppx? 2. In what patient should primaquine be avoided by malaria ppx?

1. Drug resistance to chloroquine common in many parts of the world 2. G6PD deficiency patients

A 28 year old G2P2 woman develops a fever one day after undergoing an emergent Cesarean delivery because of a nonreassuring fetal heart rate tracing. Physical examination shows uterine tenderness and foul-smelling lochia. What is the most likely diagnosis What is the treatment

1. Endometritis 2. IV broad spectrum - gentamicin + clindamycin

Answer the following questions regarding Lyme disease 1. What is the classic presenting sx in a patient with Lyme disease? 2. What abx are used in the treatment of Lyme disease?

1. Erythema Migrans 2. Doxy, amox, Ceftriaxone

A 38 year old man comes to the clinic with progressive difficulty swallowing for the last 8 weeks. Both solids and liquids seem to get stuck in his chest. He also has occasional heartburn. A barium esophagram shows narrowing of the esophagogastric junction and a "bird's beak" appearance, with dilation of the proximal esophagus. 1. What is the most appropriate study to confirm the dx? 2. How is this condition managed?

1. Esophageal manometry 2. Esophageal dilation (pneumatic dilation) Surgical myotomy Botulinum toxin injections Medical therapy (nifedipine, nitrates)

A 42 year old man complains of a tremor that is present at rest but worsens when he moves his arm. He admits that his grandmother has a similar tremor when he was growing up. He notes that the tremor is less pronounced after drinking a few beers. 1. What is the most likely diagnosis? 2. What treatment can help minimize his tremor?

1. Essential tremor 2. Beta blocker

7-year-old boy has episodes where he abruptly stops talking mid sentence, stares into space, and then resumes speaking without realizing what just occurred. Sometimes during these episodes he will smacked his lips; most appropriate treatment for this patient

1. Ethosuximide 2. Valproic Acid

Blunt Abdominal Trauma - Hemodynamically unstable - Hemodynamically stable

1. FAST exam to look for blood => + => OR If negative => CT or DPL if CT not available If normal, look for other bleeding sites (pelvic fx) 2. Low risk: Observe +/- US or CT If intoxicated, not fully alert, or mild decrease in hematocrit: Do a CT

Answer the following questions regarding endocarditis 1. What are the physical findings of bacterial endocarditis 2. What is the treatment of bacterial endocarditis

1. Fever, new murmur, osler nodes, Janeway lesions, Splinter hemorrhages, Roth spots 2. Empiric: vancomycin 4 to 6 weeks Valve replacement

A 69 year old woman comes to the clinic complaining of vague abdominal pain that has been present for 4-6 weeks. She has noted decreased appetite and has had a 4.5 kg weight loss during this time. Physical examination reveals signficiant jaundice. A CT scan of the abdomen shows a 1.5 cm mass in the head of the pancreas. What options are available to biopsy this mass? If the biopsy shows pancreatic adenocarcinoma, what factors determine if this lesion is surgically resectable?

1. Fine needle aspiration - EUS - Percutaneous w. CT or US guidelines 2. Resectable if - no mets to lymph nodes beyond peripancreatic region - no mets to liver, peritoneum or extraabdominal site - no encasement of vasculature

Which vitamin efficiency matches each of the following descriptions? 1. Macrocytic anemia with hypersegmented neutrophils 2. Swollen gums, poor wound healing, bleeding mucous membranes, spots on the skin 3. Dermatitis, diarrhea, dementia 4. Hemorrhagic disease 5. Neural tube defects 6. Pernicious anemia 7. May result from kidney disease 8. Bitot spots, keratomalacia, xerophthlamia 9. Cheilosis and glossitis 10. Dilated cardiomyopathy, edema, polyneuropathy 11. Osteomalacia and rickets

1. Folate/B12 def 2. Scurvy - vitamin C def 3. Vitamin B3 - pellagra 4. Vitamin K 5. Folic acid 6. Vitamin B12 7. Vitamin D 8. Vitamin A 9. Vitamin B2, B3, B9, B12 10. Thiamine def. 11. Vitamin D def.

What is the most likely cause of amenorrhea if associated with each of the following statements? 1. Ballet dancer with eating disorder 2. Short stature, low-set ears, webbed neck 3. Following dilation and curettage after spontaneous abortion 4. Cyclic pelvic pain + blue bulge at introitus on exam 5. Elevated testosterone level and no axillary or pubic hair on exam 6. Overweight female with acne and hirsutisum 7. Postpartum patient with inability to breastfeed 8. Patient with anosmia 9. Patient with galactorrhea 10. Most common cause of amenorrhea

1. Functional Hypothalamic Amenorrhea 2. Turner syndrome 3. Asherman 4. Imperforate Hymen 5. Androgen insensitivity syndrome 6. PCOS 7. Sheehan syndrome 8. Kallmann 9. Prolactinoma 10. Pregnancy

What is the most likely cause of amenorrhea if associated with each of the following statements? 1. Ballet dancer with eating disorder 2. Short stature, low set ears, and webbed neck 3. Following D+C after spontaneous abortion 4. Cyclic pelvic pain + blue bulge at introitus on exam 5. Elevated testosterone level and no axillary or pubic hair on exam 6. Overweight female with acne and hirsutism 7. Postpartum patient with inability to breast feed 8. Patient with anosmia 9. Patient with galactorrhea 10. Most common cause of amenorrhea

1. Functional Hypothalamic amenorrhea 2. Turner syndrome 3. Ashermann syndrome 4. Imperforate Hymen 5. Androgen Insensitivity 6. PCOS 7. Sheehan's 8. Kallmann 9. Prolactinoma 10. Pregnancy

A 21 year old man with a history of HIV infection is brought to the emergency department with headache and fever. LP shows an opening pressure of 300 mm H20 (normal < 200). Analysis of the CSF reveals low glucose and mildly elevated protein. A small number of lymphocytes is seen in the CSF and no RBCs are seen. 1. What is the most likely dx? 2. What tests should be confirmed the diagnosis? 3. What is the most appropriate tx?

1. Fungal meningitis - cryptococcus neoformans 2. India ink, cryptococcal antigens 3. Amphotericin B and Flucytosine for 2 weeks Fluconazole then for 8 weeks

A patient presents with profound fatigue after being treated for a UTI. The CBC shows normocytic anemia, and examination of the peripheral blood smear demonstrates the presence of Heitz bodies and degmacytes (bite cells) 1. What is the most likely cause of this patient's anemia? 2. What drugs will provoke this anemia?

1. G6PD deficiency 2. Anitmalarials: primaquine, chloroquine Nitrofurantoin Sulfonamides: Sulfamethaxole Dapsone: Leprosy and PCP ppx Isoniazid: used for TB Ibuprofen High dose ASA Fava Beans

A 15 year old boy is brought to the physician by his mother by tingling and weakness in his legs for the last week. He is trying out for his school's basketball team this season and has recently started on an exercise program, but he has noticed decreased coordination and difficulty with jogging. Last month, he was admitted to the hospital for two days for gastroenteritis. Physical examination reveals absent bilateral patellar and Achilles reflexes. Motor strength is 3/5 in lower extremities. Sensation is intact. The remainder of the examination reveals no abnormalities 1. What is the most likely dx? 2. What will be found on analysis of the CSF? 3. What is the most appropriate treatment?

1. GBS 2. Increase protein with normal cell count 3. ICU monitoring Plasmapharesis or IVIG

What is the treatment for gonorrhea? What is the treatment for chlamydia?

1. GC - IM Ceftriaxone + oral azithromycin x1 2. Ch - oral azithromycin x1 or Doxycycline x 7d PO

A 72 year old woman presents with a 2 day history of right sided headache and vision disturbances. She also complains of jaw pain when chewing. Her temperature is 38.1. On examination, there is tenderness to palpation on the right side of the scalp. 1. What treatment should be started even before the dx is established? 2. What test is the most appropriate first test to order? 3. What test is most likely to confirm the dx?

1. GCA - high dose steroids 2. Sed rate 3. Temporal artery biopsy

Most common causes of CAP 1. Neonate 2. Infant-5 years 3. Older children - adult 4. Elderly

1. Group B strep, E. coli 2. RSV, S. pneumoniae, S. aureus 3. S. pneumoniae, H. influenzae, M. pneumoniae, C. pneumoniae, Influenza 4. S. pneumoniae, H. influnezae, Influenza, S. aureus, Anaerobes, Gram negative rods

What is the most likely cause of LOC in each of the following scenarios? 1. High school athlete loses consciousness during a football game 2. Elderly man loses consciousness only when wearing a tie and a tight fitting collar 3. LOC + loss of bladder control + delayed return to bseline 4. LOC during venipuncture + diaphoresis, pallor, bradycarida 5. Gradual LOC with diaphoresis and tachycardia in a type 1 diabetic 6. Gradual LOC with hyperglycemia and abdominal pain in a type II diabetic

1. HOCM 2. Carotid sinus 3. Seizure 4. Vasovagal 5. Hypoglycemia 6. HHS

What does each of the following heart rate patterns indicate? 1. Early deceleration 2. Variable deceleration 3. Late deceleration 4. Sinusoidal

1. Head compression 2. Umbilical cord compression 3. Fetal Hypoxia 4. Severe Fetal Anemia

Answer the following questions regarding heat-related illness 1. What are the dx criteria for heat exhaustion and exertional heat stroke? 2. How is core body temperature measured for the dx and mngt of heat-related illness?

1. Heat exhaustion - difficulty with exercise, core temperature 101-104F, no CNS dysnfuction Heat stroke - >104F, CNS dysfunction 2. Rectal Thermometer

A 7 year old girl is brought to the ED with dark urine and jaundice. She has had 1 week of vague abdominal pain, occasional vomiting, and bloody diarrhea. Physical examination shows no dehydration. Abdominal examination reveals no distension, masses, tenderness, or guarding. There are some petechiae on the extremities. Teh peripheral blood smear reveals schistocytes and thrombocytopenia. 1. What is the most likely dx? 2. What organism is most likely implicated?

1. Hemolytic Uremic Syndrome 2. E. coli O157:H7 infection predispose to illness - MAHA - Thrombocytopenia - AKI Labs: Increase LDH, Indirect bilirubin, Decrease haptoglobin, coombs test negative Tx: Supportive, RBC/plt transfusion, dialysis, plasma exchange, ecelizumab

What malignancy is most closely associated with each of the following tumor markers? 1. alpha-fetoprotein (AFP) 2. Alkaline phosphatase 3. CA-125 4. CA 19-9 5. CEA 6. PSA 7. S-100

1. Heptaocellular cancer, testicular ca, gastric cancer 2. bone mets (paget's) 3. Ovarian CA 4. Pancreatic CA 5. Colon CA + Pancreatic CA 6. Prostate CA, BPH, Prostatitis 7. Melanoma, Schwannoma

Reed-sternberg cells Starry sky appearance Smudge cells Auer Rods

1. Hodgkin's (owl-eye) 2. Burkitt lymphoma - EBV 3. CLL 4. AML

A 32 year old woman complains of body stiffness and muscle spasms. She recently had a thyroidectomy for hyperthyroidism. The patient displays ipsilateral contraction of the facial muscles when tapping the cheek just anterior to the ear. 1. What is the most likely cause of these sx? 2. What is the most apprpropriate tx?

1. Hypocalcemia due to parathyroid glands removal 2. Calcium (IV or oral) Vitamin D Calcitriol rHPTH

What important side effects are associated with each of the following classes of antihypertensives? 1. Thiazides 2. CCBs 3. ACE-i 4. ARBs 5. Beta blockers 6. Alpha1-blockers 7. Alpha2-blockers 8. Aldosterone antagonists 9. Vasodilators - hydralazine 10. Vasodilator - minoxidil 11. Vasodilator- nitroprusside

1. Hypokalemia, Hyponatremia, Hyperuricemia 2. Headache, hypotension, peripheral edema 3. Cough, angioedema, rising BUN/Cr, hyperkalemia, teratogenic 4. Same as ACE inhibitors except no cough and angioedema 5. Bronchoconstriction (nonselective - propanolol), masks symptoms of hypoglycemia, bradycardia 6. 1st dose orthostatic hypotension, rebound hypertension 7. Somnolence, Dizziness, orthostatic hypotension, Rebound HTN 8. Hyperkalemai, Anti-androgen 9. All can cause headache, edema, reflex tachycardia - HA, Edema, Drug-induced lupus 10. Hypertrichosis, Hirsutism 11. CN toxicity

What medical conditions and drugs can cause depressive sx? Medical illness Drugs

1. Hypothyroidism, Hyperparathyroidism, Parkinsons, HIV, stroke, cancer (CNS neoplasms) 2. Sedatives (alcohol, BZs, antihistamines) Withdrawal from stimulants (cocaine, ampethamines) Some antihypertensives (methyldopa, clonidine, Beta blockers) First generation antipsychotics (Haldol) Prochloperazine Metoclopramide Long-term glucorcorticoid use IFN-alpha

A 47 year old obese man comes to the physician for sleeping problems. Pt's wife reports that he snores very loudly and stops breathing several times at night. 1. What are some clinical manifestations of OSA? 2. What are the risk factors? 3. What is the first-line dx study for OSA? 4. What are the treatments for OSA?

1. Hypoxia, Hypercapnia, Fragmented sleep, poor concentration 2. Obesity. 20-30% male, 10-20% female, advancing age, female, upper airway abnormalities, smoking, nasal congestion 3. Polysomnography: EEG, EKG, puse ox, arousal, hypocapnea 4. behavior, sleep positioning, CPAP, Oral devices, Surgery

A 23 year old woman comes to the clinic for intermittent abdominal bloating and crampy lower abdominal pain over the last six weeks. There is no nausea or vomiting. She alternates between diarrhea and constipation, and defecation often relieves the abdominal pain. There is no blood or mucus in the stools. She has had no weight loss. She recently broke up with her fiancee and is staying with friends. The physical examination, labs, imaging, and colonoscopy are all normal. 1. What is the most likely dx? 2. What initial mngt should be considered for this patient?

1. IBS 2. Initially - Avoid gas-producing foods - Diet low in fermentable short chain sacchardies - gluten free diet - high fiber diet - exercise

A 21 year old woman comes to the clinic because of recurrent gingival bleeding for the past 3 weeks. She has had two episodes of epistaxis during this time, and she has noticed a painless, non-pruritic rash on her ankles and lower legs. She denies any heavy menstrual bleeding. She has no significant medical history and takes no medications. Physical examination reveals a healthy appearing woman. There is no visible bleeding or abnormality of the oral cavity. There are numerous petechiae on her lower extremities. There is no HSM. Lab eval shows a plt count of 40,000. PT and PTT are both normal. 1. What is the most likely dx? 2. What treatment options are available?

1. ITP 2. No tx if asymptomatic Plt transfusions Oral gluocorticoids IVIG Severe: splenectomy

What are the criteria for prophyalxis and prophylactic agents used for the following HIV opportunistic infections? 1. Cryptococcus neoformans meningitis 2. Histoplasma capsulataum 3. Mycobacterium avium complex 4. PCP pneumonia 5. Toxoplasma gondii 6. Candida albicans

1. IV Amphotericin + flucytosine 2. Itraconazole 3. Azithromycin 4. Bactrim, alternatives: dapsone 5. Bactrim 6. Fluconazole, nystatin

A 32 year old woman comes to the physician with a 2 week history of bilateral arm weakness. She has difficulty lifting her arms to fix her hair in the morning, and it is getting progressively worse. She denies muscle soreness or pain. Physical examination shows modest difficulty rising from a chair and climbing onto the examination table. She has 2/5 motor strength of shoulder abduction and 3/5 motor strength of hip flexion and extension. Handgrip strength and the rest of the neurological exam are normal. She is noted to have symmetrical, pinkish-red papules on the dorsal surfaces of MCP joints and IP joints of both hands. Further examination of the skin reveals hyperpigmentation of the shoulders and anterior upper chest. 1. What serum markers are likely to be present in this patient? 2. What other dx studies would help establish the diagnosis? 3. How is this condition treated?

1. Increase creatine kinase, Increase aldolase, +ANA, +AntiJo Abs and normal sed rate - dermatomyositis 2. EMG, Muscle biopsy 3. High dose prednisone Increase risk of malignancy: 15% for polymyositis, 10% for dermatomyositis

How should pregnant patients be counseled regarding each of the following? 1. Smoking 2. Alcohol 3. Exercise 4. Travel 5. Sex

1. Increase risk of placental abruption, preterm birth, low birth weight, SIDS - smoking cessation: nicotine, wellbutrin 2. Teratogenic -> fetal alcohol syndrome - growth abnormalities, CNS problems, facial abnormalities 3. At least 30 min/day. Avoid abdominal trauma and falls 4. No plans after 36 weeks, avoid prolonging immobilization 5. OK except placenta previa

Define the following terms: 1. AKI 2. Chronic kidney disease

1. Increase serum > 0.3 mg/dL w/in 48 hrs Increase creatinine > 50% w/in 7 days Urine output <0.5 mL/kg/hr for at least 6 hrs 2. 1 of the following for >3 months - eGFR < 60 - urine abnormalities: casts

What is the most appropriate ethical response to each of the following situations? 1. A physician orders an invasive test for the wrong patient. 2. A patient tells the physician that he/she finds the physician attractive and wants to start dating. 3. An elderly woman is found to have inoperable lung cancer, and her family asks the physician to tell the patient the biopsy was negative. 4. You suspect another physician of practicing while under the influence of alcohol.

1. Inform patient a mistake has been made 2. Keep relationship professional. Use a chaperone if necessary 3. Do not Lie. Determine family's motives. Ask if pt wants to know 4. Report to Physician's superior

A 56 year old lifelong nonsmoker comes to the physician with intermittent shortness of breath, wheezing, and cough. An x-ray of the chest demonstrates hyperinflation of the lungs and flattening of the diaphragm. Office spriometry shows FEV1/FVC ratio of 0.65 which is not reversible with inhaled albuterol. Lab evaluation shows elevated ALT and AST, and a liver biopsy confirms the presence of early cirrhosis. 1. What disease-specific therapy could be used to treat his pulmonary disease? 2. What treatment would potentially cure this condition?

1. Infusion of alpha-1 antitrypsin Lung Transplant 2. Liver transplant

What is the mechanism of action of each of the following drugs Aspirin Clopidogrel Heparin Rivaroxaban Tirofiban Warfarin

1. Inhibits COX1 + 2 => inhibits thormboxane synthesis and platelet aggregation 2. Inhibits plt aggregation 3. +Antithrombin => inhibits thrombin and factor Xa 4. Direct Factor Xa inhibitor 5. Inhibits plt aggregation and crosslinking with fibrinogen 6. Inhibits synthesis of Vit K dependent coag factors 2, 7, 9, and 10.

What is the mechanism of action of each of the following drugs Aztreonam Azithromycin Doxycycline Dicloxacillin Moxifloxacine Trimethoprim

1. Inhibits cell wall synthesis by inhibiting peptidoglycan crosslinking 2. Binds to 50S ribosomal subunit and inhibits protein synthesis 3. Binds to 30S ribosome subunit and inhibits protein synthesis 4. Inhibits cell wall synthesis by inhibiting peptidoglycan crosslinking 5. Topoisomerase II and IV inhibitors 6. Inhibits DHF reductase and interferes with folate metabolism and DNA synthesis.

Fusion inhibitors Enfuvirtide Maraviroc

1. Injection only 2. Acts on CCR5

A 34 year old G4P3 woman at 34 weeks gestation comes to the physician because of a 1 week history of itching all over her body but especially on her palms and soles. The patient says that the itching is worse at night. Examination of the skin shows multiple excoriations but no rash. Lab studies show elevated total serum bile acid levels 1. What is the most likely dx? 2. What is the appropriate management?

1. Intrahepatic cholestasis of Pregnancy 2. Ursodeoxycholic acid Hydroxyurea Deliver at 37-38 weeks

Five steps of management of unknown overdose/poisoning 1. Airway = 2. Breathing = 3. Circulation = 4. Disability = 5. Elimination =

1. Intubate if not breathing or unable to protect airway 2. Supplemental Oxygen 3. Arrhythmia -> ACLS protocol Hypotension -> IVF, NE 4. Opioid overdose => Naloxone Hypoglycemia => D50 + Thiamine Seizures => BZs (phenytoin will not work for toxins) 5. Activated charcoal for ingestion of pills (ASA) Not recommended: Lavage, Iepac, Cathartics

What is the most likely cause of anemia in each of the following presentations? 1. Anemia with restless leg syndrome 2. Anemia in a patient who eats ice 3. Anemia that develops after taking a sulfa drug 4. Microcytic anemia in an alcoholic 5. Microcytic anemia in a patient with rheumatoid arthritis 6. Microcytic anemia with neurological sx 7. Megaloblastic anemia with neuro sx 8. Megaloblastic anemia w/o neuro sx

1. Iron deficiency 2. Iron def (PICA) 3. G6PD deficiency 4. sideroblastic anemia 5. Anemia of chronic disease 6. Lead poisoning 7. B12 def 8. folate def

A 7 year old girl has had 10 day history of fever with a temperature as high as 103.5F. She has bilateral eye redness, cracked red lips, and swelling of her hands. In addition to these complaints, physical examination reveals cervical lymphadenopathy. What is the most likely dx? What is the treatment?

1. Kawasaki disease: Fever 5 days or longer, polymorphous rash, oral mucous membrane changes, strawberry tongue, cracking lips, peripheral extremities has redness and edema, desquamation, cervical lymphadenpathy >1.5 cm in diameter unilateral 2. IVIG, High dose ASA

Location of MI 1. I, aVL, V5 and V6 2. II, III and aVF 3. V2, V3, V4, V5 4. V1, V2, V3

1. Lateral wall; LAD or circumflex artery 2. Inferior wall; posterior descending artery 3. Anterior wall; LAD 4. Septum; LAD

What is the most likely cause of anemia in each of the following situations? 1. Microctyic anemia with neurological sx 2. Megaloblastic anemia with neuro sx 3. Anemia in a patient who eats ice 4. Anemia that develops after taking a sulfa drug 5. Anemia in a patient with rheumatoid arthritis

1. Lead poisoning [foot drop, wrist drop, headache, confusion] 2. B12 deficiency (w/o neuro sx -> folate def) 3. PICA - iron def. 4. G6PD def. 5. Anemia of chronic disease

A 61 year old woman comes to the clinic complaining of discomfort in her legs at night. This sensation is relieved by movement. she is diagnosed with restless legs syndrome. 1. What tx options are available for patients with intermittent sx? 2. What tx options are available for patients with persistent sx?

1. Levodopa, Benzos 2. Pramipexole, Ropirinole, Gabapentin, Pregabalin

What teratogenic agent is associated with each of the following treatments? 1. Ebstein cardiac anoamly 2. Clear cell adenocaricinoma of the vagina 3. Microcephaly, intellectual disability, smooth philtrum 4. Gray baby syndrome 5. Phocomelia (malformation of the limbs) 6. Craniofacial anomalies, fingernail hypoplasia, developmental delay

1. Lithium 2. DES 3. Alcohol 4. Chloramphenicol 5. Thalidomide 6. Phenytoin

What teratogenic agent is associated with each of the following statements? 1. Ebstein cardiac anomaly 2. Clear cell adenocarcionma of the vagina 3. Microcephaly, intellectual disability, smooth philtrum 4. Gray Baby syndrome 5. Phecomelia (malformation of the limbs) 6. Craniofacial anomalies, fingernail hypoplasia, developmental delay

1. Lithium 2. DES 3. EtOH 4. Chlormaphenicol 5. Thalidomide 6. Phenytoin

A 56 year old woman has routine labwork done to evaluate her chronic type 2 DM. Her AST and ALT are moderately elevated (245 and 278, respectively). Further studies show that she is negative for hep B and hep C. serum ferritin is 540 and transferrin saturation is 72%. 1. What further tests would help confimr the dx of herditary hemochromatosis? 2. What is the most appropriate treatment?

1. Liver biopsy HFE gene mutation 2. Phlebotomy

What diuretic (or diuretic class) would be most useful in each of the following situations? 1. acute pulmonary edema 2. idiopathic hypercalicuria causing calcium stones 3. Glaucoma 4. Mild to moderate CHF and expanded extracellular volume 5. Diuretics used in conjunction with loop or thiazide diuretics to retain potassium 6. Edema associated with nephrotic syndrome 7. Increased intracranial pressure 8. Hypercalcemia 9. Altitude sickness 10. Hyperaldosteronism

1. Loop 2. Thiazide 3. Acetazolamide/Mannitol 4. Loop/Aldosterone antagonist 5. K sparing diuretics 6. Loop 7. Mannitol 8. Loop 9. Acetazolamide 10. Spirinolactone, Eplernone

Iron studies in anemia Iron deficiency AOCD Thalassemia Lead poisoning Sideroblastic anemia

1. Low MCV, Low Iron, Low Ferritin, High TIBC, Low % transferrin sat 2. Nl or low MCV, Low Iron, Nl or High Ferritin, Low TIBC, Normal transferrin sat 3. Low MCV, Normal or high Iron, Normal or High Ferritin, Normal or Low TIBC, Nl transferrin sat 4. Low MCV, Normal or high Iron, Normal Ferritin, Normal TIBC, Normal % transferrin 5. Low MCV, High Iron, High Ferritin, Low TIBC, Normal transferrin saturation

A 71 year old woman presents to her physician with leg pain when walking. The pain began about a year ago and is gradually worsening. She is generally very cactive and still works her family famr. She has pain when walking downhill, but her pain improves when she walks uphill. Sitting or lying down relieves the pain. Pysical exam shows strong, equal bilateral pulses in the feeet and ankles. The feet arm warma nd well perfused. DTRs are absent in the ankles and 1+ at the knees. There is no motor weakness or sensory deficit. straight leg test is negative. What imaging modality is most likely to confirm the diagnosis? How is this condition treated?

1. Lumbar spinal stenosis - MRI 2. Mild - PT, NSAIDs Severe: surgical decompression

What pathology is associated with each of the following murmurs? 1. Opening snap + rumbling late diastolic murmur at the apex 2. Bound radial pulse + early diastolic murmur along the left sternal border 3. Midsystolic click followed by a late systolic murmur 4. Weak radial pulses + crescrendo/decrescendo systolic murmur at the RUSB radiating to the carotids 5. Holosystolic murmur at the apex radiating to the left axilla

1. MS 2. AR 3. MVP 4. AS 5. MR

a 36-year-old man with bipolar disorder is brought to the emergency department by his wife. She reports that he has not slept for the past 3 days, and he has embarked upon and ambitious and expensive landscaping projects that he has work done continuously during that time. He has incurred over 10,000 dollars in critical charges on equipment and materials in the last 2 days. He toxic slightly about this project and is convinced that his yd will order him National media attention and a chance to host a reality TV series. What is the most likely diagnosis? What are currently considered first-line drugs for this condition?

1. Manic episode 2. Lithium Anticonvulsants: valproic acid, carbamazepine Atypical antipsychotics: olanzapine, quetiapine, aripripazole, risperidone

How do we screen for and manage the chronic complications of diabetes 1. Hyperglycemia 2. Hypertension 3. Hyperlipidemia 4. Nephropathy 5. Retinopathy 6. Peripheral neuropathy 7. Gastroparesis 8. Autonomic neuropathy (hypotension, ED)

1. Measure HbgA1c; mngt - blood glucose control, lifestyle modification, medication 2. Measure BP every visit; mngt - lifestyle mod if BP > 120/80, medication if BP > 140/90 3. Measure fasting lipids at least annually; statin if age >40 or any CVD risk factors 4. Measure urine microalbumin:creatinine ratio annually; ACE-i/ARB 5. Annual dilated eye exam; intravitreal injections or laser photocoagulation 6. Annual foot exam; pregabalin, duloxetine 7. None; metoclopramide, erythromycin may be used short term 8. None; as indicated

What is the most likely cause of headache based on each of the following descriptions? 1. Made worse by foods containing tyramine 2. Obese women with papilledema 3. Jaw muscle pain when chewing 4. Periorbital pain with ptosis and miosis 5. Photophobia and/or phonophobia 6. Most common cause of headache 7. Lacrimation and/or rhinorrhea 8. Elevated ESR 9. Worst HA of my life 10. Scintillating scotomas prior to HA 11. Responsive to 100% oxygen supplementation

1. Migraine 2. Idiopathic intracranial hypertension 3. GCA 4. Cluster HA 5. Migraine 6. Tension 7. Cluster 8. GCA 9. SAH 10. Migraine 11. Cluster

What is the treatment for each of the following pulmonary infections? 1. Histoplasmosis 2. Blastomycosis 3. Coccidioidomycosis

1. Mild to moderate: itraconazole. Severe: Ampho B 2. Mild to moderate: itraconazole. Severe: Ampho B 3. Fluconazole, Ketoconazole, or Itraconazole

Pneumothorax - Spontaneous - Secondary - Tension

1. More often in tall, thin young males 2. Occurs with existing lung pathology 3. One way valve mechanism leading to increased pressure on affected side and displacement of mediastinum to opposite side.

A 33 year old woman is brought to the ER by ambulance for difficulty breathing. She has had ptosis and difficulty swallowing for the last sxi months. Administration of edrophonium results in temporary resolution of her ptosis. Over the next 12 hours, the patient's condition deteriorates to resp failure requiring mechanical ventilation. What is the most likely dx? What is the most appropriate tx for this disorder?

1. Myasthenia Gravis 2. Cholintesterase-inhibitor (pyridostigmine) SEvere: plasmapharesis, IVIG Chronic immunosuppression (prednisone, azathioprine)

a 47-year-old man presents with abdominal pain for the last 4 days. He has not had a bowel movement in the last 5 days. He had a ventral hernia repair 8 Years ago. Physical examination shows abdominal distension and hyperactive bowel sounds characterized by a high pitch quality and rushing sounds. X-ray of the abdomen shows multiple air-fluid levels in small bowel loops. What is the most appropriate initial management for this patient? What would be the most appropriate next step if the patient develops signs of peritoneal inflammation?

1. NPO, NG suction, IV fluids 2. laparotomy

Which antiretroviral class is known for each of the following adverse effects? 1. Lactic acidosis 2. Elevated glucose and lipids 3. Rash 4. Diarrhea

1. NRTIs 2. Protease-inhibitors 3. NNRTIs 4. Protease-inhibitors

What condition is suggested by each of the following abnormalities on second trimester maternal serum screening? 1. Increase AFP 2. Decrease AFP, Decrease estriol, Increase hCG 3. Decrease AFP, Decrease estriol, Decrease hCG

1. NTD, abdominal wall defects, multiple gestation, most common is incorrect dating 2. Down syndrome (inhibin increased) 3. Edward syndrome

A 6 day old male infant develops fever, irritability, and an erythematous rash around the mouth. One day later, the rash generalizes and flaccid blisters appear. The upper layer of skin begins to slough off, especially when gentle lateral pressure is applied to the skin. 1. What is the most likely causal organism? 2. What is the treatment?

1. Nikolsky sign SSSS, mucous membranes not involved 2. IV nafcillin, oxacillin Supportive care: emoillents, IVF, correction of electrolyte abnormalities

Identify the core ethical principle described by each of the following statements. 1. A physician refuses to administer abx to a patient with a viral infection because of the high risk of dangerous side effects 2. A physician allows a cancer patient to choose between two acceptable and equally effective treatment plans. 3. A competenet patient refuses therapy for a life threatening condition. 4. A working-class patient with end stage liver disease is given higher priority for a liver transplant than an internattionally renowned actor whose liver disease is less advanced.

1. Nonmaleficience 2. patient autonomy 3. pt autonomy 4. Justice

What should the first line vasoactive medication for treating shock in each of the following patients? 1. Severe hypotension, fever, resp failure due to meningococcemia 2. Left ventricular failure from dilated cardiomyopathy 3. Hives, wheezing, and hypotension following administration of PCN

1. Norepinephrine and IV fluids 2. Dobutamine/Dopamine/Norepi 3. Epinephrine

What brain tumor is associated with each of the following descriptions? 1. Rare, slow growing, often found in the frontal lobe 2. Benign; most common childhood supartentorial tumor 3. Most common malignant primary brain tumor in adults; rapidly progressive 4. Most commonly secretes prolactin; may cause bitermporal hemianopia 5. Highly malignant cerebellar tumor in children 6. Common primary brain tumor, typically benign

1. Oligodendroglioma 2. Craniopharyngioma 3. Glioblastoma 4. Pituitary adenoma 5. Medulloblastoma 6. Meningioma

A 24 year old woman is brought to the physician because of progressive vision loss in her right eye over the past day. She also complains of right eye pain that is worse with eye movement. Examination shows visual acuity of 20/20 in the left eye and 20/100 in the right eye. There is a loss of color vision in the affected eye. When a light is shone into the left eye, both pupils constrict equally. When the light is then shone into the right eye, both pupils dilate. What is the most likely diagnosis? What study should be obtained in order to confirm the dx?

1. Optic neuritis - Demyelination of optic nerve - Associated with MS - Painful monocular vision loss, relative afferent pupillary defect (Marcus Gunn pupil) 2. MRI of brain and orbits with gadolinium contrast Tx: IV corticosteroids

A 21 year old competitive diver comes to the clinic because of left ear pain. The hearing is somewhat diminished on the left. There is no fever, sore throat, nasal congestion, or nasal discharge. Upon examination, the patient reports pain when the left pinna is manipulated. The external ear canal is edematous and erythematous. There is scant, purulent debris in the canal. the TM is normal. 1. What is the most likely dx? 2. What is the most appropriate treatment for this condition?

1. Otitis Externa 2. Clean ear canal, topical abx drops (FQs, polymyxin B+ neomycin, aminoglycosides Topical steroids (hydrocortisone, dexamethasone) Analgesics

Answer the following questions regarding chorioamnionitis 1. What are the risk factors 2. What is the treatment

1. PROM, Prolonged Labor, Multiple cervical exams, Meconium Fluid, Internal monitors (fetal scalp electrode, IUP catheter) 2. IV ampicillin + Gentamicin

A 60 year old man comes to the clinic with complaints of left knee pain, thigh pain, and back pain. He also mentions offhand that his favorite that seems too tight. The physical examination is normal. An x-ray of the knee shows a lytic lesion in the distal left femur. Lab studies show normal serum calcium, phosphate, creatinine, and hemoglobin. The serum alkaline phosphatase is elevated. 1. What additoinal test would be most appropriate to confirm the dx? 2. What is the most appropriate treatment for this condition?

1. Paget's Disease of Bone -> radionuclide bone scan. Increase osteoclasts, Axial and long bones 2. Bisphosphonates (zoledronic acid)

A 67 year old man comes to the clinic with tremor and difficulty walking that has been worsening for about two years. He tends to shuffle his feet when walking, and often finds it difficult to take the first step. He has been falling frequently for 3-4 weeks. Physical examination reveals normal cognition and affect, but limited facial expression. There is a resting tremor in the right hand. There are increased muscle tone and cogwheel rigidity in the extremities. There are no obvious cerebellar deficits. 1. What neurological disorder is most likely causing this patient's sx 2. What is the most appropriate initial therapy? 3. What surgical procedure could be considered for advanced disease?

1. Parkinson's disease 2. Levodopa/Carbidopa 3. Deep Brain Stimulation

AED meds 1. Block Na+ channels 2. Increase GABA activity 3. Block T-type Ca channels

1. Phenytoin, Carbamazepine, Lamotrigine, Topiramate, Valproic acid 2. Gabapentin, Phenobarbital, Valproic acid, BZs, Levetiracetam 3. Gabapentin, Ethosuximide, Phenytoin (high dosages)

AED drug adverse reactions 1. Teratogens 2. SJS 3. Agranulocytosis

1. Phenytoin, Carbamazepine, Valproic acid 2. Ethosuximide, Lamotrigine, Carbamazepine, Phenobarbital, Phenytoin 3. Carbamazepine (clozapine, colchicine, ptu, methimazole)

An 8 year old boy develops a 3 cm erythematous plaque on his chest. One week later, he has the onset of multiple salmon-colored, oval plaques in a "Christmas tree" pattern over his chest and back. 1. What is the most likely diagnosis? 2. What is the treatment?

1. Pityriasis Rosea HHV-8 or HHV-7 Herald patch => pink oval xmas tree 2. Self-limited, Reassurance Acyclovir may be helpful

What term best matches each of the following statements? 1. Implantation of placenta over the cervical os 2. Fetal blood vessels overlie the cervical os 3. Abnormal adherence of the placenta to the myometrium 4. Invasion of the placenta into the myometrium 5. Invasion of the placenta through the myometrium 6. Premature separation of the placenta from the uterus before delivery

1. Placenta Previa 2. Vasa Previa 3. Placenta Accreta 4. Placent Increta 5. Placenta Percreta 6. Abruptio Placentae

A 65 year old man comes to the clinic for follow up of a recent stroke. He was admitted to the hospital three weeks ago for an acute thrombotic stroke that caused slurred speech and facial weakness, and left arm numbness and paralysis. He did not receive thrombolytics. Evaluation showed a 50% stenosis of the right internal carotid artery. At discharge, his neurological deficits were improving significantly, and he has continued outpatient physical therapy and speech therapy. He currently only has weakness of the left handgrip 1. What is the most appropriate antithrombotic therapy at this time? 2. How should his carotid artery stenosis be managed?

1. Plavix, ASA + Aggrenox, or ASA 2. CEA - Sx patients with 70-99 percent - Sx men w 50-69 percent - Asx patients with 60-99 percent with life expectancy of at least 5 years

You have admitted a 62 year old woman to the inpatient ward for treatment of a DVT in the right thigh. There is no hx of recent travel, recent surgery, or immobilization. Upon detailed questioning, the patient admits to intermittent stinging or burning pain in the hands sometimes associated with redness. She also complains of intense pruritus when taking a warm bath. Hbg concentration is 18.1. Serum EPO is slightly low. Lab tests for coag factor deficiencies and mutations are negative. 1. What underlying disorder most likely predisposed to this patient's DVT? 2. Aside from treating her DVT, how should this underlying disorder be managed?

1. Polycythemia Vera - r/o hypoxemia - check BM biopsy 2. Phlebotomy Hydroxyurea Low-dose ASA Antihistamine

Hepatitis B serologies HBsAg HBsAb HBcAb HBeAg HBeAb

1. Positive with active infection 2. Positive with recovery or after vaccination 3. Positive with history of Hep B Infection (IgM/IgG) 4. Positive indicates high transmissibility 5. Positive indicates low transmissibility

Ischemic heart disease 1. Stable angina 2. Unstable angina 3. NSTEMI 4. STEMI

1. Predictable CP w exertion 2. Unpredictable CP with exertion 3. Increased cardiac enzymes w/o ST segment elevation 4. Increased cardiac enzymes w ST segment elevation STEMI - 2 mm in males, 2.5 mm in females

A 44 year old woman comes to the clinic with a 3 month history of generalized pruritus and fatigue. She also has longstanding problems with dry mouth and dry eyes, although a previous evaluation for Sjogren syndrome was inconclusive. Physical examination shows mild hyperpigmentation of the skin but no jaundice or scleral icterus. Her liver is moderately enlarged but nontender. Lab testing shows minimally elevated AST and ALT, markedly elevated alkaline phosphatase, and normal total bilirubin. ANA and antimitochondrial abs are positive. 1. What test would confirm the diagnosis? 2. What FDA-approved drug may be used to tx this condition?

1. Primary Biliary Cirrhosis - Liver Biopsy 2. Ursodeoxycholic Acid

A 39 year old man comes to the physician with pruritus and RUQ abdominal pain for the last 2 weeks. He has a 15 year history of UC. Physical examination shows mild jaundice but no other abnormalities. Lab studies show an elevated direct bilirubin and alkaline phosphatase. pANCA are strongly positive. ERCP shows "beading" of the biliary system. 1. What is the most likely dx? 2. What treatment options are available for this disorder? 3. What malignancies is this patient at increased risk of developing?

1. Primary sclerosing cholangitis (IBD, UC) p-ANCA 2. No medical therapy. ERCP for a dominant stricture. Liver transplants 3. Cholangiocarcionma GB carcinoma Hepatocellular CA

Answer the following questions regarding animal bites 1. What empiric abx ppx is used for cats and dog bites? 2. What empiric abx treatment is used for an infected cat or dog bite?

1. Prophylaxis: Augmentin, Pen VK, Cefuroxime, Doxy, Moxifloxacin, Bactrim plus Metronidazole or Clindamycin 2. Tx: wound care. wound culture Ampicillin-sulfbactam Pip/tazo ticarcillin-clavalunate Rocephin + flagyl FQ + flagyl Imipenem - clistatin Meropenem Ertapenem

A 54 year old man with chronic hepatitis C virus infection develops skin blisters whenever he goes out in the sun 1. What is the most likely diagnosis? 2. What is the management?

1. Prophyria Cutanea Tarda - defect in uroporphylinogen decarboxylase => elevated levels of uroporphyingen in blood + urine 2. Avoid triggers like alcohol, estrogens, polyhalogenated hydrocarbons Phlebotomy Chloroquine Tx underlying cause

A 65 year old man with a hx of CAD and BPH, comes to the physician to discuss tx options for erectile dysfunction. 1. What are the contraindications to using a PDE-5-inhibitor such as sildenafil? 2. What are the side effects?

1. Pts who take NTG, and alpha blocker 2. Flushing, headache, nasal congestion, blue vision

Answer the following questions regarding syphilis 1. What nonspecific lab studies are used in the screening of syphilis? 2. What specific lab studies are used in the confirmation of syphilis? 3. What lab studies cna be used to identify the spirochetes in syphilis?

1. RPR, VDRL screen 2. FTA-ABS, MHA-TP to confirm 3. Dark field microsopy of chancre Fluorescent antibody testing of syphilic lesion

A 21 year old male college student comes to the clinic with weakness and numbness in his right hand, as though his hand is asleep. It began two days ago following a party. He remembers a lot of drinking, and then he woke up with his right arm over the back of a chair. He cannot extend the right wrist or fingers 1. What nerve is most likely affected 2. What is the treatment?

1. Radial nerve palsy 2. No tx needed

Which seronegative spondyloarthropathy matches each of the following statements? 1. Asymmetric arthritis preceded by GI or GU Infection 2. Inflammatory back pain + "bamboo spine" on x-ray 3. Skin plaques with silver scales + pitting of the nails + arthritis

1. Reactive arthritis (Reiter's) 2. Ankylosing spondylitis 3. Psoriatic arthritis (pencil-in-cup fingers)

Identify each of the following types of biases that may be present in research studies 1. Individuals who experience an adverse outcome have a different likelihood of recalling an exposure than those without the adverse outcome. 2. Early detection may be confused with an increase in survival 3. The association between and exposure and an outcome is distorted by another variable 4. Subjects in different arms of a study are systematically treated differently 5. Information is distorted by the way it is gathered

1. Recall bias 2. lead-time bias 3. confounding bias 4. procedure bias 5. measurment bias

Asthma medications 1. Rapid acting B2 agonists (albuterol) 2. Long acting B2 agonists (salmeterol) 3. Inhaled corticosteroids 4. Leukotriene inhibitors 5. Mast cell stabiizers 6. Theophylline 7. Anticholinergic agents 8. systemic steroids

1. Relax airway smooth muscle; side effect of tachycardia 2. Relax airway smooth muscle; side effect of tachycardia 3. Decrease airway inflammation; side effect of oral candidiasis 4. Decrease airway inflammation; used with allergic rhinits 5. Decrease airway inflammation; multiple daily dosing 6. Relax airway smooth muslce; side effects of hypotension, seizure, and tachycardia 7. Relax airway smooth muscle; most effective for acute exacerbations

What causes secondary HTN?

1. Renal artery stenosis - Dx: MR angiogram, spiral CT with contrast, duplex renal u/s - Tx: Angioplasty/stenting 2. CKD or ESRD 3. Drugs (OCPs, NSAIDs, antidepressants, glucocorticoids) 4. Hypercortisolism (Cushing) - Dx: Cortisol, ACTH, Dexamethasone suppression test 5. Primary hyperaldosteronism (conn syndrome) - Triad of HTN, hypokalemia, met alkalosis - Dx: PAC:PRA ratio (will be high) 6. Pheochromocytoma - Episodic hypertension with diaphoresis, palpitations, headaches - Dx: 24 urine for catecholamines and metanephrines, CT abdomen 7. Hypo or hyperthyroidism - Dx: TSH 8. Hyperparathyroidism - Dx: Serum Calcium and PTH 9. COA - BP high in arms and low in legs; weak dorsalis pedis pulses - Dx: Echo 10. OSA - Dx: sleep study

A 60 year old woman develops the sudden onset of "floaters" and flashes of light in her left eye. She then experiences a loss of vision in that eye that she describes as a "curtain coming across" her field of vision. What is the most likely diagnosis? What is the most appropriate next step in the management?

1. Retinal detachment - Painless vision loss, floaters, flashes of light 2. Immediate referral to opthomalogist - medical emergency

Cardiac enzymes Troponin I CK-MB

1. Rises in 2-3 hours, Peaks at 12 hours, Normalize in 1-2 weeks 2. Rises in 2-12 hours, Peaks at 10-24 hours, Nomralizes in 2-3 days

A 53 year old woman comes to the clinic for follow up of chronic hypothyroidism. She is asymptomatic on her current dose of levothyroxine, but recent lab studies show a slightly suppressed TSH. 1. What are the risks of giving supratherapeutic doses of levothyroxine? 2. What is the most appropriate mngt for this patient?

1. Risks: accelerated bone loss, A fib 2. Reduce levothyroxine. Re-eval in 6 weeks

Name likely CNS pathogen associated with each of the statements 1. CSF leak 2. Temporal lobe lesion 3. College student with rash and stiff neck 4. Unvaccinated child 5. Most common cause of bacterial meningitis

1. S. pneumnoiae 2. HSV 3. N. meningitidis 4. HiB 5. S. pneumoniae

What red flags on history or physical exam should prompt further evaluation of a headache? 1. Sudden-onset HA 2. New onset HA after age 50 3. New onset HA in a cancer patient 4. New onset HA in a patient with HIV risk factors 5. HA that is more frequent or more severe 6. HA following head trauma 7. Signs of systemic illness 8. Neurologic signs 9. Papilledema

1. SAH 2. GCA or mass lesion 3. Brain mets 4. Meningitis/brain abscess 5. Brain tumor 6. Subdural/epidural hematoma 7. Meningitis/encephalitis 8. Mass lesion, hemorrhagic stroke, encephalitis 9. Brain tumor, Pseudotumor cerebri

A 54 year old man is brought to the ED for loss of consciousness. He was in a meeting at work when he suddenly collapsed. He regained consciousness after about 10 minutes and began complaining of severe right sided headache. He has no prior history of headaches. 1. What is the most likely dx? 2. What is the most appropriate initial dx test? 3. What treatment is necessary to prevent death?

1. SAH due to ruptured aneurysm 2. CT contrast head w/o contrast 3. Surgical clipping/embolization of the vessel

A 12 year old obese boy comes to the physician because of a 4 week history of left hip pain that causes him to limp. There is no hx of trauma to the hip or leg. On physical examination, the patient had a waddlign gait and prefers to hold his hip flexed and externally rotated. He had restricted internal rotation of the hip What is the most likely dx in this patient? What is the most appropriate treatment in this patient?

1. SCFE 2. Tx to stabilize the joint

A 56 year old woman develops a painful rash 2 weeks after starting sulfasalazine for ulcerative colitis. She says that the rash was preceded by fever, malaise, and arthralgias. Her temperature is 39C, and pulse is 110/min. Physical exam shows conjunctivitis, oral ulcers, and erythematous target-like lesions on her face, trunk, and extremities. Over the next few days, bullous lesions develop and the patient's skin starts to slough off. 1. What is the most likely dx? 2. What is the appropriate mngt for this patient?

1. SJS/TEN Meds: allopurinol, carbamazepine, lamotrigine, phenobarbital, phenytoin, sulfamethaxole, sulfasalazine, mycoplasma pneumoniae, CMV 2. Admit to ICU/burn unit Stop drugs Wound Care IVF/electrolyte/pain control Monitor for Bacterial Superinfection

A 43 year old woman presents with a 2 day history of left sided chest pain. The pain is sharp and is worse during deep inspiration or coughing. It is owrse when lying supine and is somewhat relieved by sitting up and leaning forward. She reports minimal shortness of breath. She c/o malaise, intermittent fever, and joint pains in her hands and knees. Physical examination shows a rash on the patient's face, arms, and upper chest consistent with photosensitivity. Breath sounds are diminished in the left posterior lung field. There is mild tenderness of the joints in the hands. Lab studies show Serum creatinine: 1.9 Hbg: 10.6 Leukocyte count: 3600/mm^3 An x-ray of the chest shows diffuse, hazy pulmonary infiltrates and a left-sided pleural effusion. 1. What is the most likely dx? 2. What is the best initial dx test for this condition? 3. What serologic tests are most specific for this condition?

1. SLE 2. ANA 3. Anti-dsDNA Anti-smith

What disorder is most strongly associated with each of the following autoantibodies? 1. Anti-dsDNA 2. Anti-histone 3. Anti-La/SSB 4. Anti-Smith 5. Anti-topoisomerase 1 6. Anticentromere

1. SLE 2. Drug-induced lupus 3. Sjogren 4. SLE 5. Diffuse cutaneous systemic sclerosis 6. Limited cutaneous systemic sclerosis

which antidepressant best matches each of the following descriptions 1. generally first-line treatment for unipolar major depression 2. good choice for depressed patient with insomnia 3. good choice for patient who discontinued an SSRI due to sexual dysfunction 4. good choice for depressed patient with appetite suppression and weight loss 5. may help with smoking cessation 6. overdose causes sedation and life-threatening arrhythmias

1. SSRIs 2. trazodone, mirtazapine 3. bupropion 4. mirtazapine 5. bupropion 6. TCAs

which personality disorder fit each of the following descriptions? 1. lifelong voluntary social withdrawal, lax close friends, limited range of emotional expression, indifferent to the praise/ criticism of others 2. socially inhibited, shot, feelings of inadequacy, hypersensitive to rejection or criticism 3. social ox redness and difficulty with personal relationships, plus eccentric appearance, behavior or thinking 4. distrustful, suspicious, litigious 5. unstable personal relationships, unstable mood and behavior, unstable self-image, and pulse of nurse, suicide threats or gestures, sense of emptiness and loneliness

1. Schizoid 2. Avoidant 3. Schizotypal 4. Paranoid 5. Borderline

STIs HIV Chlamydia and gonorrhea Syphilis

1. Screen all patients age 15-65 Screen older/younger patients at high risk (MSM, IV drug users, multiple unprotected sex partners, exchange sex for drugs or money 2. screen high risk women no recommendation for high-risk men 3. Screen MSM who engage in high-risk sexual behavior, commercial sex workers, and people in correctional facilities

A 35 year old woman with a history of IVDU comes to the physician because of a 3 day history of left knee pain and fever. She is sexually active with multiple partners. Her temperature is 39.2, pulse is 100/min, and bp is 140/90. She is alert to person, place and time. The left knee is tender and warm to the touch. Lab studies show leukocytosis and elevated ESR. 1. What is the most likely dx in this patient? 2. What is the most appropriate next step in mngt?

1. Septic arthritis 2. Arthrocentesis with joint fluid analysis - WBC - Gram stain and culture - Histology

What type of shock would most likely result in each of the following sets? 1. MAP low, HR high, CO high, PCWP low, SVR low 2. MAP low, HR high, CO low, PCWP low, SVR high 3. MAP low, HR low, CO low, PCWP low/nl, SVR low 4. MAP low, HR variable, CO low, PCWP high, SVR high

1. Septic shock 2. Hypovolemic shock 3. Neurogenic shock 4. Cardiogenic shock

A 62 year old man comes to the physician because of a 1 day history of a painful rash on his right side. He says that he first felt a "burning" sensation in the area, and then the rash developed a day later. Examination of the skin shows multiple groups of erythematous vesicles over the right flank. 1. What is the most likely diagnosis? 2. What is the most appropriate mngt?

1. Shingles (herpes zoster) - dermatomal, painful, postherptic neuralgia 2. Acyclovir, Valacyclovir, Famciclovir Start within 72 hrs 7 day course Analgesic+Shingles Vaccine

Management of warfarin overdose INR <5 (no bleeding) 5-9 (no bleeding) >9 (no bleeding) Any serious or life threatening bleed

1. Skip one dose of warfarin, consider reducing warfarin dose 2. Skip 1-2 doses of warfarin, consider low-dose vitamin K, reduce warfarin dose 3. Hold warfarin until INR is therapeutic, Vitamin K 4. Hold warfarin until INR is therapeutic, Vitamin K, PCC/FFP

A 58 year old woman comes to the clinic with a 2 week history of low back pain, urinary incontinence, and difficulty walking. She is a former smoker and was diagnosed with lung cancer one month ago. Physical examination reveals bilateral weakness in the lower extremities, decreased sensation in dermatomes L2 and lower, and 1+ deep tendon reflexes in the lower extremities. X-rays show no degenerative changes of the spine. 1. What is the most likely cause of the patient's sx? 2. What is the first step in treatment?

1. Spinal cord compression 2. Steroids

What is standard error of the mean? What is the 95% confidence internval?

1. Standard deviation/(square foot of sample) = standard error of the mean 2. Mean score +/- 2*(SE of mean)

A 30 year old woman comes to the clinic with neck pain. Physical examination of the neck reveals a tender thyroid gland that is diffusely enlarged. There are no thyroid nodules. The patient is started on naproxen for the pain, and thyroid blood tests are ordered. She returns to the clinic 2 days later for scheduled follow up. Her thyroid pain has improved, but now she is experiencing palpitations and fine tremor int he hands. Results from the initial lab studies show a decreased serum TSH and an increased free T4. A radioactive iodine uptake scan shows decreased uptake. 1. What is the most likely diagnosis? 2. What would be the most appropriate treatment at this time? 3. What would you expect the serum TSH to be at follow up in 6 months?

1. Subacute thyroiditis - hyperthyroid stage 2. Propanolol, atenolol, no methotrexate, no ptu 3. Increase TSH or Normal

Impetigo depth of infection causative organisms exam findings treatments

1. Superficial 2. S. aureus, S. pyogenes 3. Papules => vesicles => pustules => honey-colored crusts 4. topical mupirocin or retaplamuln oral dicloxacillin or cephalexin

A 38 year old executive comes to the clinic for 10 days of nasal congestion, nasal discharge, sinus pressure, headache, cough, and intermittent fever. 1. What are the indications for antibiotic therapy in a patient with acute rhinosinusitis? 2. What antibiotics are recommended for empiric therapy of acute bacterial rhinosinusitis?

1. Sx > 7days, plus purulent nasal discharge, maxillary tooth or facial pain, especially unilateral. Unilateral maxillary sinus tenderness 2. Augmentin - empirical treatment Doxycycline, Moxi/Levo 5-7 days Nasal steroid spray

A 50 year old man comes to the clinic with loss of sensation of pain and temperature in a cape-like distribution over the neck, shoulders, and down both arms. An MRI of the spinal cord shows a dilated cavitation in the cervical spinal cord 1. What is the most likely diagnosis? 2. What intervention would be indicated?

1. Syringomyelia 2. Surgical shunt

For each of the following causes of Cushing syndrome, what is the most appropriate tx? 1. Exogenous glucorticoid administration 2. Corticotroph pituitary adenoma 3. Hyperfunctioning adrenal adenoma 4. Ectopic ACTH production from a non-pituitary site

1. Taper the glucocorticoids 2. Surgery, Transphenoidal resection +/- irradation of the pituitary 3. Unilateral adrenalectomy 4. Surgical resection Tx tumor with chemo Ketoconazole Mitotane Adrenalectomy

Bullous Pemphigoid Bullae appearance Presence of oral lesions Treatment

1. Tense; hand; difficult to rupture 2. 10-30% 3. Topical corticosteroid: clobetasol +/- immune modulator - azathioprine, mycophenolate

What is the most likley cause of headache based on each of the following description? 1. Most common cause of headache 2. Perioribital pain with lacrimation and/or rhinorrhea 3. Obese woman with papilledema 4. Scintillating scotomas prior to headache 5. Jaw muscle pain when chewing 6. Responsive to 100% oxygen supplementation

1. Tension HA 2. Cluster HA 3. Idiopathic ICH 4. Migraine HA 5. GCA 6. Cluster HA

Which type of spontaneous abortion best matches each of the following statements? 1. Bleeding before 20 weeks + no passage of POC + closed cervix 2. Bleeding befoer 20 weeks + cramping + no passage of POC + open cervix 3. Passage of some POC + open cervix 4. Passage of all POC + closed cervix 5. Fetal death before 20 weeks gestation + no passage of POC + closed cervix 6. Spontaneous abortion complicated by uterine infection

1. Threatened 2. Inevitable 3. Incomplete 4. Complete 5. Missed 6. Septic

Which type of spontaneous abortion best matches each of the following statements? 1. Bleeding before 20 weeks gestation + no passage of POC 2. Bleeding before 20 weeks gestation + no passage of POC + open cervix 3. Passage of some POC + open cervix 4. Passage of all POC + closed cervix 5. Fetal death before 20 wks gestation + no passage of POC + closed cervix 6. Spontaneous abortion complicated by uterine infection

1. Threatened abortion 2. Inevitable abortion 3. Incomplete abortion 4. complete 5. Missed 6. septic

A 19 year old woman is brought to the ED by her bf becase of a 1-day history of fever, rash, and altered mental status. The bf says that over the past day the patient has developed a rash that looks like sunburn, but he denies that they've spent any time outdoors. He thinks her last menstrual period started 3 days ago. She appears lethargic. Her temperature is 39C, pulse is 120/min, and bp is 85/50. Physical exam shows an erythematous macular rash involving her whole body, including the palms, soles, and mucous membranes. What is the most likely diagnosis? What is the most appropriate management for this patient?

1. Toxic Shock Syndrome 2. Remove source of infection Supportive care: IVF and Vasopressors Abx: Clindamycin + Vanc

What abx should be used to treat UTI? 1. Women with uncomplicated lower UTI? 2. Women with upper UTI? 3. Elderly men with UTI?

1. Treat for 3-5 days with bactrim, macrobid, cipro(2nd) 2. Treat for 7-14 days with O/P: Oral cipro, TMP/SMX Inpatient: IV fluro, extended-spectrum ceph (rocephin, ceftazidime), extended-spectrum PCN (pip/tazo) 3. Treat for 7-14 days often associated with prostate pathology and urinary obstruction

A 1 day old newborn has a physical examination that is significant for a "clunk" of the right femoral head when gentle posterior pressure is applied to the femur while adducting the flexed hip. 1. What is the most appropriate next step in the dx of this condition? 2. What is the treatment of this condition?

1. U/S of bilateral hips 2. Abduction splint

Indicate whether each of the following conditions is most consistent with an upper or lower motor neuron lesion 1. Stroke 2. Bell's Palsy 3. Spasiticity 4. Transient ischemic attack 5. GBS 6. Cauda Equina Syndrome

1. UMN 2. LMN 3. UMN 4. UMN 5. LMN 6. LMN

What bone or bones are of concern in each of the following fractures 1. Monteggia fracture 2. Galeazzi fracture 3. Boxer's fracture 4. Jones fracture 5. Greenstick fracture

1. Ulna 2. Radius 3. 5th MCP 4. 5th metartarsal 5. Long bones

Bacterial conjunctivitis - tx Viral conjunctivitis - tx Allergic conjunctivitis - tv

1. Unilateral, thick purulent discharge tx: Topical erythromycin, topical trimethoprim-polymyxin B 2. Unilateral -> Bilateral, watery/mucous discharge, gritty sensation tx: Self-limited 3. B/L, watery discharge, itching Tx: Topical antihistamines, artificial tears

A 43 year old man comes to the clinic because of black stools. He denies abdominal pain. Rectal examination reveals black stool that is strongly guaiac positive. 1. Is this most likely an upper GI or lower GI bleed? 2. What anatomical structure marks the boundary between upper and lower GI bleed? 3. What is the most important aspect in mngt of an acute, large volume GI bleed?

1. Upper GI bleed 2. Ligament of Treitz 3. IVF resuscitation

Erysipelas depth of infection causative organisms exam findings treatments

1. Upper dermis 2. S. pyogenes 3. Painful red raised lesion w/ clear demarcations 4. Oral PCN or amoxicillin IV ceftriaxone or cefazolin

Answer the following questions regarding osteomyelitis 1. What is the empiric abx tx approach for an adult patient with OM? 2. What other tx options are important in OM besides abx?

1. Vancomycin FQ: Cipro or Levo 3rd gen ceph: ceftazidine 4th gen ceph: cefepime 2. Surgical debridement Hyperbaric oxygen Negative pressure wound therapy

A 32 year old woman comes to the clinic with two days of sore throat. she has no fever, nasal discharge, or cough. Her two preschool-aged children were recently treated for streptococcal pharyngitis. 1. What are common causes of acute pharyngitis? 2. What clinical features can be used to determine which patients with pharyngitis should be tested for GAS? 3. What antibiotics may be used to treat GAS pharyngitis?

1. Viral infection (influenza), noninfectious, Group A/C/G strep, M. pneumonia, C. pneumonia, C. diptheriae 2. Centor Criteria: Tonsillar exudates, fever, anterior cervical adenopathy, absence of cough 3. Oral amoxicillin or PCN V for 10 day. 1 dose IM pen G Cephalexin, Macrolide for PCN allergic Clindamycin if resistant

Which type of vasculitis fits each of the following? 1. Vasculitis of the kidney, upper airways, and lungs 2. Vasculitis of the kidney and GI tract, but spares the lungs 3. Palpable purpura on the legs, associated with IgA nephropathy 4. Vasculitis in a young asthmatic 5. Vasculitis in a young male smoker 6. 2 year old asian girl with strawberry tongue and desquamtion of the hands/feet 7. 20 year old asian woman with weak pulses in the upper extremities 8. elderly woman with unilateral headache and jaw claudication 9. associated with hep B 10. assoicated with perforation of the nasal septum 11. Associated with PMR

1. Wegener's granulomatosis with polyangiitis 2.Polyarteritis Nodosa 3. HSP 4. Churg-Strauss syndrome (eosinophilic granulomatosis with polyangiitis) 5. Thromboangiitis obliterans 6. Kawasaki 7. Takayasu arteritis 8. GCA 9. Polyarteritis nodosa (hep C too) - spares lungs 10. Wegener's 11. GCA

A 65 year old woman comes to the clinic with headache, confusion, weakness, myalgias, fever, and malaise for three days. He has been working at a children's summer camp for the last four weeks, where mosquitoes have been prevalent. Physical examination reveals a maculopapular rash. CSF shows elevated protein, normal glucose, and a moderate number of lymphocytes 1. What is the most likely dx? 2. What tx is indicated?

1. West Nile Encephalitis 2. Supportive Care

A 28 year old woman is referred for evaluation of mildy elevated AST and ALT, which were discovered on an insurance physical. she does not drink alcohol. Total bilirubin and alkaline phosphatase were normal. Hepatitis B and C serologies are ngative. Serum ferritin is 62 (normal 12-150). Serum ceruloplasmin is 12 (normal 15-60) 1. What other tests could help confirm the diagnosis 2. How are patients with this disease treated?

1. Wilson's disease: Low serum copper, high 24 hr urine copper, slit-lamp exam kaser-fleischer rings, liver biopsy, genetic testing 2. Cu chelation therapy with penicillamine and/or Trienterene

1. HbsAg (-), HbsAb (-), HbcAb (+), HbeAg (-) 2. HBsAg (+), HbsAb (-), HbcAb (+), HbeAg (+) 3. HBsAg (-), HbsAb (+), HbcAb (+), HbeAg (-) 4. HBsAg (-), HBsAb (+), HBcAb (-), HBeAg (-)

1. Window period 2. Acute Dz 3. Recovered 4. Vaccinated

COPD Exacerbation Presentation Evaluation Management

1. Worsening cough, worsening dyspnea, increased sputum production, may be associated with a respiratory infection 2. Pulse ox, ABG, CXR, CBC, electrolytes, glucose, +/- BNP, blood and sputum cx not helpful 3. O/P: albuterol/ipratropium nebs, oral steroids, +/- oral abx Inpatient: - Supp O2, - Nebulized albuterol plus ipratroprium, - IV methylprednisolone to improve lung function and reduce length of hospitalization, - Abx (ceftriazone plus azithromycin, or levo, or pip/tazo) - +/- ventilatory support (NPPV/intubation)

A 32 year old woman presents with tingling and weakness in the first three digits of her right hand. She experiences these symptoms only on workdays. She works in an office and spends at least seven hours on the computer. 1. What initial management could be suggested to the patient? 2. If sx persist, what procedure may offer relief before surgical intervention?

1. Wrist brace/splint 2. Steroid injection

Answer the following questions regarding GERD 1. How should mild GERD be treated? 2. If initial treatment of GERD is ineffective, what is the next tx options?

1. Wt loss, elevated head of bed, quit smoking, limit alcohol, caffeine, and chocolate. 2. H2 blockers/PPI

A 45 year old man comes to the clinic because of heartburn. He was treated for a peptic ulcer in the proximal duodenum about three months ago, and now he has recurrent epigastric abdominal pain, weight loss, and diarrhea. EGD preveals prominent gastric folds. There is a new peptic ulcer in the distal duodenum. The serum gastrin concentration is 560 (normal < 110) What is the most likely dx? With what endocrine tumors might this condition be associated How should this patient be managed?

1. ZES: gastrinemia (duodenum pancreateic islet cells) recurrent peptic ucer disease, diarrhea 2. MEN1 - hyperparathyroid, pituitary adenoma, pancreatic islet Dx: serum gastrin; CT/MRI of abd 3. PPI - omeprazole Octreotide - somatostatin Surgical resection

car seat safety

1. age less than 2 years: rear facing car seen, preferably in the middle of the back seat 2. age 2-5 years: bigger forward facing car seat with a chest harness 3. age over 5 years: booster seat with the seat-belt in backseat, until height is over 4 ft 9

ALL Peak incidence? Associated with? Presentation?

1. ages 2 to 5 2. associated with trisomy 21 3. Bone pain, lymphadenopathy, recurrent infections, anemia, thrombocytopenia, abnormal bleeding

a 17-year-old girl is being evaluated for amenorrhea. During the interview you note that she is excessively thin and quit upon questioning about her weight she says that she exercises 2-3 times daily and eats only salads because she is worried about gaining weight and wants to lose a few more lb before her senior prom. She denies binge eating what is the diagnosis or treatment options are available

1. anorexia nervosa 2. interdisciplinary team approach with cognitive behavioral therapy, inpatient therapy may be needed, nutritional therapy, calcium and vitamin-D, and treat comorbid depression

a 68-year-old man comes to physician because of episodes of left arm pain and fatigue better committee by dizziness, blurred vision, and unsteady gait. He recently bought a new house and has been painting the anterior walls. He finds that the symptoms occur when he tries to pain using his left arm. What feature distinguishes subclavian steal syndrome from thoracic outlet syndrome? What can be used to aid in the diagnosis? what is the treatment for this patient's condition?

1. arm claudication and neuro symptoms in subclavian steal syndrome, thoracic outlet syndrome has no neural symptoms 2. Doppler ultrasound, MR or CT angiogram 3. bypass surgery

What are the clinical manifestations of each of the following stages of syphilis? Primary Secondary Tertiary

1. chancre 2. Palms and soles rash, condylomata lata, systemic sx 3. Syphilitic aortitis, gumma, Tabes dorsalis

A 57 year old man comes to the clinic for a routine health evaluation. He has not seen a physician in some 20 years. He admits to moderate to heavy alcohol use but denies tobacco use. Physical examination shows jaundice, spider angiomata, gynecomastia, and abdominal distension with shifting dullness and a fluid wave. 1. What condition is most likely responsible for the physical exam findings? 2. What lab tests on the ascitic fluid would help confirm this diagnosis?

1. cirrhosis + portal htn (HBV, HCV, hemochromatosis, cholestasis) 2. Serum-ascitic albumin gradient SAAG > 1.1 - liver disease, CHF, portal thrombosis SAAG < 1.1 - pancreatitis, nephrotic syndrome

a 32-year-old man is brought to the emergency department with cough and fever. He has a history of treatment resistant schizophrenia. An x-ray of the chest reveals a left lower lobe consolidation consistent with pneumonia. CBC shows leukocyte count of 900, which is felt to be due to his antipsychotic medication. Which antipsychotic medication is he most likely taking what testing/monitoring should be performed when starting this medication

1. clozapine 2. CBC, before starting and then weekly for 6 months. Baseline weight, BMI, waist circumference. Baseline fasting blood glucose with hemoglobin A1c, and baseline EKG. Weekly sed rate and troponins for 1st 4 weeks

were the signs and symptoms of heroin/opioid overdose what is the reversal agent and cases overdose

1. depressed consciousness, miosis, respiratory depression 2. Naloxone

a 7-month-old girl has progressively worsening lethargy over several days. She is feeding less due to a weak suck and has decreased bowel movements. Her mother says she has been feeding her more table foods lately. She has ptosis of both eyelids and overall decreased muscle tone. 1. What food may have caused this condition 2. what is the most appropriate management for this patient

1. honey or home canned foods because of botulism 2. close monitoring for respiratory failure, botulinum immunoglobulin

67-year-old former smoker who undergoes a screening abdominal ultrasound which reviews a 5.2 cm abdominal aortic aneurysm located inferior to the renal arteries. How Should this finding be managed? what is the most important modifiable risk factor for the worsening of an existing AAA?

1. less than 5.5 cm and asymptomatic equals observation more than 5.5 cm or symptomatic equal surgery 2. smoking

a 76-year-old woman is brought to the emergency department because of a 1 hour history of nausea, vomiting, and severe periumbilical abdominal pain. She still rates the pain as 10/10 point scale. She has a 20 year history of hyperlipidemia, and she had a myocardial infarction 2 years ago. Physical examination shows a slightly distended abdomen, normal bowel sounds, and mild tenderness to palpation. There is no rebound tenderness or guarding What is the most likely diagnosis? what is the most appropriate next step

1. mesenteric ischemia 2. CT angiogram of abdomen

which personality disorder fits each of the following descriptions 1. controlling, perfectionist ache, order leak, stubborn, possibly hoarding 2. excessive need to be taken care of, submassive and clingy, low self confidence, fears of separation and losing support 3. from analogy, unable to conform to social norms, disregard for others rights, reckless disregard for safety 4. grandiosity, sense of entitlement, lack of empathy 5. excessive dramatic, emotional, and extra Verde, attention seeking, sexually provocative behavior, unable to maintain intimate relationships

1. obsessive-compulsive personality disorder 2. dependent personality disorder 3. antisocial personality 4. Narcisitic personality disorder 5. histrionic personality disorder

What is the most likely cause of vision loss given each of the following statements? 1. Young women with painful loss in one eye; relative afferent pupillary defect on exam. 2. Elderly woman with headache, jaw claudication, and elevated ESR 3. Acute, painless vision loss in one eye; cherry-red spot in macula on fundoscopic exam 4. Vision loss preceded by floaters and flashes of light 5. Progressive vision loss starting in the periphery; cupping of optic disc on fundoscopic exam 6. Sudden onset of painful red eye with headache, N/V, and blurred vision 7. Gradual vision loss due to opacification of the lens?

1. optic neuritis 2. temporal arteritis 3. Central retina artery occlusion 4. Retinal detachment 5. Open angle glaucoma 6. Closed angle glaucoma 7. Cataracts

What is the treatment for each of the following condition? 1. Tinea unguium 2. Tinea pedis 3. Tinea corporis 4. Tinea capitis

1. oral antifungal (terbeinafine, itraconazole, griseofulvin) - 6 to 12 weeks 2. topical antifungal (terbinafine, naftifime, clotrimazole) - 1-4 weeks 3. topical antifungal 4. oral antifungal - 12 weeks

A 22 year old woman comes to the clinic because of intermittently bloody diarrhea that began one month ago. She also complains of colicky abdominal pain and fatigue. Colonoscopy shows continuous circumfernetial inflammation of rectum and sigmoid colon 1. What serology would suggest a diagnosis of UC? 2. What is the most appropriate initial therapy for mild cases of UC?

1. p-ANCA + 2. 5-ASA, Steroids, Immunosuppresants (cyclosporine, azathioprine, 6-MP), Anti-TNF-alpha infliximab

a 36-year-old woman comes to the physician because of a 1 month history of sharp heel pain. She is a runner and has recently started running long distances in preparation for marathon. She says that the pain is worse when she 1st starts walking and then gradually improves as she continues to walk. Physical examination of the foot shows decreased dorsiflexion and point tenderness near the medial process of the calcaneal tuberosity. What is the most likely diagnosis? what is the appropriate management of this patient's symptoms?

1. plantar fasciitis 2. stretching exercises, avoiding flat shoes, avoiding walking barefoot, shoe insert, NSAIDs, if patient has 6 months to 1 year of no improvement, may do glucocorticoid injection and surgery

What is the best choice for management of each to following disorders? clavicular fracture Colles fracture scaphoid fracture femoral neck fracture intertrochanteric fracture femoral shaft fracture trigger finger and De Quervain's tenosynovitis

1. sling or figure 8 bandage 2. closed reduction and casting 3. thumb spica 4. femoral head replacement 5. open reduction and pinning 6. intramedullary rod fixation 7. steroid injection

Developmental milestones 2 months 4 months 6 months 9 months 12 months 18 months 2 years 3 years 4 years 6 years

1. social smile and eye follow to midline 2. babies are aware of the caregiver and eyes now follow past midline 3. rollover, sits upright, grasps/rakes, babbles 4. crawling, grasping with thumb, maybe one word 5. 2-5 words, pulls to stand (cruising), walks with help, pincer grasp, 2-block tower, stranger anxiety 6. runs/walks backwards, use spoon + cup, 2 word sentences, copies parent, 4 block tower 7. walks up and down stairs with help, jumps, 6 block tower, 2 or 3 word sentences, 50-75 words, follows 2 step commands 8. ride tricycle, copies a circle 9. plays with others, draw a + and triangle, 250 words, 4 word sentence 10. skipping, draws a stick figure

What substance overdose could be treated for the following substances? Aminocaproic acid? Atropine? Deferoxamine? Dimercaprol? Flumazenil? Fomepizole? Hyperbaric oxygen? N-acetylcysteine? Penicillamine? Physostigmine? Pralidoxime? Protamine Sulfate?

1. tPA 2. Organophosphates or cholinergic agonist (bethanechol) 3. Iron 4. Mercury, Lead, Gold, Arsenic 5. Benzos 6. Ethylene glycol and Methanol 7. Carbon Monoxide 8. Acetaminophen 9. Copper 10. Anticholinergics like atropine 11. Organophosphates or cholinergic agonist 12. Heparin

24-year-old man with schizophrenia is brought to clinic by his mother because of 2 weeks of uncontrollable movements of his lips and mouth. Neither the patient nor his mother can't remember the name of the antipsychotic medication, although described as a round white pill. Physical examination shows persistent lip-smacking and movements of the mouth and jaw what is the name of this movement disorder what class of antipsychotic is most likely to cause this side effect what other movement disorders can be seen in patients taking this class of drug

1. tardive dyskinesia 2. high potency traditional antipsychotics 3. traditional 1st generation low potency like chlorpromazine and thiaridozine can have anticholinergic effects but high potency like haloperidol can have tardive dyskinesia. First several days he may have acute dystonia and torticollis, 1st month can have parkinsonism, 1st 2 months can have echo the showed, months to years and do developed tardive dyskinesia which is irreversible. The atypical 2nd generation causes sedation, weight gain, diabetes

a 16-year-old boy is brought to the emergency department after the sudden onset of severe groin pain with radiation to lower abdomen. On exam, the affected testicle is oriented transversely in the scrotum. The cremasteric reflex is absent. What is the most likely diagnosis what study can help confirm the diagnosis what is the management

1. testicular torsion 2. Doppler ultrasound scrotum 3. surgery

a 21-year-old man with scrotal pain is seen in the emergency department. He complains of fever and chills for the last 2 days. He is sexually active and admits to a new sexual partner. Cremasteric reflex is present. The pain is relieved when the scrotal contents her manual elevated. Ultrasound reveals normal testicular blood flow will other study should be used to assist in diagnosis? What is the most appropriate treatment for this patient

1. urinalysis, urine culture, culture of discharge 2. antibiotics that cover gonorrhea chlamydia, cipro plus doxy

A 26 year old woman is brought to the emergency department because of prolonged bleeding from a minor laceration on her arm. She cut her arm while doing some yard work, and it has continued to ooze blood for two hours. She has always bruised easily and she has occasional nosebleeds. She admits to heavy menstrual bleeding. Lab studies show normal plt count, normal PT, and increased PTT. Bleeding time is elevated. Ristocetin cofactor activity is markedly reduced 1. What normal substance is most likley deficient in this patient? 2. What is the treatment for this disorder?

1. vWD 2. 1st - DDAVP so to stimulate vWF in endothelial cells - intranasals or IV 2nd: Concentrated vWF

Answer the following questions regarding malignant hyperthermia 1. What substances are known to cause malignant hyperthermia? 2. What is the treatment for malignant hyperthermia?

1. volatile anesthetics: holathane, desfulrane, enfluane, isoflurane, sevoflurane Succinylcholine 2. Stop offending agent, 100% O2, hyperventilation, dantrolene Core temperature >102.2F - cool patient (cold IV saline) - stop at 101.3 Acidosis: Bicarb Dysrthymais: ACLS Hyperkalemia: Calcium chloride, bicar, furosemide, insulin+glucose

What is the NNT?

1/ARR ARR = C/C+D - A/A+B

Number needed to treat

1/ARR (C/C+D - A/A+B)

A child is brought to the office for a routine well child visit. The child appears anxious during the examination. His mother says he is able to stand unassisted, walk with assistance, says mama and dada, and can stack two blocks. What is the most likely developmental age of this child?

12 months: rule of 2s

A 56 year old man with type II DM is admitted to ICU for sepsis. what is the target range for blood glucose?

140-180

What is the first line treatment for aspiration pna?

1st line - Clindamycin

Management of PCOS

1st line: Diet and Exercise 1st line meds: OCPs Add spironolactone for hirsutism 2nd line medication: Metformin Clomiphene if pregnancy desired

What medications are most useful for generalized anxiety disorder?

1st line: SSRI: Paroxetine, Setraline, Citalopram SNRI: Venlafaxine, Duloxetine 2nd line: TCAs Imipramine BZs: Clonazepam, Diazepam Buspirone

CLL Presentation?

25% are asymptomatic Painless LAD/splenomegaly Fatigue Fever Night sweats Weight loss Recurrent infections Autoimmune hemolytic anemia

What are the recommendations for infective endocarditis antibiotic ppx prior to dental procedures?

2g amoxicillin 30-60 minutes prior to procedure Endocarditis prophylaxis 1. Prosthetic cardiac valve 2. previous IE 3. Congenital heart disease - Unrepaired cyanotic congenital heart defect - repaired congenital heart defect with prosthetic material - cardiac transplantation with valvuloplasty

Transfusion reactions Delayed hemolytic reaction

3-20 days after transfusion (antibodies to minor RBC antigens) Usually mild hemolysis, fever Tx: None

Asthma Mild persistent

3-6 daytime episodes/week 3-4 night time episodes/month >80% FEV1 PRN albuterol + low dose ICS +/- cromolyn +/- montelukast

How is acute stress disorder different from PTSD?

3d < 1 month > 1 month

An 18 month old girl is diagnosed with her first febrile UTI. What is the management and follow up for this patient?

3rd ceph: cefixime, cefdinir, ceftibuten Renal and bladder U/S in patients IF - child <2 yo with first febrile UTI - child of any age with recurrent febrile UTI - child of UTI and family hx of GU dz, poor growth, or HTN - child who does not respond to abx

To diagnose a MDD, a patient must have at least ______ symptoms over a period of _______, and those symptoms include either___________ or ___________

5/2weeks/anhedonia/depressed mood

Lupus nephritis

50% of SLE will get nephritis Diagnosis: ANA, Anti-dsDANA, renal biopsy Tx: Corticosteroids, mycophenolate mofetil, cyclophosphamide, azathioprine

Lacunar infarcts pure motor hemiparesis

50% of lacunar infarcts where there is weakness of the face, arm, and leg on one side of the body. No sensory or cortical signs (aphasia, neglect, apraxia, hemianopsia)

What is the mngt of a patient with a 5 mm renal stone

50% of passing. Pain control with narcotics, NSAIDs Tamsulosin 8 mm usually will pass

At what age are febrile seizures most common?

6 months to 5 years

A full-term newborn has a strawberry hemangioma on his left posterior neck. What is the most appropriate management for his skin lesion?

60% resolve in 2 years 90% disappear in 9 years OBSERVATION If @ eye, oral propanolol

When will a patient experience most severe symptoms associated with acetaminophen toxicity?

72-96 hrs

in pediatric patients with a needle troponin dependent precocious puberty what is the primary goal with treatment

< 7 or 8 in girls or <9 boys early closure of above seal plates look at future predicted adult height and you can treat with GnRH analog therapy monthly injection

In a patient with diabetes, what are the goals for each of the following? Blood pressure LDL HbgA1c

<140/90 above this -start meds <100 <7.0%

Asthma Mild intermittent

<2 daytime episodes/week <2 night episodes/month >80% FEV1 PRN albuterol

What are some persistent causes of hematuria?

<20: GN disease 20-50: ADPKD >50 - neoplasms

A fib AC risk

> 48 hr or unknown: anticoag 1st 4 weeks and then cardiology after TEE to r/o thrombus

What are the defining features of preeclampsia?

>140/90 after 20 weeks Proteinuria > 300 mg/24 hrs If no proteinuria - Thrombocytopenia, Renal insufficiency, Impaired liver function, pulmonary edema, cerebral/eye problems

What precautions should be taken prior to cardioversion to prevent an embolic event in a patient with stable atrial fibrillation?

? or > 48 hrs => TEE - no thrombus => cardiovert and warfarin; dabigatrin for 1 month - positive thrombus => AC for 3 weeks => rpt TEE <48 hrs => Cardiovert +/- AC depending on risk

What is the classic presentation of HOCM?

A young athlete with syncope/sudden death during exercise. Syncope, dyspnea, or chest pain that is worse with exertion S4 Systolic murmur

PPV

A/A+B

Sensitivity

A/A+C

What is the management of suspected spontaneous abortion?

ABCs - if unstable or bleeding heavily -> IVFs, blood, D+C Hx and Px U/S Labs - cbc with diff - type and screen (If Rh negative, give Rh immune globulin) - Quantitative serum B-hCG Misoprostol can be given orally or vaginally to evacuate uterus D+C If septic abortion, give IV broad spectrum abx - Gentamicin + Clindamycin - Pip/tazo Expectant management

What is the treatment of a child with a febrile seizure?

ABCs Benzos short term: lorazepam Acetaminophen/Ibuprofen

Ischemic stroke treatment?

ABCs Fluid and glucose status If thrombolytics - 185/110 below If not likely - 220/120 below Alteplase if sx onset is <4.5 hrs Anti-platelet with ASA, dipyridamole, plavix within 48 hrs WARFARIN (if due to a fib) Heparin or LMWH for patients with progressive thromboembolism Statins (start within 3 days)

Outline the management of a patient who presents with third trimester bleeding

ABCs IVF monitor vitals, urine output, fetal HR CBC, Type and screen, DIC labs Blood transfusion US before digital exam Corticosteroids RhoGAM if delivery

Management of third trimester bleeding

ABCs IVF, monitor urine output and vitals External fetal monitoring Physical exam Labs - CBC with diff, type and screen, DIC labs (fibrinogen, D-dimer, PT, PTT) Blood transfusion U/S Consult OBGYN If <34 weeks gestation: betamethasone (IM, 2 doses) If rh negative: RhoGAM

What is the management of eclampsia?

ABCs Maintain airway and prevent aspiration Roll the patient onto left side Prevent trauma Supplemental oxygen Anticonvulsant - IV mag sulfate (diazepam, phenytoin - 2nd line) BP control - IV hydralazine, IV labetalol, oral nifedipine Delivery (definitive treatment) DO NOT PERFORM C/S IN ACTIVE SEIZURE

Treatment of MI Initial management?

ABCs Prelim H+P 12 lead EKG Cardiac monitor O2 PRN (if SpO2 < 90%) IV Access and draw labs ASA Nitrates (unless contraindicated)

What is the initial management of an acute MI?

ABCs Prelim H+P 12-lead EKG Cardiac monitor O2 as needed IV access and draw labs ASA, Nitrate, Choose reperfusion strategy, Beta-blocker, High-intensity statin, Clopidogrel, UPH/Lovenox

How is TCA overdose managed?

ABCs: Airway, breathing, circulation Activated charcoal: 1g/kg up to 50 kg unless ileus is present or patient cannot protect airway Continuous cardiac monitor for at least 6 hours => if no problems, then clear for psychiatric evaluation. Frequent neuro checks Labs/studies: TCA, chem 7, EKG If QRS > 100 msec, use sodium bicarbonate If hypotensive => IVF (NS or LR) => if ineffective use Norepinephrine If agitation or seizures => Use Benzos, but not phenytoin because it is ineffective against toxin-induced seizures

What is the one test that can be used to evaluate for hypercapnia?

ABG - For CO2 retention

What medications can cause hyperkalmeia?

ACE-i ARBs K-sparing Diuretics Beta blockers Digoxin NSAIDs Azole Antifungals Trimethoprim

A 52 year old man complains of exertional dyspnea and is diagnosed with aortic regurgitation. He asks to delay valve replacement surgery for 3 weeks until after his daughter's wedding. what classes of medication could help relieve his sx?

ACE-i or ARB

A 57 year old man with diabetes has a routine checkup. He has no complaints. Physical examination is normal. His BP is 129/76 mmHg. Lab studies are shown: Hbg 12.3 Hbga1c 5.9 Creatinine 1.0 LDL 88 24-hour urine albumin 220/24 hours What class of medication would be most appropriate to begin at this time?

ACE-i/ARB

A patient with type 2 DM is found to have proteinuria. What drug class will help slow the progression of diabetic nephropathy?

ACE-i/ARB

what medications are indicated to reduce mortality in patients with CHF?

ACE-inhibitors (ARBs) Beta blockers - bisoprolol, carvedilol, ER metorpolol Aldosterone antagonists - spironolactone, eplernone

Opportunistic infections HIV Crytococcal meningitis

AMS, fever, headache, stiff neck Elevated opening pressure (LP) Dx: India ink stain, cryptococcal antigen Tx: IV Ampotericin + Flucytosine for 2 weeks, oral fluconazole for 8 weeks

What are the characteristic features of serotonin syndrome?

AMS: anxiety, agitation, delirium, restlessness, disorientation Autonomic excitation: diaphoresis, tachycardia, hyperthermia, hypertension, vomiting, diarrhea Neuromuscular hyperactivity: tremor, muscle rigidity, myoclonus, hyperreflexia - Ocular clonus: slow continuous horizontal eye movement - Spontaneous or inducible clonus - Positive Babinski sign bilaterally

Normocytosis (MCV 80-100)

AOCD CKD Acute blood loss Hemolysis

A 4 year old girl comes to physician for a 2 day history of right ear pain. Her temperature is 102.3F. The patient has not taken any antibiotics in the last year. Physical examination shows an opaque, erythematous tympanic membrane with mild bulging. What is the appropriate treatment?

AOM >2 years: observation if appropriate tx High dose Amoxicillin 90mg/kg/d 1st line

A 34 year old man is in the hospital for sepsis that began from severe cellulitis in the lower extremities. Over the course of several days, his status starts to worsen, and he develops severe sob, and decreased responsiveness. He becomes hypoxemic, and a cxr shows bilateral pulmonary edema. His BNP is normal. He has a normal echo. The patient has to be intubated and placed on mechanical ventilation. What other treatments are most appropriate at this time?

ARDS - Admit to ICU - Tx underlying dz - Mechanical ventilation w low tidal volumes and adequate PEEP - Conservative mngt to fluid mngt to decrease pulmonary edema. Goal CVP 4-6 mmHg (furosemide and albumin) - Prone position improves oxygenation but does not improve - Minimize oxygen consumption by preventing fever, anxiety,a nd pain and limit resp muscle use. - Transfuse < 7 hbg

Long-term management of MI?

ASA Plavix (STEMI at least 12 months, NSTEMI at least 1 month) B-blocker ACE-i/ARB (if patient has HF or LVEF <40%) Aldosterone antagonist (spironolactone or eplerenone) if patient has LVEF < 40% or diabetes STATIN NITRATE Exercise No smoking Dietary modification

A 22 year old woman is brought to the ED with AMS. She is conscious but lethargic. Her friends report that she had been depressed and they are worried that she may have tried to commit suicide by overdose. The patient admits to nausea, vomiting, and diarrhea. She also complains of vertigo and tinnitus. On examination, she is breathing very quickly and deeply, and has a temperature of 38.8C. What is the most appropriate treatment for this patient?

ASA overdose - Tinnitus, Tachypnea, Hyperpyrexia, Mixed resp alkalosis + met acidosis with elevated anion gap Tx - Activated charcoal w/in 1 hr - Alkalinization of urine with IV NaBicarb - HD for severe

What is the most appropriate next step in mngt given each of the following abnormal Pap results? ASCUS

ASCUS => age = 25 and older => HPV => colposocopy if pos ASCUS => 21-24 yoa => rpt => colposcopy if positive

How can the ratio of AST to ALT help distinguish between alcoholic hepatitis and other causes of acute hepatitis?

AST double the ALT increase GGT in alcohol

What are the diagnostic sx of MDD?

AT least 5 of the following sx during a 2 week period - Depressed mood - Sleep disturbance - Interest (diminished) - anhedonia - Guilt or worthlessness - energy loss (fatigue) - Concentration (impaired) - Appetite or weight changes - Psychomotor agitation or retardation - Suicidal ideation (recurrent thoughts of death)

What are the complications of pancreatitis?

Abscess Pseudocyst - 4 weeks but 40% will resolve Fistulas Renal Failure Chronic pancreatitis Hemorrhage Shock DIC Sepsis Resp failure

A 76 year old man comes to the clinic for f/u of diabetes, hypertension, and a fib. He is accompanied by his caregiver, a family friend that spends six days a week with the patient. Review of the medical record shows that he has not been refilling his medication as often as he used to. He has had a 4.5 kg weight loss since last year. Physical examination shows bruising of his arms at different stages of healing. What would be the most appropriate course of action for this patient?

Abuse + Neglect Report suspected abuse + neglect Speak to patient w/o cargiver present

Wolff-Parkinson-White syndrome

Accessory conduction pathway -> preexcitation "delta" waves Predisposes to SVT, atrial fibrillation/flutter, and other arrhythmias Treat WPW-induced SVT with adenosine, verapamil, or procainamide Long-term therapy; RFA

Heparin LMWHs - Enoxaparin - Dalteparin

Activate antithrombin => enhanced inactivation of thrombin and factor Xa => reduce thrombus formation

What is the treatment for ASA overdose?

Activated charcoal IV Sodium bicarbonate to alkalinize the urine If it's bad enough, hemodialysis

Fondaparinux

Activates anithrombin => enhanced inactivtion of factor Xa => reduced thrombus formation

What is the typical presentation of ischemic colitis?

Acute abdomen, bloody diarrhea, vomiting, pain out of proportion to exam

Pancreatitis history and physical

Acute epigastric pain with radiation to the back with associated n/v. Grey turner sign with hemorrhage and eccymoses of the flanks. Cullen sign is periumbilical ecchymosis and dilated loops of bowel near pancreas seen on Abd XR. CT will show enlarged inflamed pancreatitis, with associated pseudocyst.

Croup

Acute inflammation of the larynx Causes: parainfluenza virus types 1, 2, and 3, RSV, influenza, adenovirus, Mycoplasma Presentation - Affects children 3 months to 5 years, nasal congestion, fever, barky, harsh seal-like cough, dyspnea, retractions, increased work of breathing, inspiratory stridor, expiatory wheezing Dx testing: CXR might show narrowing trachea (steeple sign) Tx: changes in ambient temperature/humidity, humidified oxygen Dexamethasone single dose for mild cases Racemic epinephrine for moderate or severe cases

What drugs are used to treat acute and chronic gout?

Acute: NSAIDs, Ibuprofen, Naproxen, Colchicine, Glucocorticoids (intraarticular/oral) Chronic: Allopurinol, Febuxostat, Probenecid

An 8 year old boy with a pmh of asthma comes into the office for his well child exam. His only medication is an albuterol inhaler. He has no complaints and reports that his asthma is under good control. He rarely uses his albuterol inhaler during the day, and at night uses it 3-4 times per month. What is the most appropriate next step in this patient's management?

Add low dose inhaled steroid

What is the treatment for a pt who presents two hours after the onset of am embolic ischemic stroke?

Admit to ICU Alteplase 4.5 hours BP control - 185/110 if thrombolysis - 220/120 if not ASA + Dipyrimadole/Plavix Anticoag if a fib with heparin or LMWH Statins within first 3 days

Further testing confirms the diagnosis of DKA. What are the initial steps in managing this patient?

Admit to ICU Central IV access IV regular insulin IV fluids (NS) IV potassium Stop the insulin drip when anion gap normalizes Search for the underlying cause of DKA

What is the treatment for a SAH?

Admit to ICU DC all anticoagulants Ventriculostomy to monitor ICP in select patients SBP < 160 only if cognitive function is intact (adequate cerebral perfusion pressure) until the aneurysm is clipped to prevent rebleeding. If the cerebral perfusion pressure is not adequate, then lowering the BP with labetalol is preferred. AVOID nitroprusside and nitroglycerin which can cause increase intracranial pressure Oral nimodipine to prevent vasospasm q4 hours for 3 weeks Prevent physiologic derangements that may worsen brain injury like avoiding hypoxia, hypoglycemia, maintaining normal pH, euvolemia, and or normothermia Surgical clipping or insertion of a metal cord into aneurysm is treatment definitive.

How should an aortic dissection be managed?

Admit to ICU Morphine Control BP with IV esmolol, labetalol, propanolol/nitroprusside/enalapril Stanford A (involvement of ascending aorta) => emergent surgical repair Stanford B (confined to descending aorta) => medical management unless LE ischemia, viscerial ischemia, or pain w control of BP

A 76 yo woman is diagnosed with community acquired pneumonia and sepsis. She has a hx of PMR which has been controlled with prednisone 10 mg daily. She is admitted to the ICU and appropriate antibiotics are started. She subsequently develops profound hypotension, which does not correct with aggressive IV normal saline and vasopressors. What pathological process may account for her persistent hypotension and how should it be treated?

Adrenal insufficiency - IV Glucocorticoids - Hydrocortisone

Granulomatosis with polyangiitis (Wegener granulomatosis)

Affects the upper airway (sinusitis, otitis media) Affects the lungs (pulmonary lesions, hemoptysis) Positive c-ANCA Tx: Cyclophosphamide + corticosteroids

Who should receive a CXR prior to surgery?

Age > 50 Known Lung disease Anticipated surgery time > 3 hours * Preop abx with COPD and pulmonary infection

Preoperative risk factors?

Age > 70 Pulmonary problems (COPD, tobacco - stopping smoking 8 weeks before surgery) Heart disease (recent MIs, arrhythmias, significant valuvular disease, decompensated heart failure) Renal insufficiency Riskier surgeries (CEA, AAA repair, CABG, large blood loss)

A 68 year old man is brought to the physician because of a 3 year history of gradually worsening vision. He says that objects in the center of the visual field appear blurry. He has no history of serious illness and takes no medications. He has smoked one pack of cigs daily for 20 years. Opthlamology examination shows multiple small yellow spots in the maculae bilaterally. What recommendations should be made in order to slow the progression of this disease?

Age-Related Macular Degeneration Dry type - Drusen - Slow, gradual course Wet type - New blood vessels - Rapid course - Severe vision loss Smoking Cessation AREDS2 daily supplement - Vit C, Vit E, Zinc, Cu, Lutein, Zeaxanthin

Trauma Assessment

Airway Breathing Circulation Disability Exposure Secondary Survey

What are the general treatments for a patient with sepsis?

Airway and breathing: oxygen, monitor pulse ox, secure airway if necessary Circulation - Indicators for inadequate perfusion: BP, Temperature of the extremities, Uoutput and renal function, serum lactate level - Tx for hypotension: IVF (keep MAP >65), Vasopressor (norepi), IV glucocorticoids Identify and treat underlying infection: - Blood/urine/sputum cx - CXR - Culture surgical incision - Culture catheters - Broad spectrum IV abx Nutrition: tube feeds or TPN Insulin to maintain glucose 140-180

Which patients should receive the HPV vaccine?

All girls and boys at age 11 or 12 all men < 21 The following patients if <26 - All women - Men who have sex with other men - Men with compromised immune system

A 62 year old woman has a 7 mm solid solitary pulmonary nodule in the left lower lung. The woman has no smoking history, but she does have a family hx of lung cancer. What is the most appropriate management for this patient?

All nodules <8 mm, serial CT scan Growth => Bx

A 46 year old man comes to clinic with sneezing, nasal congestion, watery nasal discharge, and itching eyes. He has similar sx every spring and fall. Physical examination shows nasal congestion and non-purulent nasal discharge. There is no cervical lymphadenopathy. What tx options are available to treat this condition?

Allergic rhinitis - Glucocorticoid nasal spray for 2 hours to a week or 2 - Nasal decongestant - oxymeterizolne - Antihistamine - loratidine, fexofenadine, cetirizine - sbuQ immunotherapy injection - nasal rinse - montelukast - cromolyn - Ipratropium nasal spray

A 76 year old woman presents with progressive loss of executive function, personality changes, and memory loss over the last two years. What is the differential diagnosis of a patient with dementia?

Alzheimer's Vascular dementia Dementia with Lewy Bodies - Parkinsonian Parkinson disease Frontotemporal dementia (pick's disease) Huntington disease Wilson disease - cu builds up -> dysarthria, ataxia, tremor, dementia HIV-associated dementia Neurosyphilis Vitamin B12 deficiency Hypothyroidism NPH CJD Brain tumors

What drugs are used in treating a stable, asymptomatic ventricular tachycardia?

Amiodarone - 1st line Procainamide Lidocaine

TCAs and SEs

Amitryptiline, Clomipramine, Desipramine, Doxepin, Imipramine, Nortriptyline Anticholinergic effects, sedation, sexual dysfunction, weight gain, dangerous in overdose

What is the treatment for a flail test

Analgesia and resp support

A 19 year old man is brought to the emergency department with pruritus, hives, and difficulty breathing that began afer being stun by a wasp. His bp is 82/46 and pulse is 104/min. How should this patient be managed?

Anaphylactic shock IM Epi Supp O2, and possibly intubation IVF Antihistamines Albuterol Glucorticoids Monitor 8-10 hrs after

How does multiple myeloma present?

Anemia Weakness and fatigue Weight loss Back pain and bone pain Pathologic fractures, vertebral fractures, cord compression Renal failure Hypercalcemia Immunosuppression

What hematologic problem is associated with chronic renal disease?

Anemia (due to decreased EPO) IVF Follow Urine output and creatinine

What is the differential diagnosis for metabolic acidosis?

Anion Gap: MUDPILES Non-anion gap: Diarrhea or RTA depending on positive urine anion gap (positive is RTA, negative is diarrhea)

A 73 yo man with a past medical history of stable angina comes to the physician with chest pain. His troponin I is elevated and his EKG shows STEMI in leads V2, V3, V4, V5. Where is the location of his MI and which coronary artery is likely occluded?

Anterior MI LAD

A 36 year old woman develops deep vein thrombosis (DVT) during her first trimester of her pregnancy. WHat is the appropriate management?

Anticoagulate patient - LMWH - Unfractionated Heparin STOP anticoagulant when labor begins or 24 hours before planned delivery Remove AC 12 hours postpartum Heparin + Warfarin postparum for over 6 weeks

A 42 year old woman comes to the physician for redness and swelling of the left lower leg. Her only medication is OCPs. She recently returned from traveling in Asia. A lower extremity doppler us reveals a DVT. What is the most appropriate next step?

Anticoagulate with LMWH; stop after 3 months

Tension Headache

Any patient - Dull HA, mild to moderate pain, usually bilateral - Tx: NSAID

A 61 year old man comes to the physician for recurrent syncope. He reports three episodes of syncope over the last two weeks, each occuring while he was working in his yard. Each episode was preceded dby sob and a feeling of lightheadedness. There were no injuries from any of these events. The third episode was witnessed by his wife, who says the patient regained consciousness after about 15 seconds, and it was she who urged the patient to seek medical attention. VS are pulse 88/min and regular, respirations 12/min, and blood pressure 124/78 mmHg. Cardiac examination shows a grade 2/6 systolic crescendo-decreescendo murmur heard best at the right 2nd intercostal space radiating to the carotids. The radial pulses are weak and dlayed. The physical examination is otherwise unremarkable. An ECG shows NSR and is otherwise unremarkable. What is the most likely cause of this patient's syncopal episodes?

Aortic Stenosis

What are post-MI complications?

Arrhtymias Mural thrombus Ventricular wall rupture (4-8 days after MI) Pericarditis (Dressler syndrome) 2-4 weeks after MI

Differential diagnosis of syncope Cardiovascular syncope

Arrthymias (sick sinus syndrome, conduction abnormality, SVT, Vtach) Structural heart disease - Valvular disease - HOCM - Left atrial myxoma - MI Pulmonary emoblism Cardiac Tamponade

A 62 year old woman presents with tearing chest pain and elevated blood pressure. CXR shows a widened mediastinum and CT of the chest demonstrates a large aortic dissection extending from just proximal to the brachiocephalic artery to the celiac artery. How should this patient be managed?

Ascending Aorta Stanford A - Admit to ICU - IV beta-blocker to control BP - Emergent surgery Descending Aorta Stanford B - manage medically

Smoking Cessation

Ask about tobacco use and encourage patients to quit Refer to behavioral counseling Pharmacotherapy - Nicotine replacement (gum, lozenges, patches, inhalers) - Buproprion (wellbutrin, Zyban) - Varenicline (Chantix) Pharmcotherapy plus cousneling is more effective than either one by itself

What are the steps in the management of a patient with an acute upper GI hemorrhage?

Assess HDS Admit to ICU Type + Screen two units of pRBCs Labs: CBC, PT/PTT, BUN/Cr If bleeding source is uncertain (upper v lower), consider NG lavage Medications: IV PPI, if suspected variceal bleeding - ocreotide EGD

What are the steps in the management of a patient with an acute lower GI hemorrhage?

Assess Hemodynamic stability Type and screen two units of pRBCs Labs: CBC, PT/PTT, BUN/Cr NG lavage and/or EGD to r/o massive upper GI bleed Colonoscopy If colonoscopy is nondiagostic or not feasible (too much active bleeding which obscures visualization) and bleeding persists, consider: - Angiography (AVM) - Radionuclide scan (tagged RBC scan) - small bowel - Capsule endoscopy

What are the clinical features and treatment of each of the stages of syphilis? Early Latent

Asymptomatic (<1 year since acquisition) IM Benzathine PCN G x1

What are the clinical features and treatment of each of the stages of syphilis? Late latent or latent of unknown duration

Asymptomatic (>1 year or unknown duration since acquisition) IM Benzathine PCN G x3

A 32 year old G3P2 woman comes to the physician for her first prenantal visit. She has no complaints aside from mild nausea. A screening urine culture grows 10^5 CFU/mL of E. coli. What is the most appropriate next step in mngt?

Asymptomatic bacteruria Oral Abx: Macrobid, Amoxicillin, Augmentin, Cephalexin for 3-7 days. Fosfomycin - 1 dose Rpt urine culture 1 week after completion of antibiotics, if positive suppressive tx w Macrobid

CLL Tx?

Asymptomatic: no tx needed Warm-agglutinin hemolytic anemia: steroids/splenectomy Symptomatic patients: chemo with fludarabine

A 2 year old boy is brought to the office by his mother because of a 1 year history of dry skin despite frequent application of moisturizing lotion. She says that he constantly scratches his skin. Physical examination shows erythematous patches and scaling on the face, neck, and antecubital and popliteal fossae. What is the most likely dx? The patient is at increased risk for what condition later in life?

Atopic dermatitis (Eczema) Asthma

A 46 year old man presents with depressed mood and gradual cognitive decline. Physical examination reveals prominent involuntary, non-repetitive movements of the arms and trunk. What MRI finding would confirm the diagnosis?

Atrophy of the caudate and putamen.

What is the treatment for SVT?

Attempt vagal maneuvers - Valsalva - Carotid massage IV Adenosine - quick! Stop heart for a minute Ventricular rate control - CCB (diltiazem) - Beta blocker (metoprolol) RFA

What is the first-line pharmacotherapy for Raynaude phenomenon?

Avoid cold, smoking Longacting CCBs (nifedipine, amlodipine)

A patient with a history of genital herpes seeks advice on preventing transmission to her partner. How should she be counseled?

Avoid intercourse during active outbreaks or when prodromal sx are present (itching, burning, pain) HSV may still be transmitted even in the absence of lesions or prodromal sx Condoms reduce but do not eliminate the risk of transmission Suppressive therapy reduces, but does not eliminate the risk of transmission.

What is the treatment for acquired methemoglobinemia?

Avoid triggering agent - Dapsone, topical anesthetics, pilocarpine, lidocaine, nitrites, aniline dyes Methylene blue or Ascorbic Acid in G6PD def patients Blood exchange/transfusion Hyperbaric oxygen

A 28 yo Arizona woman comes to the physician after testing positive for HIV during routine screening. Lab studies show a CD4+ T lymphocyte count of 45. Which of the following is the most appropriate regimen for preventing opportunistic infections in this patient?

Azithromycin, TMP-SMX

Which drugs are most commonly used to control the HR in a patient in atrial flutter?

B-blocker (metoprolol) Non-dihydropyridine CCB (diltiazem)

Workup for amenorrhea

B-hCG, FSH, Prolactin, TSH Progestin challenge - + withdrawal bleeding => no outflow tract obstruction and most likely anovulation (PCOS, functional hypothalamic amenorrhea) - (-) withdrawal bleeding => estrogen-progesterone challenge => (-) => ashermann syndrome, anatomic problems/ (+) => Premature ovarian failure

A 34 year old homeless man with a history of chronic alcohol abuse is brought to the emergency department by the police. He is disoriented, inattentive, and largely uncooperative with the history and examination. He is unable to stand without assistance. When he attempts to walk, his gait is notable for short steps and a wide stance. Neuro examination also shows lateral nystagmus when looking to either side. What treatment should be administered immediately?

B1 thiamine def. Tx: Immediate IV THIAMINE

What is the mngt of preeclampsia with severe features?

BP > 160/110 End-organ dysfunction: thrombocytopenia, renal insufficiency, impaired liver fxn, pulmonary edema, cerebral or visual disturbance BP Control <160/110 - IV hydralazine, IV labetalol, oral nifedipine Sz ppx - IV Mag sulfate DELIVERY

Preeclampsia with severe features

BP > 160/110 Thrombocytopenia Renal insufficiency Impaired liver function Pulmonary edema Cerebral or visual symptoms

A 62 year old man comes to the clinic with urinary hesitancy, frequency and a weak urinary stream, gradually worsening over 3 months. There is no dysuria or hematuria. On examination, the prostate is symmetrically enlarged smooth, and nontender. The genital examination is normal. UA is normal. What non-surgical option are available to treat this condition?

BPH - alpha1 blockers = nonselective - doxazosin, terazosin, alfuzosin = selective - tamsulosin, silodosin - 5 alpha reductase inhibitor = finasteride = dutasteride

A 56 year old man complains of dizziness and fatigue. Earlier today, he participated in his first marathon. He has cracked lips and dry mucous membranes. His creatinine is 1.5 mg/dL and his BUN is 50 mg/dL. What would you expect his fractional excretion of sodium to be?

BUN/Cr > 20 = prerenal FeNa should be low! due to reabsorption to preserve more fluid

What are the dx for ischemic colitis?

Barium enema Diffuse submucosal changes (thumbprint sign) Sigmoidoscopy - bloody and edematous mucosa CT - air within bowel wall, bowel wall thickening

a 36-year-old race car driver is brought to the emergency department 1 day after crashing his car during practice. His wife insisted he come to the hospital after she noticed bruising on his face. Physical examination shows ecchymosis around both eyes and b.i.d. years. There is a continuous leakage of clear watery fluid from his nose. What is the most likely diagnosis? What is the management?

Basilar skull fracture admit for observation, CSF leak is self limited, noncontrast CT of head

Why is the volume of the lung cavities problematic for patients with hemothorax?

Because it can lead to hypovolemic shock and increased pressure on the mediastinum and trachea. Treatment is with chest tube

A 20 year old woman presents with two days of right sided facial droop and decreased taste sensation on the anterior portion of her tonuge. What is the most likely cause of the patient's condition? What treatmnet is indicated?

Bell's palsy Steroids +/- acyclovir

What cardiovascular problems are associated with Polycystic kidney disease?

Berry aneurysms - SAH MVP - midsystolic click

A 15 yo sprinter experiences an episode of syncope during a track meet. His echocardiogram demonstrates hypertrophy of the interventricular septum and moderate LV outflow tract obstruction. What do you recommend?

Beta blockers, Avoid dehydration, Limit physical exercise, f/u echo

What medications can cause hypokalemia?

Beta-agonist Thiazides Loop diuretics Chloroquine Insulin

What is treatment for HOCM?

Beta-blocker, avoid dehydration, avoid diuretics, limit physical activity

What medication is given before 34 wks GA to promote fetal lung maturity?

Betamethasone

PID Presentation

Bilateral lower abdominal pain - often worse with intercourse - may begin during or within one week of menses Vaginal bleeding New vaginal discharge

GPIIb/IIIa inhibitors Abiciximab Eptifibatide Tirofiban

Binds GpII/IIIa on platelet surface => prevents crosslinking of platelets with fibrinogen

What medications are indicated to reduce mortality in patients with CHF?

Bisoprolol, carvedilol, ER metorpolol ACE-i ARB Aldosterone antagonist ASA

What about nonpharm treatment?

Biventricular pacing when there is prolonged QRS (Class III/IV) ICD (<35% eF) Heart Transplant Class (III/IV)

Describe the mngt of postpartum hemorrhage due to uterine atony

Blood loss >500 mL - vaginal delivery Blood loss >1L - c section MCC: Uterine atony Fundal or bimanual massage Explore uterine cavity for retained tissue Uterotonic agent - oxytocin, methylergonovine (contraindicated in hypertension), carboprost (contraindicated in asthma) IVF +/- blood transfusion uterine artery ligation/Hysterectomy

What is the differential diagnosis for 3rd trimester bleeding?

Bloody show, Placenta Previa, Vasa Previa, Placenta abruption, Uterine rupture

What is the differential diagnosis for third trimester bleeding?

Bloody show, Placenta Previa, Vasa Previa, Placental Abruption, Uterine Rupture

A 13 year old boy is brought to the physician because of a painful "boil" on his back. Physical examination shows a 2 cm fluctuant, erythematous nodule that is tender to the touch. What is the most appropriate management?

Boil, furuncle is a small abscess surrounding hair follicle Carbuncle: multiple hair follicles Tx: warm compress - small Incision and drainage - larger (abx: clindamycin/bactrim)

How does the presentation of serotonin syndrome differ from the presentation of Neuroleptic malignant syndrome?

Both cause autonomic instability, hyperthermia, and muscle problems. But serotonin syndrome has rapid onset of hyperkinesia, overactivity, and clonus. NMS has gradual onset of bradykinesia and rigidity.

What's the differences in the presentation between a branchial cleft cyst and a thyroglossal duct cyst

Brachial cleft cyst - lateral neck Thyroglossal duct - midline (moves with swallowing), ectopic thyroid tissue (most commonly around the tongue)

A 22 year old man is brought to the emergency department with a penetrating stab wound to the back. He complains of decreased sensation. Physical examination shows a 1 inch laceration near the spine. Neurological examination shows a loss of proprioception and vibration sense on the left side and pain and temperature loss on his right side. Where is the lesion located?

Brown-Sequard syndrome Hemisection of spinal canal. Left side

Atypical antidepressants and side effects

Buproprion - no sex dysfunction; help with smoking cessation Mirtazapine - sedation and weight gain

Which antibiotics inhibit protein synthesis by binding to the 30S subunit of bacterial ribosomes? The 50S subunit?

Buy AT 30 Aminoglycosides - Gentamicin, Neomycin, Tobramycin Tetracyclines - Tetracycyline, Doxycycline CCEL at 50 - Clindamycin - Chloramphenicol - Erythromycin (macrolides): Azithromycin, Clarithromycin - Linezolid

Absolute risk reduction

C/C+D - A/A+B

What tests should be ordered in a patient with suspected MM?

CBC + peripheral smear BMP SPEP (monoclonal protein M spike) UPEP (Ig light chains - Bence Jones) Skeletal survey (lytic lesions) B2 microglobulin (high levels => low prognosis) BM Bx (increase plasma cells)

A 56 year old woman comes to the physician with fever, chills, sob, and cough productive of rust-colored sputum. She has difficulty performing activities of daily living because of exertional dyspnea. An x-ray of the chest shows a left lower lobe consolidation. What is the most appropriate next step in the management of this patient?

CBC + sputum cx IV Rocephin + azithromycin/Resp Fluroquinolone If long QT syndrome => switch to Doxycycline

Fever in 29 - 90 day infant (ill appearing)

CBC, Blood cx, UA, urine cx, CSF cell count and cx CXR for any resp sx S. pneumoniae, H. influenzae, N. meningitidis Cefotaxime or Ceftriaxone +/- ampicillin if <8 weeks to cover Listeria +/- vancomycin for skin infections or signs of meningitis

What are other orders in management of unknown overdose/poisoning?

CBC, CMP, UA, EKG, Finger glucose, UDS, ABG, serum acetaminophen level, serum ASA level, other toxins (ETOH, Digoxin, Antiepileptics)

What labs should you order for working up microcytic anemia?

CBC, Peripheral smear, Reticulocyte count, Iron studies +/- Hbg electrophoresis

Fever in the 29-90 day infant

CBC, blood culture, UA, CSF (only with signs of meningitis) CXR for any resp sx Ceftriaxone IM single dose and follow up in 24 hours

Fever without a source in the 3-36 month child (well appearing, immunized incompletely)

CBC, blood cx, UA, CSF (only with signs of meningitis) CXR for any resp sx or if WBC > 20,000 If WBC count > 15,000: IM rocephin single dose

What labs should you order when working up normocytic anemia?

CBC, peripheral smear, reticulocyte count (decreaseed in thalassemia and renal failure, but increased in hemolysis/hemorrhage), Haptoglobin, LDH, total and direct bilirubin

CLL Eval?

CBC: lymphocytosis Peripheral smear: smudge cells Bone marrow biopsy

Opportunistic infections HIV PCP

CD4 < 200 CXR: perihilar patchy infiltrates Dx: Silver stain, DFA Tx: TMP-SMX and steroids if PaO2 <70 Ppx: TMP-TMX when CD4<200, if allergic then use dapsone, aersolized pentamidine

Opportunistic infections HIV MAC

CD4 < 50 Fever, weight loss, night sweats, LAD Dx: blood cultures Ppx: weekly azithromycin when CD4 < 50

Dx of AIDS?

CD4 <200 and opportunistic infection - PCP, esophageal candidiasis, Kaposi Sarcoma, Crytococcal meningitis

What is CHA2DS2-VASC score

CHF - 1 HTN - 1 Age > 75 - 2 DM - 1 prior Stroke - 2 Vascular - 1 Age 65-74 - 1 Female - 1 >2 - AC

A 66 year old woman comes to the emergency department with acute dyspnea. She has a hx of systolic heart failure. Her temperature is 37C, pulse is 106/min, and regular, respirations are 22/min, and bp is 112/74. Pulse oximetry on room air shows an oxygen saturation of 89%. Physical examination shows JVD, bilateral crackles halway up the posterior lung fields, S3 gallop, and 3+ pitting edema. There is no cardiac murmur. What is th emost appropriate therapy at this time?

CHF Exacerbation (NO LIP) - Nitrates (NTG) - dilate periopheral veins to decrease preload - oxygen - Loop diuretics - Inotropic drugs (dobutamine, milirinone) - Position (legs down)

A 38 year old stockbroker has been hospitalized because of progressive cognitive decline that has manifested itself over a period of one month. He startles easily every time you enter the patient's room and exhibits diffuse myoclonic jerking. What diagnostic test should be performed to confirm the diagnosis?

CJD LP - test CSF for elevated 14-3-3 protein

A 72 year old man complains of occasional mild shortness of breath and a frequent, non-productive cough. He has smoked 2 packs of cigarettes daily for 45 years. On examination, he has diminished breath sounds diffusely. PFTs are performed and his FEV1 is 48% of predicted. What is the most appropriate tx?

COPD - quit smoking - short acting bronchodilator (albuterol, ipratropium) - long-acting bronchodilator (salmeterol, tiotropium) - long-acting nhaled steroid (fluticasone, budenoside)

What is the treatment for pulseless electrical activity (PEA) or asystole?

CPR 30:2 for 2 mins Epinephrine 1 mg q3-5 min CPR for 2 minutes and eval and tx reversible causes Rpt Epi => CPR 2 min => Epi

HSV Encephalitis?

CSF has high wbc, high protein, normal glucose, high rbc count. Confirm dx with PCR. 70% mortality Tx: High dose IV acyclovir for 14-21 days. IVF

What is the appropriate next step in managemetn of patient?

CT Chest w contrast

A 76 year old man is brought to the ED for severe headache, fever, and altered mental status. The physical exmination shows lethargy and confusion. Funduscopic examination shows papilledema. He has nuchal rigidity but no focal neruo deficits. What should be ordered prior to performing a lumbar puncture?

CT before LP for - AMS - Papilledema - Sz within past week - Immunocompromised state - Focal Neurologic deficits

A 68 year old man comes reluctantly to the ED at the insistence of his wife. The patient woke up three hours ago with slurred speech and weakness of the right side of his face. It seems to be getting better but has not completely resolved. He is hemodynamically stable, and physical examination shows no other abnormality. What should be the first imaging study performed in this patient?

CT head w/o contrast

What imaging is most often used to diagnose appendicits?

CT scan with IV and oral contrast - allows visualziation of entire abdomen. CT scan w rectal contrast may be faster CT scan w/o contrast has higher rate of false negatives

What can be seen on CT with diverticulitis

CT will showed increased soft tissue density, colonic diverticula, bowel wall thickening, and abscess formation

A 2 year old girl is brought to the physician for a 2 day history of fever. The parents report no other ill-like symptoms. The child is lethargic and clings to her mother. Her immunizations are up to date. Her temperature is 103F and pulse is 90/min and respirations are 30/min and blood pressure is 90/55. CBC shows a leukocyte count of 22,000. UA shows no abnormalities. Blood and urine cultures are obtained. What is the most appropriate management?

CXR - if ill then hospitalize, can't have adequate orla intake - If o/p: single dose of ceftriaxone, F/u in 24 hours

A 2-year-old girl is brought to clinic for a 2 day history of fever 103 F. The patient reports no other ill symptoms. The child is lethargic and claims to mother. Immunizations are current. Vital signs stable, physical exam normal. White blood cell count is 22000. Urinalysis is normal. Blood and urine cultures obtained. Appropriate next?

CXR for WBC > 20,000 IM rocephin if 24 hours after

A 75 yo man with a long history of tobacco use and ischemic heart disease presents to the ED with acute onset of orthopnea and dyspnea at rest. The physical examination demonstrates an obese man in acute respiratory distress with JVD to the angle of the jaw, an S3 gallop, diffuse bilateral rales, and 3+ pitting pretibial edema. His pulsse ox on room air is 94%. What is the most appropriate immediate intervention?

CXR, EKG, Echo, Loop diuretics, Morphine, Nitrates, Oxygen, Positioning/Pressors

Ranson criteria (48 hours)? CALvin and HOBBes

Ca < 8 Hematocrit (fall by >10%) Oxygen (PaO2 <60) BUN (increase by >5 mg/dL) Base deficit > 4 mEq/L Sequestration of Fluid > 6L

A 60 year old Caucasian woman undergoes screening for osteoperosis. A DEXA scan shows reduced bone mineral density with a T-score of -2.7. In addition to starting treatment with a bisphosphonate, what lifestyle counseling should she receive?

Calcium + Vitamin D Weight Bearing Exercises Fall Prevention Avoid smoking and alcohol intake

What is the mngt of shoulder dystocia?

Call for help Suprapubic pressure McRoberts maneuver Deliver posterior shoulder Zanavelli: push head back in and STAT C-section

What is the difference between caput scuccedaneum and a cephalohematoma?

Caput succedaneum: diffuse swelling of scalp, cross suture lines, resolves within days, caused by pressure on head during live birth Cephalohematoma: subperiosteal hemorrhage, does not cross suture lines, resolves weeks-months, can indicate skull fracture

A 46 year old man with a history of epilepsy complains of fever and a productive cough for the past four days. He had a mildly pruritic rash on his chest for two days, but it resolved spontaneously. CXR is normal. Labs reveal agranulocytosis. What antiepileptic medication is he likely taking?

Carbamazepine

What features TCA overdose?

Cardiotoxicity: tachycardia, hypotension, conduction abnormalities CNS toxicity: sedation, obtundation, coma, seizures antiCholinergic sxs: Mydriasis, Xerostomia, Ileus, Urinary retention

Genital warts (condyloma acuminata)

Causative organism: HPV - Types 6 and 11 cause majority of genital warts - Types 16 and 18 cause majority of cervical cancers (also associated with penile, anal, vulvar, vaginal, and head and neck cancers) Dx: physical exam show flesh-colored, "cauliflower-like" papules in genital region - Lesions will turn white with application of 5% acetic acid

Urge incontinence

Cause: Detrusor muscle overactivity Sx: Preceded by sudden urge to void Tx: Lifestyle, Antimuscarinics (oxybutynin)

Stress incontinence

Cause: weakening of pelvic floor Sx: occur with increase in abdominal pressure Tx: lifestyle, pessary, surgery

What are the treatment options for condylomata acuminata?

Caused by HPV - Podophyllin - Trichloroacetic Acid - 5-fu - Imiquimod - IFN-alpha - Cryotherapy - Laser therapy - Surgical excision

Epiglottitis

Causes: most common due to HiB, Staph, and Strep Inflammation and swelling leading to upper airway obstruction Presentation - Children ages 2-7 years - High fever, oral intake, sore throat, dysphagia - Toxic appearing, drooling, respiratory distress, stridor, hypoxia, voice change or muffled crying, "tripoding" Dx testing: lateral x-ray may show a "thumb" sign Tx: keep child calm until able to intubate if necessary Tracheostomy if intubation unsuccessful culture of blood and epiglottis Abs: clindamycin or vancomycin (s. aureus), ceftriaxone or cefotaxime (HiB)

SAH

Causes: rupture of berry aneurysm, AVM, trauma "worst headache of my life" Dx: CT scan +/- LP => MRA Treatment: clip aneurysm, nimodipine w/in 4 days onset and continue for 3 weeks. High mortality with rebleeding

A term 4 day old newborn born by C/S due to fetal distress develops a temperature of 101.2F. The patient has a witnessed seizure that lasts less than one minute. What anti-infective treatment regimen is most appropriate for this patient?

Cefotaxime + Ampicillin +/- Acyclovir

Sickle Cell Disease Treat any fever empirically with?

Ceftriaxone

What is the recommended empiric therapy for a 75 year old man with suspected bacterial meningitis?

Ceftriaxone + Vancomycin + Ampicillin

What is the recommended empiric therapy for a 35 year old man with suspected bacterial meningitis and gram-positive cocci seen on gram stain?

Ceftriaxone/cefotaxime + Vancomycin + Dexa (prior or with 1st dose of antibiotics)

A 30-year-old G2POA1 woman at 15 weeks gestation comes in for a routine prenatal visit. She denies any contractions. She has a history of a prior second trimester pregnancy loss, and she a LEEP performed seven years ago because of high-grade cervical dysplasia. On physical exam, the cervix is 1 cm dilated. TVUS shows a shortened cervical length. What is the appropriate management?

Cervical insufficiency -painless cervical dilation during 2nd trimester Tx: Placement of Cerclage

A 7 year old boy is brought to the pediatrician because of poor performance in school. His teacher says that he is unable to grasp many of the lessons in class and that he is often not paying attention in class. What is the most appropriate next step in the diagnosis of this patient?

Check Hearing and Vision

Outpatient treatment of CAP?

Children < 5 years: Amoxicillin Older children and adults: - azithromycin, doxycycline, clarithromycin Abx in last three months or comorbiditis: - Levo or moxifloxacin - Beta-lactam (amoxicillin) plus macrolide

What are the most likely causes of Reye syndrome?

Children who receive ASA for viral syndrome Rash, vomiting, hepatoencephalopathy, HA, confusion, hypoglycemia, stupor, coma, death Kawasaki: Tx high dose ASA Tx: Supportive

What drugs are used to treat Alzheimer disease?

Cholinesterase-inhibitor: Donepezil, Rivastigmine, Galantamine NMDA-antagonist: Memantine

What is the differential diagnosis for primary amenorrhea?

Chromosomal abnormalities with gonadal dysgensis - Turner syndrome (due to 45XO karyotype) Anatomic anomalies - Mullerian agenesis - Imperforate hymen (associated with cyclic pain) Congenital GnRH Deficiency - Kallmann syndrome (associated with anosmia) Androgen insensitivity (due to mutated androgen receptor) - 46XY - phenotype female Functional hypothalamic amenorrhea: associated with eating disorder, excuse, and stress

BP > 140/90 before 20 weeks gestation BP > 140/90 after 20 weeks gestation with proteinuria or end-organ dysfunction Preeclampsia + seizure BP > 140/90 after 20 weeks gestation with no proteinuria or end-organ dysfunction

Chronic HTN Preeclampsia Eclampsia Gestational HTN

COPD subtypes

Chronic obstructive pulmonary disease - disease of the airways that causes chronic, progressive airflow obstruction, +/- inflammatory component Emphysema - destruction of the alveolar walls, resulting in enlargement of distal air spaces Chronic bronchitis - inflammation of the bronchial tree - Chronic productive cough for at least 3 months/year for 2 consecutive years.

CML Presentation?

Chronic phase: fatigue, night sweats, abnormal bleeding, leukocytosis Blast crisis: myeloblasts, splenomegaly, abd pain

Autoimmune hemolytic anemia

Circulating autoantibodies bind to the patient's RBCs "Warm agglutinins" (IgG) - Tx: steroids or splenectomy "Cold agglutinins" (IgM) - Tx: avoid cold temps

Class SSRI and SEs

Citalopram, Escitalopram, Fluoxetine, Fluvoxamine, Paroxetine (SIADH), Sertraline Sex dysfunction, insomnia/agitation, weight gain

GERD in infants

Clinical dx pH probe upper GI series Endoscopy be performed for tx failure Labs: stool test for occult blood, cbc, met panel, celiac dz panel

What is the w/u for a palpable breast movies?

Clinical exam Imaging: Diagnostic mammogram, ultrasound Needle Biopsy Triple Test => Negative then >99.9% benign => Positive then Excisional biopsy

A 30 year old homeless man was admitted to the hospital with severe head trauma after being struck by an automobile. He has been on life support for the last two weeks. There is no advanced directive or living will on file. Who is authorized to make decisions about withdrawal of life support?

Closest relative and/or physician

Etiologic agents of bacterial meningitis with empiric therapies Neurosurgery

Coag negative staph, s. aureus, gram neg rods including pseudomonas Tx: Vancomycin + Cefepime/Meropenem

What are the C's of Huntington's disease

Cognitive decline (dementia) Chorea (rapid, involuntary, non-repetitive movements of the trunks and limbs) Cuaranta = 40 = age of onset CAG repeat disorder on chromosome 4 MRI shows atrophy of the caudate and putamen

Colon cancer screening

Colonoscopy every 10 years for everyone starting at age 50, stop @ 75 - Patients with significant family hx of colon cancer should begin screening at age 40 (or 10 year before the age at which the family member was diagnosed) Alternate screening - Flexible sigmoidoscopy every 5 years + FOBT every 3 years - FOBT annually - Double contrast CT q5 years CT colongraphy (virtual colonoscopy) is not recommended

What is the most appropriate next step in mngt given each of the following abnormal Pap results? ASC-H

Colposcopy

Antihypertensives for long term managment

Commonly used in pregnancy - Hydralazine - Methyldopa - Beta blockers (Labetalol) - CCBs (nifedipine) Contraindicated: - ACE-i and ARB => fetal renal and cardiac abnormalities - Spironolactone => feminization of male fetuses

A 29 year old primigravid woman at eight weeks gestation comes to the physician for her first prenatal visit. She complains of constant nausea and frequent emesis. She also says that she has had vaginal spotting over the past week. On physical exam, the uterus is consistent in size with a 16 week gestation. Her serum B-HCG concentration is markedly elevated (>100,000) U/S shows a "snowstorm" appearance of the utereus but does not demonstrate a fetus. What is most likely diagnosis? What is the management?

Complete Hyaditiform MOle Evacuation by suction curretage and then serial B-HCG monitoring

A 7 year old boy has nocturnal enuresis most nights. His parents have attempted motivational therapy in the past with no improvement. He denies any dysuria and has no problems with daytime enuresis. UA shows no abnormalities. What is the most appropriate long-term therapy for this patient?

Complete physical exam Enuresis Alarm

A 2 month old boy is brought to the physician by his aunt, who is babysitting him while his parents are out of town. She is concerned that the boy is being abused. She says, "When I change his diaper, I noticed a large bruise on his buttocks." He appears well nourished and is in no apparent distress. Physical examination shows an 8 x 10 cm blue-gray macule over the lumbosacral region. There is no edema or tenderness to palpation. What is the most appropriate next step in mngt?

Congenital dermal melanocytosis "Mongolian Spot" Tx: Reassurance

A 21 year old primigravid woman at 10 weeks gestation has a positive test result for syphilis. she has a history of allergy to penicillin. What is the most appropriate therapy for this patient?

Congential infection - Growth restriction, Prematurity, Stillbirth, Snuffles, Hutchinson teeth, Saber shins Screen w/ RPR/VDRL - 1st visit, 3rd trimester in high risk - confirm with FTA-ABS/MHA-TP Desensitiziation IM Benzothine PCN G

Congenital diaphragmatic hernia

Contents of abdomen protrude into chest resulting in pulmonary hypoplasia Presentation: resp distress, barrel chest, and scaphoid-appearing abdomen 50% will have chromosomal abnormalities, congenital heart disease or neural tube defects Tx: Immediate intubation and resp support

What is the treatment for mastitis in a postpartum patient?

Continue to breast feed with pump Antistaphylococcal PCN Abscess? I+D

Anterior cerebral artery stroke

Contralateral lower extremity weakness (more severe) Contralateral upper extremity weakness (less severe) Personality changes Urinary incontinence

MCA stroke

Contralateral upper extremity weakness (more severe) Contralateral lower extremity weakness (less severe) Speech abnormalities Eye deviation toward lesion Most common artery involved in embolic stroke

Thrombolytics - alteplase - reteplase - tenectaplase

Convert plasminogen to plasmin => cleaves fibrin clots

Methods of contraception Other

Copper IUD (paragard) Notes: Irregular bleeding, heavy periods, dysmenorrhea, uterine perforation, expulsion, increase risk of PID within 3 weeks of insertion

what is the most common treatment for amblyopia?

Correct underlying vision problem. Patch over the good eye

A 19 year old woman comes to your office for a f/u visit. She has been seeing you for severe nodular acne that has not responded to topical or oral medications. You are considering starting oral isotretinoin. What should be done prior to initiating therapy?

Counseling + Education 2 neg Pregnancy tests + repeat neg 2 forms of birth control Check lipids, LFTs, CBC (bone marrow low cbc)

Veterbrobasilar artery stroke

Cranial nerve abnormalities Contralateral body weakness Decreased sensation Vertigo Loss of coordination Speech problems Visual problems Coma

A 2-year-old girl is brought to the clinic with low-grade fever, runny nose, and a bark-like cough for the past three days. She is somewhat tired but has been playing regularly during the day, and her cough worsens during the nighttime. On examination, the patient has coarse breath sounds but no wheezing or stridor. The patient's oxygen saturation is 99%. What is the most appropriate next step at this time?

Croup - bark like cough worsening at night Tx: Supportive Care - 1 Dose of Dexamethasone Worse: Admit + Nebulized racemic epinephrine

A 28 year old woman is found to have a 12 cm ovarian cyst with benign sonographic features. The patient is asymptomatic. What is the most appropriate next step in management?

Cystectomy due to risk of ovarian torsion (5-10 cm - watch) >10 cm then resect

A 3 year old boy is hospitalized for severe acute bronchitis. This is his third hospitalization in the past six months. He tends to have a mild cough and nasal congestion most of the time and then becomes more ill as his cough becomes more productive. His mother says he has very foul smelling stools and that at birth he was diagnosed with a meconium ileus. What test will likely confirm the dx in this patient?

Cystic Fibrosis Sweat Chloride Test

A 57 year old male smoker comes to the clinic with a 3 day history of painless hematuria. Physical examination shows no abdominal masses or tenderness. The GU and prostate examinations are normal. The patient provides a urine sample that is grossly bloody, and UA shows no evidence of infection. No urinary casts are seen. What is the most appropriate next step to eval this patient?

Cystoscopy RCC has flank pain

A 23 year old man is brought to the emergency department for mild sob. His symptoms began spontaneously after he was outside mowing the lawn. He is very concerned about a PE because his mother had a PE after she was diagnosed with breast cancer several years ago. His physical exam, CXR, and EKG are normal. What is the most appropriate next step in determining if this patient has a PE?

D-dimer

Specificity

D/B+D

How is idiopathic intracranial hypertension managed?

D/C any inciting agents (excess vitamin A, isotretinion, tetracyclines) Weight loss in obese patients Acetazolamide - first line Invasive treatment options: serial LPs, optic nerve sheath decompression, lumboperitoneal shunting

NPV

D/C+D

What is the treatment for serotonin syndrome?

DC all serotonergic agents => symptoms usually resolve in 24 hours Supportive treatment to normalize VS - Oxygen, IVF, cardiac monitor - If medical treatment for tachycardia or HTN is needed, use short acting agents like esmolol or nitroprusside Sedation with BENZOs If temperature is >41.1C => sedation, paralysis, and ET tube => mechanical cooling (ice, cooling blankets, misting fans) - Paralysis should relieve the hyperthermia cause by muscle activity - There is no benefit in using antipyretics in this situation If agitation despite BZs => serotonin antagonist like cryptoheptadine After resolution of symptoms, assess the need to resume serotonergic agent

Screening for osteoporosis?

DEXA - women > 65 yo - Postmenopausal women <65 with risk factors = <-2.5sD = T score

Which antidiabetic drug best fits each of the following descriptions? MOA: Increases activity of endogenous GLP-1 by inhibiting metabolic enzyme

DPP4-inhibitor

Asthma Moderate persistent

Daily daytime episodes or >1 night time episode 60-80% FEV1 PRN albuterol + mod-dose ICS +/- long acting B2 agonist +/- montelukast

AML Tx?

Daunorubicin + cytarabine In M3 AML, add all-trans retinoic acid

Sideroblastic anemia

Defective heme synthesis leading to microcytic anemia Causes: - congenital - acquired - alcoholism, isoniazid, chloramphenicol, linezolid Ringed sideroblasts on Prussian Blue stain of the bone marrow Tx - Remove the toxin - Phlebotomy or iron chelation to avoid iron overload - Patients on isoniazid should receive vitamin B6

What is the first line treatment for a patient with ventricular tachycardia without a pulse?

Defib

Hypertensive urgency and emergency

Definitions - Urgency: BP > 180/120 without evidence of end-organ damage - Emergency: BP > 180/120 with evidence of end-organ damage Tx: - Admit to ICU - Rapidly reduce DBP to 100 mmHg using IV nitroprusside, labetalol, nicardipine - In the first 2 hours, the DBP should not be reduced by more than 25% to avoid ischemia, then eventually DbP to 100 then oral to 90 in next 2-3 months

What differentiates delirium from dementia?

Delirium: hours to days, fluctuates hour to hour, waxes and wanes consciousness, disogranized thought production, common hallcuinations, common causes are meds, uti, and hypoxemia, prognosis is reversible DEmentia: gradual, consistent and progressive course, no alternations in consciousness, impoverished though production, uncommon hallucinations, common causes are alz, vascular, and generally irreversible

What are the unique features of dementia with Lewy Bodies (DLB)?

Dementia Parkinsonian features (bradykinesia, tremor, cogwheel rigidity, festinating gait) Visual hallucinations Episodes of syncope

What are the unique features of frontotemporal dementia?

Dementia plus inappropriate social behavior Dementia plus progressive aphasia

SNRIs and SEs

Desvenlafaxine, Duloxetine, Venlafaxine Sex dysfunction, insomnia/agitation, nausea, dizziness

What is the management of PCOS?

Diet + Exercise OCPs Spirinolactone Metformin Clomiphene

A 32 year old woman comes to the clinic with epistaxis that began 30 minutes ago. Examination reveals a briskly bleeding excoriation on the anterior portion of the nasal septum. What options are available to stop the bleeding?

Direct Pressure 10-15 mm Oxymetazoline nasal spray (AFrin) Cautery - silver nitrate. Electrical cautery Nasal packing: gauze

Prostate cancer screening

Discuss screening using PSA with/without DRE starting at age 50

What is Beck's triad?

Distended neck veins, muffled heart sounds, and hypotension - seen in cardiac tamponade. Dx: CXR show enlarged cardiac silhouette, Echo shows effusion, EKG shows electrical alternans Tx: pericardiocentesis

A 19 year old primigravid woman at 10 weeks gestation comes for a routine prenatal visit. She complains of frequent nausea and occassional episodes of vomiting. She has gained 2 lbs since her last visit two weeks ago. Physical examination and lab studies are unremarkable. What is the appropriate management of this patient's nausea and vomiting?

Distinguished from Hyperemesis Gravidarium: Severe N/V => dehydration, abnormal labs THIS PATIENT: monitor sickness: eat bland foods, frequent meals, eat slowly, avoidance of trigger Tx: B6 + doxylamine Diphenhydramine Meclizine Zofran

What are some conditoins in association with Down's?

Doudenal atresia Hirschprung's Annular pancreas Celiac disease

What is the classic presentation of Hirschsprung disease?

Due to failure of neural crest cell migration Lack of Auerbach and Meissner plexuses in distal colon Cannot produce normal peristaltic waves to expel feces

Respiratory distress syndrome of the newborn

Due to surfactant deficiency A lecithin/sphingomyelin > 2 suggest fetal lung maturity X-ray shows low lung volumes with diffuse hazy interstitial infiltrates: "ground glass" give intramuscular cortical steroids for labor prior to 34 weeks CPAP for all babies if weak respiratory drive or FiO2 over 0.4, then intubated and give exogenous surfactant

Transfusion reactions Urticarial (allergic) reaction

During transfusion Urticaria Tx: Diphenhydramine, then continue transfusion

Transfusion reactions Acute hemolytic reaction

During transfusion (ABO incompatibility) Fever, hypotension, severe hemolysis, flank pain, red/brown urine, DIC Tx: stop transfusion, aggressive supportive care

Atrial fibrillation

Dx testing - EKG, Electrolytes, low TSH, +/- cardiac enzymes, Echo Management - Acute new onset (<48 hr duration): DC cardioversion followed by anticoagulation - Chronic: Anticoagulation (warfarin, dabigatran, rivaroxaban), evaluate for thrombus with TEE, Rate control with Beta-blocker (metoprolol) or CCB (diltiazem/verapamil)

Intussusception Dx and Risk factors

Dx: Barium enema, Ct, Us Air or saline enema for tx Surgery for refractory risk factors: mecke's, HSP, Cystic fibrosis, Adenovirus, Cancer

A 42 year old man is brought to the ER after a MVA. He complains of shortness of breath and mild chest pain. His vitals are stable and his lung exam is normal. He has distant heart sounds and moderate jugular venous distension. A CXR reveals an enlarged cardiac silhouette. After some time in the ER, his blood pressure decreases to 100/50. What is the likely dx and treatment?

Dx: Cardiac Tamponade Tx: Stable - do an echo Unstable - Pericardiocentesis

UR treating infective endocarditis in a 21-year-old male IV heroin user. Consulting ID colleagues reminds to monitor for evidence of withdrawal. What are the features of withdrawals from opiates?

Dysphoria, restlessness, insomnia, yawning, rhinorrhea, lacrimation, mitral acids, myalgias, arthralgias, nausea, vomiting, cramping, diarrhea, diaphoresis, piloerection

A 34 year old woman is brought to the ED with sharp, left sided chest pain. The pain is relieved when sitting up and leaning forward. Physical examination shows no chest wall tenderness. The lungs are clear to ausucultation. S1 and S2 heart sounds are normal. There is pericardial friction rub. No murmur or gallop is present. What tests would be the most useful in confirming the diagnosis?

ECG - diffuse ST elevation, diffuse PR depression, Diffuse T wave inversion Echo - usually normal +/- pericardial effusion +/- ESR, CRP, WBC CXR- normal

A 62 year old woman presents with tearing chest pain and elevated bp. You suspect a thoracic aortic dissection. What tests should you order for intial dx eval

EKG, CXR, Card enzymes - CKMB/trops, CBC/CMP

What is tx for gallstone pancreatitis?

ERCP, Sphincterotomy, Cholecystectomy

What is the next step in the w/u of a diastolic murmur in an otherwise healthy asymptomatic patient?

Echo always in diastolic murmur

Which antiretroviral should be avoided during pregnancy?

Efavirenz

A 68 year old woman with a hx of varicose veins comes to the physician because of a rash on her lower legs that has developed over the past 3 years. Physical examination shows edema and an erythematous, hyperpigmented scaling rash up to the mid-shins of the lower extremities bilaterally. What is the most appropriate mngt?

Elevate legs, compression stockings, treat underlying venous stasis

What dx workup should be ordered in a patient with newly diagnosed hypertension?

Elevated BP on at least 3 different days Cardiac exam - S4 heart sound, arrhythmia, or murmur Good Eye Exam for Fundoscopic Changes - AV nicking, cotton wool spots, and flame hemorrhages on the retina BMP, EKG, UA, Fasting lipid panel

What do you see in hepatic failure?

Elevated LFTs Jaundice Coagulopathy Hepatic encephalopathy Kidney failure Multisystem failuer

Brain imaging prior to LP is important if you suspect what?

Elevated intracranial pressure Mass lesion Bulging fontanelles, papilledema, new onset seizures, localizing neuro exam HIV, pupillary changes, facial nerve palsies, resp changes

What are causes of ischemic colitis?

Embolus, bowel obstruction, inadequate systemic perfusion, medications, surgery induced vascular compromise

What interventions shold be performed for life-threatening cases of acute cholangitis?

Emergency ERCP decompression IV Pip/Tazo or IV Rocphin + Metronidazole Cholecystectomy

an elderly man with headache following a fall in which he bumped his head on a coffee table. CT scan of the head shows a lens shaped hematoma for with midline deviation. What is the most appropriate management for this patient?

Emergency craniotomy

Opportunistic infections HIV Histoplasmosis

Endemic to Ohio and Mississippi River Valleys PPx: Itraconazole when CD4 < 150

A 24 year old woman comes to the physician because of a 2 year history of severe pain with menses. She complains that her menstrual periods are also accompanied by urinary frequency, dysuria, and pain with defecation. She is sexuallly active with one male partner, and she says that she experiences pain with deep penetration. Physical examination shows tenderness to palpation of the posterior wall of the vagina. What is the most appropriate treatment for this patient?

Endometriosis - NSAIDs: ibuprofen, naproxen - Combination OCPs - GnRH agonist: Leuprolide - Progestin - Danazol - Surgery

A 52 year old man presents with progressively worsening epigasatric pain for the last 3 months. He describes the pain as burning in quality, and it is worse when he eats. He has had a 3 kg weight loss. Stool examination is guaiac negative. Physical examination shows no abnormalities. What is the most appropriate initial step in management and why?

Endoscopy to rule out Gastric Cancer.

What are the most common causes of an epidural hematoma and a subdural hemotoma

Epidural: rupture of the middle meningeal artery Subdural: rupture of bridging veins after head trauma

IgA nephropathy (Berger disease)

Episodic hematuria that begins 1-2 days after an URI Steroid and ACE inhibitors may slow progression to chronic renal failure

A 21 year old man has an outbreak of genital herpes. One week later, he develops painful oral erosions and a rash consisting of raised target lesions on his trunk and extremities. What is the most likely diagnosis?

Erythema Multiforme - Immune mediated reaction after HSV and Mycoplasma pneumoniae Tx: topical corticosteroids, oral antihistamines, oral prednisone

Differential diagnosis of GI bleeding Upper

Esophagitis Esophageal varices (cirrhosis) Mallory Weiss tears Peptic ulcers (gastric ulcer) Gastritis Gastric/duodenal ulcers

What are common causes of dilated cardiomyopathy?

EtOH, Thiamine deficiency, ischemia from CAD, Chagas Dz, Cox B viral myocarditis, cocaine induced ischmeia, Doxorubicin, Peripartum induced, Radiation induced, Hemochromatosis

What is the drug of choice for absence seizures?

Ethusuximide

What is the management of a patient with a penetrating wound to the abdomen?

Ex-Lap

Methods of contraception Barrier

Examples: Condom, Diaphragm, Sponge Notes: Condoms offer best protection against STDs

Hodgkin Lymphoma Eval?

Excisional lymph node biopsy (Reed-sternberg cells) Staging: CXR, PET/CT of chest, abd, and pelvis (with contrast)

What is the treatment of osteoperosis?

Exercise, Smoking cessation, Fall Precaution, Calcium + Vitamin D, Bisphosphonates (alendronate), Raloxifene

Treatment of osteoporosis?

Exercise, smoking cessation, fall prevention Calcium and Vitamin D Bisphosphonates (alendronate, risedronate) - GI side effects - osteonecrosis of jaw - atypical femur fractures Raloxifene

When evaluating a patient with a pleural effusion, what are the differences between an exudate and transudate?

Exudate - specific gravity >1.020 - protein content: high - highly cellular - causes: infection, inflammation, cancer, lymphatic obstruction Transudate - specific gravity <1.02 - Low protein content - Hypocellular - Na+ retention, increased hydrostatic pressure, low oncotic pressure

A 28 year old woman brought to the ER after a MVA, she was restrained by a seatbelt and now complains of diffuse abdominal pain. She has no identifiable source of bleeding. Her blood pressure is 90/40. Other than moderate abdominal tenderness, her exam is normal. Two large-bore IVs are placed and IVF started. What is the most appropriate next step for the patient?

FAST exam - Rapid US to look for blood - if positive - go to OR - if negative - do a CT, but if CT not available, DPL

Lacunar infarcts Dysarthria (clumsy hand syndrome) - least common

Facial weakness, dysarthria, dysphagia, and slight weakness and clumsiness of one hand No sensory or cortical signs

What are the diagnostic criteria for diabetes?

Fasting glucose > 126 or higher Random glucose > 200 or higher with hyperglycemia sx 2-hr glucose tolerance test > 200 or higher Hemoglobin A1c > 6.4

AML Presentation/

Fatigue, anemia, abnormal bleeding, recurrent infections, DIC (especially with M3 subtype)

Vasa Previa

Fetal blood vessels overlie cervical os Classic presentation: painless bleeding that begins with rupture of membranes Treatment: Emergent C/S

What does each of the following fetal heart rate patterns indicate? Early decel Variable decel Late decel Sinusoidal

Fetal head Compression of umbilical cord Hypoxia Severe fetal anemia

Causes of Microcephaly

Fetal toxin exposure Chromosomal trisomies Congenital infection Cranial anatomic abnormalities Metabolic disorders Neural tube defects

PID Supporting criteria?

Fever > 38.3C Mucopurulent discharge Increase WBCs on wet prep of vaginal fluid Increase ESR or CRP (+) test for gonorrhea or chlamydia

Fever in the 0-28 day neonate

Fever > 38C or >100.4F CBC, Blood cx, UA, urine cx, CSF cell count and culture, +/- CRP CXR for any resp sx GBS, E.coli, Listeria, HSV Ampicillin + Cefotaxime or Gentamicin +/- Acyclovir for ill, skin lesions or CSF pleocytosis

Fever without a source in the 3-36 month child (ill appearing)

Fever > 39 or >102.2 CBC, blood cx, UA, CSF (only with signs of meningitis) CXR for any resp symptoms or if WBC > 20,000 S. pneumoniae, N. mengititidis, H. influenzae, S. aureus If unstable: admit and start empiric abx If stable: Ceftriaxone IM single dose and f/u in 24 hours

Encephalitis?

Fever, HA, Mental status changes (behavioral changes, seizure activity)

A 34 year old G3P3 woman comes to the physician because of increasingly heavy menstrual periods over the last two years. On physical examination, the uterus is enlarged with irregular controus. Her hbg concentration is 10.2. U/S shows multiple masses in the uterus consistent with leiomyomata uteri. The patient is very concerned about these findings. How should she be counseled?

Fibroids are benign tumors of smooth muscle of the uterus. Most are symptomatic. Menorrhagia and pelvic pressure are most common. If they want kids, hormonal ocps, endometrial ablation, myomectomy If they don't want kids, uterine artery emoblization, hysterectomy

During what time frame in the postoperative period will a patient's risk of MI be greatest?

First 48 hours

Pemphigus Vulgaris Blluae appearance Presence of oral lesions Treatment

Flaccid, easy to rupture, positive nikolsky sign Almost always Systemic glucocorticoids: prednisone, prednisolone +/- immune modulation: azathioprine, mycophenolate

when immunizations would be recommended in the patient described above?

Flu, pneumococcal, herpes, Tdap booster

What is outpatient treatment of diverticulitis

Fluroquinolone + Metronidazole TMP-SMX plus metronidazole Amoxicillin plus clavalunate

Macrocytosis (MCV > 100)

Folate deficiency B12 deficiency Liver disease Alcohol abuse

What can be seen on radiology for upright x-ray with diverticulitis

Free air under the diaphragm if perforated

What are the most common features of urogenital infection with gonorrhea and/or chlamydia?

Frequently asymptomatic Cervicitis - Vaginal discharge, spotting, postcoital bleeding - Erythematous and friable cervix Urethritis - Frequency, urgency, dysuria - UA: pyuria with no bacteriuria (chlamydia) - Urethral d/c

Trichomonas Vaginalis Vaginal D/C

Frothy, yellow, green fishy odor pH > 4.5 Multiple, pear-shaped trichimonads Metronidazole -> avoid EtOH for at least 3 days

What are the components of a comprehensive geriatric assessment?

Functional capacity Fall Risk Cognition Mood Polypharmacy Social/financial support End-of-life care/advanced care Nutrition and weight changes Continence Vision Hearing Dentition Sexual Function

potential side effects of lithium use in the treatment of bipolar disorder

GI: nausea, vomiting, diarrhea, metallic taste, weight gain CNS effects: depression, tremor, cognitive dulling thyroid: hyper/hypothyroidism, euthyroid goiter nephrogenic diabetes insipidus: polyuria, thirst, polydipsia

Which antidiabetic drug best fits each of the following descriptions? MOA: Increases glucose-dependent insulin release, slows gastric emptying, improves satiety

GLP-1 agonist

GOLD STAGING?

GOLD 1: FEV1>80 predicted (mild) GOLD 2: FEV1 50-80% (moderate) GOLD 3: FEV1 30-50% (severe) GOLD 4: FEV1 < 30% (very severe)

the initial treatments for 21 hydroxylase deficiency?

Glucocorticoid and mineral corticoid

Ranson criteria (admission) GA LAW

Glucose > 200 AST > 250 LDH > 350 Age > 55 WBC > 16,000/mm^3 IF there is 3 or more then there may been worsening prognosis

What interventions should be taken to prevent vision loss due to diabetic retinopathy?

Glycemic control - HbgA1c < 7% BP control - BP < 140/90 Annual dilated fundoscopic exam - Type 1: Start 3-5 years after diagnosis - Type 2: start @ time of diagnosis Laser photocoagulation

How does Goodpasture syndrome differ from granulomatosis with polyangiitis?

Goodpasture affects the lungs but spares the upper airways and + Anti-GBM Abs tx: plasmapharesis + immunosuppressants

A 66 year old man comes to the O/P clinic because of severe pain and swelling in the right foot for the last 12 hours. He cannot bear weight on the foot because of pain. There is no history of injury. Physical examination shows redness, warmth, swelling, tenderness, and decreased range of motion in the left first metatarsophalangeal joint. What is the most likely dx? What is the most appropriate next step in mngt?

Gout Arthrocentesis

A 38 year old man comes to the physician with hemoptysis and recurrent sinusitis. His urine shows microscopic hematuria. What is the most likely dx and what is the treatment?

Granulmatosis with polyangiitis Tx: Cyclophosphamide + Corticosteroids

A 38 year old man presents with hemoptysis and recurrent sinusitis. His urine shows microscopic hematuria. What types of GN does he most likely have, and what is the treatment?

Granulomatosis w Polyangiitis - upper airways (Wegener's) Positive c-ANCA Biopsy Tx: Cyclophosphamide + Glucocorticoids

A 3,232 g female newborn is delivered at term by C/S due to fetal distress. The 4 day old neonate develops a temperature of 101.2F. She has a witnessed seizure that last less than one minute. What antibiotic therapy is most appropriate for this patient?

Group B strep, E. coli Ampicillin with cefotaxime or Ampicillin with Gentamicin If baby has ill-appearing seizure, vesicles, CSF pleocytosis, then add acyclovir

Etiologic agents of bacterial meningitis with empiric therapies 0-1 month

Group B strep, Listeria, E.coli Ampicillin + gentamicin +/- cefotaxime

Etiologic agents of bacterial meningitis with empiric therapies 1-3 months

Group B strep, Listeria, S. pneumoniae, N. mengitidis Tx: Ceftriaxone/Cefotaxime + Vancomycin

What clinical features are suggestive of a brain tumor?

HA - dull, constant, worse at night, worsens over time N/V Seizures Cognitive dysfunction (memory problems, mood changes, fatigue) FND (muscle weakness, sensory deficits, aphasia) Papilledema

A 2 year old boy is brought to the physician because of a rash. His mother says prior to the rash he has an abrupt fever with a temperature of 103.7F but otherwise did not appear ill. After 3-4 days, the fever suddenly improved and was followed by a pink rash that started on the trunk and spread to entire body. Today, his temperature is within normal limits. Other than the rash, the patient has a normal physical examination. what is the most appropriate tx?

HHV-6 - Roseola infectiosum Supportive care is treatment

A 66 year old man undergoes a right total knee replacement for OA. HE is given subcutaneous enoxaparin to prevent DVT. On postoperative day 3, he is discharged to an inpatient rehab unit, and on post-op day 6, the nurse notices some skin necrosis at the site of enoxaparin injection. On post-op day 7, the patient c/o pain and swelling in the left calf and the doppler u/s reveals a DVT in the right popliteal vein. Lab evaluation shows a plt count of 70,000. Prior to admission to inpatient rehab, the plt count was 210,000. What is the most appropriate mngt of this patient's condition?

HIT - induces formation of antibodies against Platelet factor 4 - Thrombocytopnia - plts used STOP Heparin, Enoxparin Confirm for anti-plt 4 ab Select alternate AC - Fondaparinux, Argatiban, Bilavirudin Transition to Warfarin (3-6 mo.) when plt ct is normal.

What are the risk factors associated with a parenchymal hemorrhage?

HIV Vascular malformation Vasculitis Brain Tumors Iatrogenic Cause Bright white on CT scan

A 76 year old woman presents with the new onset of right headed headache, fever, jaw pain, and tenderness of the right temple. The physical exam is otherwise normal. What are the most appropriate next steps in managing this patient?

HIgh dose prednisone ESR = if elevated, temporal artery biopsy

What is the most appropriate next step in mngt given each of the following abnormal Pap results? HSIL

HSIL => Age 21-24 => Colpo HSIL => Age over 25 => LEEP or Colpo

What are risk factors of Aortic dissection?

HTN, Syphilis, Ehlers-Danlos, MARFAN

A 5 year old girl is admitted to the hospital with a 5 day hx of worsening abdominal pain and bloody diarrhea. Lab studies show a hbg of 8.7, plts of 90,000, and a creatinine of 2.7. What is the most likely organism found on stool culture?

HUS - anemia, thrombocytopenia, kidney disease EHEC in contaminated meat

What drugs are known to cause/mimic psychosis in patients?

Hallucinogens, Stimulants, Withdrawals from BZs, alcohol or barbiturates Anabolic steroids Glucocorticoids

A 52 yo white male Presents with chronic pallor and fatigue. There is no history of bleeding. The CBC reviews a hypochromic, microcytic anemia. His iron studies are normal. What is the most appropriate next step?

Hbg electrophoresis

A 48 year old woman presents with pallor and fatigue. There is no history of bleeding. The CBC reveals a hypochromic, microcytic anemia. Her hbg is 9.4 and iron studies are normal. What is the most appropriate next step?

Hbg electrophoresis for thalassemia

A 64 year old asymptomatic woman with a history of obesity and hypertension is found to have a random blood glucose of 216. What is the next step in management?

HbgA1c Fasting glucose 2 hr glucose tolerance test

What screenings should be performed routinely in all diabetic patients, and what is recommended frequency for each?

HbgA1c - twice annually BP - every visit Fasting lipid - annually Urine microalbumin - annually Dilated eye exam - annually Foot exam - annually

What is the classic description of a cutaneous squamous cell carcinoma lesion?

Head + neck - papule, plaque, or nodule - Ulceration - Crusting - Hyperkeratosis - Non-healing ulcer

A 5 year old girl is found to have lice on routine screening at her school. What is the treatment?

Head lice: visualzie - permethrin - malathion - Benzyl alcohol - Ivermectin Lindane (2nd line) - neurotoxin

Physical signs for meningitis?

Headache, fever, nuchal rigidity, photophobia, AMS

Bradycardia

Heart rate < 60 bpm Caused by increased vagal tone, AV nodal disease, sick sinus syndrome, medications, hypothyroidism Presentation - asymptomatic often - +/- fatigue, lightheadedness, syncope/presyncope Tx - If asymptomatic: stop precipitating meds - If symptomatic: IV atropine, pacemaker

What are the treatment options for primary dysmenorrhea?

Heat Therapy NSAIDs: Ibuprofen, Naproxen Combined HRT GnRH: Leuprolide Surgery

what are common causes of restrictive cardiomyopathy?

Hemochromatosis Amyloidosis Sarcoidosis

What is classic triad of findings in HUS

Hemolysis Uremia thrombocytopenia (E. coli O157:H7)

What is the classic pentad of TTP?

Hemolysis uremia thrombocytopenia fever neuro sx (confusion, seizures, coma) Tx: Plasmapharesis

HELLP syndrome

Hemolysis, Elevated Liver Enzymes, Low Platelets

A 1 day old male neonate is diagnosed with severe anemia. He was born to a 21 year old G2P2 woman who did not receive prenatal care during either of her pregnancies. Lab studies confirm the presence of anti-D antibodies in the mother. What intervention could have prevented the development of anemia in this infant?

Hemolytic disease of the newborn - If mom is Rh(-), then she can develop antibodies if fetus is Rh(+) - In future pregnancies, antibodies attack fetal RBCs => SEVERE ANEMIA Prevent by RhoGAM - 28 weeks - Delivery - Risk of Fetomaternal Hemorrhage

Which hepatitis viruses are transmitted through the fecal-oral route?

Hep A + E

A 55 year old man undergoing treatment for chronic hepatitis B comes to the clinic because of new onset jaundice and abdominal swelling. Physical examination shows HSM and ascites. An abd u/s shows a 2 cm mass in the liver. What is the most likely dx?

Hepatocellular Cancer - AFP - secrete EPO => erythrocytosis

What's the empiric antibiotic management for a patient with epiglottitis?

Hib - ceftriaxone or cefotaxime S. aureus - clindamycin or vancomycin

What study is used to diagnose injury to the urethra or bladder following trauma?

High Riding Prostate Blood in urethral meatus Retrograde Urethrogram

A 32 year old woman with uterine fibroids comes to the hospital because of heavy vaginal bleeding for the past two weeks. Vital signs are within normal limits. Her hbg concentration is 8 g/dL. What medications may be used to terminate this episode of prolonged bleeding?

High dose estrogen - Followed by course of progestin Combination OCP taper High Dose Progestin TXA

Tx of GCA?

High dose prednisone

What is the treatment for CO poisoning?

High flow O2 Intubation Hyperbaric oxygen Tx coexisting CN poisoning (amyl nitrite) - Hydroxocobalamin, sodium thiosulfate, amyl or sodium nitrite

PULMONARY EMBOLISM Dx

High likelihood: CT pulmonary angiogram Low likelihood: D-dimer Consider venous doppler US of the lower extremities to evaluate for DVT ABG: Increased A-a gradient CXR is often normal Alternative imaging: V/Q scan, pulmonary angiogram

DVT Dx

High probability: compression US Low probability: D-dimer

A 1 year old boy is brought to the ED after having a tonic-clonic seizure one hour prior to arrival. He has had an intermittent fever as high as 102.8 Farenheit over the past 8-12 hours. According to theparents, he has had no other ill-like sx. The patient has not recieved regular immunizations due to the religous beliefs. On exam, patient is alert and tearful. What is the most appropriate next step for the dx of this patient?

High risk - CBC, UA, Blood cx, will need LP => culture

Treatment of MI First 24 hours

High-intensity Statin Beta blocker (metoprolol, atenolol) Antiplatelet therapy (plavix, prasugrel, or ticagrelor) in addition to aspirin. Parenteral anticoagulant therapy - if undergoing PCI: UPH or bivalirudin - if no PCI: Lovenox or Fondaparinux Keep K+ > 4 and Mg > 2

Meconium aspiration syndrome

Higher risk in postmature infants and in fetal distress, meconium staining of skin, tachypnea, intercostal retractions No intrapartum suction No suction on vigorous infants (depressed respiration, decreased tone, HR<100) CXR: streaky, linear densities, hyperinflated lungs, flat diaphragms O2 as needed Intubation for resp distress Empiric antibiotics for pna or sepsis

Hospital acquired and ventilar associated pna

Hospital acquired pna develops at least 48 hrs after hospitalization Ventilator-associated pna develops being on a ventilator for at least 48 hours Etiology: gram-positive organisms (esp MRSA) and gram-negative rods Tx: - Broad spectrum antibiotic: pip/tazo, cefepime, meropenem, imipenem - Add a second antipseudomonal: levofloxacin, gentamicin - Add vancomycin or linezolid to cover MRSA

Evaluation of syncope?

Hx Physical - Cardiac and Neuro exams - Orthostatic VS Testing: - CBC, BMP, CK-MB, troponin, EKG - If you find a murmur -> echo - If you find FND or headache => CT - If you suspected a seizure => CT+EEG - If dx of vasovagal syncope is not clear, consider Tilt table test

What history and physical exam findings suggest CHF?

Hx: - Dyspnea on exertion - Orthopnea - Paroxymal Nocturnal Dysnpea - Fatigue - Weakness Physical Exam - Tachycardia, Narrow pulse pressure, cool, pale extremities, rales, S3, JVD, Lower extremity edema, Hepatic congestion

Sickle Cell Disease Chronic maintenance therapy?

Hydroxyurea Pneumococcal and meningococcal vaccines Avoid dehyration

What are tx of complications for MM?

Hypercalcemia: IV NS, possibly a bisphosphonates (zoledronic acid) Renal failure: hydration, hemodialysis Skeletal lesions: bisphosphonate Immunodeficiency: Influenza and pneumococcal vaccine

a 29-year-old woman with recently diagnosed depression has been seeing a psychotherapist, but she now wants to add an antidepressant. She remembers that her mother's depression the responded well to the monoamine oxidase inhibitor traynylcypromine 1. Use of MAOIs is limited by concerns over what major side effect? 2. What dietary substance is likely to trigger tihs SE in a patient taking an MAOi?

Hypertensive crisis Tyramine - smoked/aged meats - fermented cheese - most beers and wines - soy sauce

Which endocrine disorder is most likely to cause a fib

Hyperthyroidism

Pyloric stenosis

Hypertrophy of the pyloric sphincter causing obstruction of gastric outlet Presentation - Nonbilious projectile emesis, poor weight gain - Palpable epigastric olive-sized mass Dx testing - Hypochloremic, hypokalemic metabolic acidosis - Barium swallow with thin pyloric channel (string sign) - U/S with increased pyloric muscle thickness Tx: pyloromyotomy

A 3 week old infant is brought to the clinic for frequent projectile vomiting. The patient is formula fed and vomits almost immediately after all meals. On inspection of the growth chart, the patient was at the 50th percentile of weight at one week of age and now is at the 20th percentile. Arterial blood gas and metabolic panel are ordered. What metabolic abnormalities would you expect in this patient?

Hypokalemic, hypochloremic metabolic alkalosis

what medical conditions can cause major depression?

Hypothyroidism, hyperparathyroidism, Parkinson's, CNS neoplasm, pancreatic cancer, strokes especially anterior cerebral artery strokes

What are the H causes?

Hypovolemia => rapid volume resuscitation through multiple IV sites and/or central line Hypoxemia => correct via intubation, chest tube, or oxygen H+ ions => IV push 1-2 amps bicarbonate (commonly needed in a prolonged code) Hyperkalemia => CaCl2 IV push, bicarbonate, insulin/glucose Hypokalemia => administer KCl Hypoglycemia => 1 amp D50 IV push Hypothermia => warming

A 77-year-old woman is being seen for follow up for her CHF. She is asymptomatic on a regimen of furosemide, enalapril, carvedilol and spirinolactone. An EKG shows NSR, QRS duration of 110 msec, and LVH which is unchanged from previous studies. Her most recent echocardiogram showed a stable LV EF of 30%. What non-pharmacological intervention would be appropriate for this patient?

ICD! Since EF is 30% No biventricular pacing (no QRS prolongation)

What is the empiric treatment for gonorrhea/chlamydia?

IM rocephin + Azithromycin

What is the first line pharamcotherapy for a patient with paroxymal supraventricular tachycardia?

IV adenosine

What are common inpatient treatments for CAP?

IV ceftriaxone + azithromycin (QT prolongation) - can use doxycycline instead OR Resp FQ (increase QT too)

What is the treatment for acute alcohol withdrawal?

IV fluids IV supp nutrition: K, Mg, phosphate, Thiamine, glucose BZs: Diazepam, Lorazepam, Chlordiaxepoxide Propofol - very bad Intubation + mechanical ventilatin

A 35 year old man is brought to the ED for hematemesis. He vomited blood prior to arrival, then vomited blood in the triage area, and later filled an emesis basin with blood. He is drunk and tachycardic. What is the next step in managing this patient?

IVF resuscitation

What are the treatment for ischemic colitis?

IVF, Bowel rest, Abx, Surgery

Sickle Cell Disease Tx

IVF, Supp O2, Analgesics (NSAIDs or morphine), Evaluate for infection (cbc, UA, blood cx, CXR)

What is treatment of uterine rupture?

IVF, blood, emergent C/S, surgical repair and/or hysterectomy

What happens in hepatitis B infected moms?

If mom is infected give Hep B immunoglobulin + Hep B vaccine

What are the classic findings of Henoch-Schonlein Purpura?

IgA Vasculitis - most common vasculitis in kids - Palpable purpura w/o thrombocytopenia, below waist - Arthritis/Arthralgia - Abdominal Pain - Renal Disease

CML Tx?

Imatinib (inhibits BCR-ABL protein product)

Transfusion reactions Anaphylactic reaction

Immediate (may be due to IgA deficiency) Hypotension, Angioedema, Resp distress Tx: Stop infusion, Epinephrine, IV fluids, maintain airway

Overflow incontinence

Inability to void normally => overdistension of the bladder Continuous leakage of urine Tx underlying cause - intermittent cath

Direct Xa inhibitors - Rivoraxaban - apixaban - Edoxaban

Inactivates Factor Xa => reduced thrombus formation

Direct thrombin inhibitors - Bivalirudin - Argatroban - Dabigatran

Inactives thrombin => reduced thrombus formation

What can be used to decrease the risk of atelectasis after surgery?

Incentive spirometry Pain control and Physical therapy

What findings of chronic liver disease are associated with increased surgical mortality?

Increase bilirubin Decrease albumin Prolonged PT Hepatic encephalopathy

You are treating a 45 year old white man for HTN. There is no other significant past medical history and he takes no medications. Lab studies are normal. You recommend lifestyle modification and start daily amlodipine. After one month, bp is 145/88 in both arms. What is the most appropriate management for patient's htn?

Increase dosage of amlodipine or add thiazide, ACE-i or ARB. F/u in one month

Why should the use of unopposed estrogen be avoided?

Increase risk of endometrial hyperplasia

What findings would be expected on pulmonary function testing in a patient with COPD?

Increased TLC, Decreased FVC, Decreased FEV1, FEV1/FVC < 70%

a 44-year-old man is brought to the emergency department after a motor vehicle collision. He appears confused and disoriented. A CT scan of the head suggest a focal contusion with surrounding cerebral edema. What is the appropriate management of this patient?

Increased intracranial pressure+> ventriculostomy, elevate head of bed, IV mannitol, hyperventilation, sedation, hypothermia

A 68 year old man has idiopathic pulmonary htn. He is currently taking furosemide, diltiazem, and is on sup o2. His pulmonary artery pressure is stable. What other treatment should be added to this patient's regimen?

Increased risk for intrapulmonary thrombosis and thromboembolism Tx: Long term AC with warfarin

ADP receptor blockers Clopidogrel Ticlodipine Ticagrelor Prasugrel

Inhibit ADP receptor on plts => decreased activation of GpIIb/IIIa => inhibit plt aggregation

Warfarin

Inhibits Vit K epoxide reductase => impaired production of factors 10, 9, 7, 2 => reduced thrombus formation

What is the MOA of FQs?

Inhibits topoisomerase II and IV

DVT Tx

Initial anticoagulation: LMWH, UF Heparin, Fondaparinux, Apixaban, Rivaroxaban Long-term anticoagulation: Warfarin, Apixaban, Dabigatran Duration of 3 months if reversible risk factor present (OCP use, immobilization) Duration of 6 months or indefinite for idiopathic DVT or ongoing risk factor IVC filter for patients with contraindication to AC or failure of AC Pregnancy: LMWH for anticoagulation, IVC filter indicated when anticoagulation is contraindicated

What is the treatment for hepatitis B?

Interferon, Laniverdine, Adefovir, Entecavir, Telbuvidine, Tenofovir

Lacunar infarcts Ataxic hemiparesis

Ipsilateral weakness and limb ataxia out of proportion to the motor defect Possible gait deviation to the affected side No cortical signs

Microcytosis (MCV < 80)

Iron deficiency Thalassemia AOCD - severe Sideroblastic anemia Lead poisoning

Multifocal atrial tachycardia

Irregular tachyarrhythmia associated with at least 3 different P wave morphologies Associated with pulmonary disease (esp COPD/PE/Pna) Management: - Correct hypokalemia or hypomagnesemia if present - Control rate with verapamil, diltiazem or Beta blocker (may be contraindicated in advance COPD) - Does not respond to DC cardioversion

NEC

Ischemic necrosis of the intestinal mucosa Risk factors: Preterm birth, Enteral feeding Presentation: bilios vomiting, lethargy, poor feeding, diarrhea, hematochezia, abd distension/tenderness, signs of shock Dx testing - Met acidosis, hyponatremia - Abd x-rays: may show bowel distension, free air under the diaphragm, or pneumatosis intestinalis - U/S Tx- Stop enteral feeds, start parenteral feeds - Abx: ampicillin, cefotaxime, metronidazole - Surgery

What is the treatment for latent TB?

Isoniazid for 9 months (b6)

Chronic HTN

Known HTN prior to pregnancy BP > 140/90 before 20 weeks GA 2 readings 4 hrs apart BP remains elevated >3 months postpartum Tx: <160/<110 without target organ damage => no tx

What is the most appropriate next step in mngt given each of the following abnormal Pap results? LSIL

LSIL => age = 21-24 => Rpt Pap LSIL => age = 25-29 => colposcopy LSIL => age = over 30 => HPV -> positve colpo but negative rpt HPV in one year

Seizure workup?

Labs: Electrolytes, Glucose, Calcium, Magnesium, CBC, Renal function, Liver function, Toxicology LP in anyone with suspicion of infection ECG with any loss of consciousness CT scan (MRI later if normal CT) EEG

What are the surgical treatment for ectopic pregnancy?

Laparoscopy or Salpinectomy

A 72 year old man presents with abrupt, severe weakness of his right arm. He also has less severe weakness of his right leg and slurred speech. Which artery is most likely affected in this patient?

Left MCA

CML Eval?

Leukocyte alkaline phosphatase is low t(9;22) Philadelphia chromosome positive

A 40 year old woman develops a pruritic rash on her wrists and shins. The rash is composed of violaceous, polygonal papules. What is the most likely diagnosis?

Lichen Planus - Purple, Polygonal, Papules - drugs + hep C Tx: Topical Corticosteroids

Sepsis

Life threatening organ dysfunction caused by a dysregulated host response to infection. Organ dysfunction is defined as an increased in the SOFA score of two or more points SOFA score incorporates the following - CNS (GCS) - Cardiovascular (amt of vasoactive meds required to maintain MAP) - Renal (serum creatinine or urine output) - Coagulation (plt count) - Hepatic (serum bilirubin level) - Respiratory (Pao2:Fio2 ratio)

Hodgkin Lymphoma Tx

Limited/localized disease: chemo + rads Advanced/widespread disease: chemo only Most common chemo ABVD - Adriamycin, Bleomycin, Vinblastine, Dacarbazine

Antiretrovirals NRTIs Class side effect Zidovudine Didanosine Stavudine Abacavir Tenofovir

Lipoatrophy, Hepatic steatosis, Lactic Acidosis 1. Used during labor - causes anemia 2. Peripheral neuropathy - pancreatitis 3. Peripheral neuropathy 4. Fatal hypersensitivity reaction in HLA-B5701 5. Renal Failure

A 69 year old man complains of mild-to-moderate LLQ pain. He has mild constipation but no hematochezia or melena. His appetite is decreased, but he has been tolerating liquids well. On exam, he has mild tenderness in the LLQ. No rebound or guarding present. Bowel sounds are decreased but present. CT of his abdomen reveals bowel wall thickening of the descending colon. What is the most appropriate treatment?

Liquid diet Cipro + Metronidazole (do not use alcohol!!)

a 39-year-old woman with a history of bipolar disorder presents with depressed mood, loss of appetite, insomnia, lethargy, and recurrent thoughts of death. How is depression management patient's with bipolar disorder?

Lithium, anticonvulsants like lamotrigine and valproate, atypical antipsychotics like olanzapine and quetiapine, ECT

What antihypertensives are common used in pregnancy? What antihypertensive should be avoided?

Long term tx: methyldopa, labetalol, beta blockers, CCBs Acute tx: IV hydralazine, IV labetalol, oral nifedipine Contraindicated: ACE-i/ARB: Fetal renal and cardiac abnormalities, Spirinolactone: feminilazation of male fetuses

a 2 month infant is brought to the emergency department and is diagnosed with a left humerus fracture. The parents report that the baby rolled off the bed at home. The baby also has bruises on his legs. After stabilization of the fracture, what are the most appropriate next steps for this case?

Look for other irregularities, PT, PTT, CBC, CMP, skeletal survey, CT of head contact CPS

What is the tx of acute CHF with pulmonary edema?

Loop diureitcs, morphine, nitrates, oxygen, positioning and pressors

what additonal classes of drugs are used to relieve symptoms in chronic CHF?

Loop diuretics - weight themselves digoxin - positive inotrope Hydralazine and ISosorbide Dinitrate - decrease afterload and decrease preload

What is mag toxicity?

Loss of DTRs, resp depression, CV collapse Tx: stop infusion, IV calcium gluconate

What is the management of preeclampsia with severe features?

Low BP: IV hydralazine/IV labetalol/oral nifedipine + nimodipine Seizure ppx: IV mag sulfate Delivery (induce w C/S)

What lab markers suggest anemia due to hemolysis?

Low H/H with normal MCV High reticulocyte count High LDH High indirect bilirubin Low haptoglobin

DIC Labs?

Low plts High PT/PTT Low fibrinogen High fibrin split products (d-dimer) Peripheral smear: schistocytes

Which cancers tend to mets to the brain?

Lung, Breast, Skin, Kidney, GI

What are the clinical features and treatment of each of the stages of syphilis? Secondary

Lymphadenopathy, Diffuse skin rash, Condylomata mata lata IM Benzathine PCN Gx1

What are the major causes of appendicitis?

Lymphoid hyperplasia - children Fibroid bands or Fecaliths - Adults

A 21 year old man presents with recent weight loss, pruritus, and night sweats. Physical exam reveals hepatosplenomegaly and nontender cervical Lymphadenopathy. What is the most likely dx and what is the single best test to confirm the dx?

Lymphoma (Hodhkin) Excisional lymph node biopsy

A 56 year old postmenopausal woman comes to the physician because of a 1 week history of light vaginal bleeding. What is the most appropriate next step in management?

MC: ATrophy Tx: Endometrial biopsy

What is the most common cause of interstitial nephritis?

MEDICATIONS: Abx: B-lactam, Sulfonamides, Aminoglycosides, NSAIDs

A 66 year old man presents with progressive memory loss that began about three years ago. His mother died of Alzheimer disease at 75 years of age. The patient's TSH and serum B12 level are nomal, and his wife and children are eager to start him on medication. What is the most appropriate next step?

MRI of brain to r/o cancer, NPH

A 66 year old man presents with progressive memory loss that began about three years ago. His mother died of Alzheimer's disesae at 75 years of age. Pt's TSH and serum B12 levels are normal and his wife and children are eager to start him on medication. What is the most appropriate next step?

MRI of the brain

A 43 year old woman comes to the clinic for electrical pain that radiates down her spine and into her arms whenever she tilts her head downward, beginning two days ago. She has no numbness or tingling in the extremities, although she reports that she has had episodes of numbness in the legs in the past. She also recalls a 2-week period several months ago in which there are a dark spot in the center of her field of vision, only affecting her right eye. There was some eye pain at that time, although the vision disturbance and pain resolved spontaneously after about a month. Today, her physical exam shows no abnormalities. MRI of the brain and spinal cord shows several plaques of demyelination in various stages of healing. What is the most appropriate treatment for acute exacerbations of this disorder?

MS - Optic neuritis - sensory deficits - Lhermitte sign - motor weakness - Bowel/bladder problems - Gait/balance problems - Generalized fatigue Tx: Acute-steroids

Differential ddx of chest pain

MSK: worse with palpation and movement Pleuritic: worse with deep inspiration GERD: worse w eating PE: sudden SOB with tachycardia and altered mental status Aortic dissection: "tearing" pain Anxiety: pain associated with anxiety Pericarditis: improves when leaning forward; friction rub Pna: productive cough and fever Pneumothorax: sudden SOB

What are the causes of high anion gap metabolic acidosis?

MUDPILES Methanol Uremia DKA Propylene glycol Iron/Isoniazid Lactic Acidosis Ethylene glycol Salicyclates

What is the management of an eclamptic seizure?

Maintain ariway and prevent aspiration Roll patient to the left side Prevent trauma Supportive Oxygen IV mag sulfate Decrease BP Delivery - def tx

A 56 year old smoker with hx of HTN and HLD is being evaluated for recurrent angina. What findings on cardiac catherization would be indications for CABG?

Major indications for CABG - Left main coronary artery stenosis > 50% - Severe 3 vessel coronary artery stenosis

PULMONARY EMBOLISM TX

Massive PE: tPA, streptokinase, urokinase Smaller PE: LMWH, wafarin Duration of 3-6 months if reversible risk factor like OCP use or immobilization Duration indefinite for idiopathic or ongoing risk factors Pregnancy: LMWH for anticoagulation, thrombolytics only life-threatning PE

What should be performed at every prenatal visit?

Maternal weight, Maternal BP, Assess for GA, Fetal Heart Tones, Fetal Presentation 12 wks: pubis symphysis 16 wks: midway pubic to umbilicus 20 wks: umbilicus 28 wks: 28 cm

A 6 year old boy is brought to the physician for a 2 day history of an erythematous maculopapular rash that started on the head adn then spread down to the feet. Prior to that, the patient has two day so cough, congestion and eye redness. On physical, the patient has multiple white-gray spots on his buccal mucosa. What is the most appropriate treatment?

Measles - 3Cs - cough, coryza, conjunctivitis, Koplik spots - rash 4-5 days from head to trunk to feet Supportive care + Oral vitamin A

A 5 year old boy has several episodes of painless GI bleeding. His hbg is normal and is in no distress. What study would be most appropriate to diagnose a Meckel diverticulum in this patient?

Meckel's sign (Technetium-99 scan) Hemodynamically unstable: Angiography

A 48 year old man comes to the physician at his wife's insistence because of a mole on his back. She says that the mole has grown larger over the past 6 months. Examination of the skin shows a 0.7 cm, asymmetric, brown lesion; the lesion contains black and gray areas and has an irregular border. What is the most appropriate next step in mngt?

Melanoma = excisional biopsy

What are the clinical features and treatment of each of the stages of syphilis? Neurosyphilis

Meningitis, CN dysfunction, General paresis, Tabes dorsalis IV aqueous crystalline PCN G x 10-14 days

A 53 year old woman with no past medical history comes to the physician complaining of frequent hot flashes. She says that she averages 10 hot flashes daily and that they often wake her from sleep, leading to excessive tiredness during the day. Her last menstrual period was 14 months ago. Vital signs are within normal limits. Physical examination shows pale, dry vaginal mucosa but is otherwise unremarkable. What is the appropriate next step in management?

Menopause Tx: HRT with estrogen + progesterone Lowest dose first

What are the indications for emergent hemodialysis in acute renal failure?

Metabolic acidosis Hyperkalemia: >6.5, rising rapidly Uremia refractory volume overload Toxic levels of dialyzable drugs (isoprotenol, lithium, salicyclates, theophylline), Alcohol (methanol, ethylene glycol, ethanol)

Antiretrovirals - Protease-inhibitors Class Side effects - Ritonavir - Indinavir - Atazanavir

Metabolic derangements - hyperglycemia, hyperlipidemia GI sx - diarrhea 1. Inhibits cytochrome P450 2. Kidney stones 3. Jaundice from hyperbilirubinemia

What are the sx of acetaminophen toxicity?

Metabolized primarily by glucuronidation and sulfonation Produces non-toxic byproducts Small amount excreted, unchanged in urine CYP450 oxidation => free radical toxin => destroys hepatocytes Ingestion of >4 grams in one dose or >10-12 g in 24 hours

A 72 year old man is diagnosed with type 2 DM. Besides lifestyle changes, what oral medication is the first-line treatment for hyperglycemia?

Metformin

A 72 year old man is diagnosed with type 2 diabetes. What medication is the most appropriate first-line treatment?

Metformin

Which antidiabetic drug best fits each of the following descriptions? Lactic acidosis is a rare but serious side effect

Metformin

Which antidiabetic drug best fits each of the following descriptions? MOA: Decreases hepatic gluconeogenesis

Metformin

Which antidiabetic drug best fits each of the following descriptions? Recommended first-line treatment for type 2 DM?

Metformin

Which antidiabetic drug best fits each of the following descriptions? Classes of drugs that may help with weight loss

Metformin, GlP-1 agonist, SGLT-2 inhibitor

What is the drug of choice for eclampsia?

Mg Sulfate

Lead poisoining Clinical manifestaton

Microcytic anemia (lead inhibits heme synthesis) Headache Memory loss and difficulty concentrating seizures and encephalopathy Joint and muscle pain Peripheral neuropathy (foot/wrist drop) Abdominal pain Constipation Lead lines on the gums Lead lines on x-rays of the long bones in children

What is the treatment for mild acute pancreatitis?

Mild - Aggressive hydration, NPO, NG suction, correct electrolyte abnormalities, and opioid

An 8 year old boy with a past medical history of asthma comes to the physician for his well child examination. His only medication is an albuterol inhaler. His mother says he has no complaints and that his asthma is under good control. He rarely uses his albuterol inhaler during the day, and at nightime he uses it 3-4 times per month. His physical exam is normal. What is the most appropriate next step in the mangement of this patient?

Mild persistent asthma + Long acting inhaled steroid

What is included in the dx workup of a patient with dementia?

Mini-Cog: 3 words recall -> 3 - none, 1-2 - clock drawing, 0 - dementia Labs: Vit B12, TSH, CMP, CBC, UA, RPR, HIV Imaging: Noncontrast CT, MRI of brain

A 63 year old man comes to the clinic with exertenional dyspnea. He denies chest pain or cough, but he does have epigastric abdominal pain. Lab evaluation shows a hematocrit of 31.2% and evidence of iron deficiency. EGD reveals a clean-based peptic ulcer in the gastric antrum. Bx is positive for H. pylori and negative for gastric adenocarcinoma. What is the most appropriate management of this patient's anemia?

Mngt of anemia - iron supplement - need to r/o colon CA - colonoscopy

What is the clinical manifestation associated with an opthalmic artery stroke?

Monocular vision loss which is also called Transient Amarousis Fugax

What features of the HHS can help to distinguish it from DKA?

More Gradual onset over several days Minimal ketosis No acidosis Worse hyperglycemia Worse hyperosmolarity -> neuro sx (obtundation + coma) Treatment is similar: ICU admission, IV insulin, replete K+

Cluster HA

More common in men - Severe unilateral HA around the eye: redness of the eye, lacrimation, nasal congestion, nasal discharge, horner syndrome - Headaches "clustered" together in time Tx: 100% O2

Migraine

More common in young adult women - Unilateral, throbbing headache - Associated with N/V, photophobia, phonophobia - May be preceded by an aura Tx: NSAIDs, Triptans

Renal cell carcinoma

Most common primary malignant neoplasm of the kidney Risk factors: Smoking, exposure to cadmium or asbestos, age Presentation: classic triad - flank pain, hematuria, abdominal mass, Fever, WEight loss, Scrotal varicocele Labs: Polycythemia due to increased EPO Tx: Nephrectemy, immunotherapy, radiation therapy, chemo

A 66 year old woman fractures her hip after falling from standing. Radiographs reveal "punched out" lesions in the vertebrae, hips, and femur. The patient says that she has had increasing back pain, weakness, and fatigue, but she has attributed all of that to aging. Lab evaluation demonstrates, anemia, hypercalcemia, and increased BUN and creatinine. What further studies would help to establish the diagnosis?

Multiple Myeloma SPEP - Monoclonal antibody spiked M UPEP - Bence jones protein BONE MARROW BIOPSY - Increase plasma cells

Genital herpes Clinical features?

Multiple vesicles on an erythematous base -> shallow, painful ulcers Primary outbreak may be associated with systemic sx (fever, malaise, myalgias, dysuria, painful Lymphadenopathy) Recurrent outbreaks tend to be less severe and of short duration

Causes of emoblic stroke

Mural thrombus Atrial thrombus Bacterial thrombus (bacterial endocarditis) Plaque rupture Patent foramen ovale with DVT

What is the treatment for acetaminophen toxicity?

N-acetylcysteine within 8 hours of Ingestion

A 2 day old female newborn delivered at 30 weeks gestation has bilious vomiting and lethargy. An abdominal x-ray reveals air in the wall of the intestines. What is the most appropriate management of this patient?

NEC -stabilize patients -stop enteral feeds -start parenteral feeds -Abx: Ampicillin, cefotaxime, metronidazole -Intestinal perforation => surgery

A patient comes to the ED with hematochezia. What tests could be used to r/o upper GI hemorrhage?

NG lavage/EGD

A 72 year old man is brought to the clinic by his daughter because of memory problems. His daughter says he has lost interest in his normal activites and has a porr attention span. The patient is starting to move very slowly and has had urinary incontinence on several ocassions. Physical exam reveals a broad-based gait with very short steps. There is no tremor or rigidity. What is the most effective treatment for this patient's condition?

NPH Tx: Ventricular shunting

A 22 year old woman presents with RLQ. She complains of N/V and decreased appetite. On exam, she has significant guarding and rebound tenderness. CT scan of abdomen reveals non-perforated appendicitis. What is the most appropriate steps?

NPO Pain meds IV Fluids Abx: Cefoxitin, Ampicillin/sulfbactam, cefazolin + Metronidazole Surgery

How is endometriosis treated?

NSAIDs OCPs GnRH agonists (leuprolide) SEs: hot flashes, bone loss Progestins Danazol Surgery

What are the three main studies used to evaluate fetal well-being during pregnancy?

NST: at least 2 accelerations in 20 minutes Contraction stress test: monitor any decelerations w contractions BPP: NST, fetal breathing, fetal movement, fetal tone, and amniotic fluid level

A 67 yo man with a pmh of htn and diabetes is brought to the ED c/o of chest pain, sob, and nausea that began while sitting. His initial EKG shows no abnormalities. His initial cardiac enzymes are normal. His second set drawn 6 hours later shows an elevation in troponin I, but an EKG perfromed at the same time remains normal. What is his dx?

NSTEMI

What are the four major classes of antiarrhythmic drugs? "No Bad Boy Keeps Clean"

Na channel blockers Beta blockers (atenolol, esmolol, metoprolol) K channel blocker (amiodarone) Ca channel blocker (diltiazem, verapamil)

The patient above has not improved after 6 hours in the ED. An X-ray shows a dense infiltrate in his right lower lobe. O2 is 91% on 4L of oxygen by Nasal Cannula. He lives alone and has limited social support. The decision is made to admit to the medicine floor. What treatment do yo order on admission?

Neb Albuterol IV methylprednisolone IV abx - levo/moxi, 3rd ceph, pip/tazo, rocephin + azithromycin Supp O2

A 23 year old male is brought to the ER after a MVA. He is conscious but c/o right sided chest pain and shortness of breath. One xam, he has absent breath sounds on the right side. A chest xray shows a shift of the mediastinum to the left. After several minutes, the bp drops to 80/40 and he becomes less responsive. What is the most appropriate next step?

Needle Decompression

Serotonin modulators and SEs

Nefazodone, Trazodone, Vilazodone Sedation, N/V, Diarrhea

A 32 year old man complains of swelling throughout his body which has been worsening over the past several weeks. He does not drink alcohol. He denies dysuria, hematuria, or any other sx. His bp is 155/93 mmHg. Lab studies are shown: WBC 6700 Hbg 12.1 Creatinine 1.2 24 hr urine protein 5.5g/24 hrs Serum albumin 2.0 (L) What is the most appropriate treatment for this problem?

Nephrotic syndrome - Proteinuria >3.5g/d - Hypoalbumniemia - Peripheral edema Focal segmental glomerulosclerosis - Glucocorticoids - ACE-inhibitors or ARB - STATIN

What is gestational HTN?

New onset BP > 140/90 after 20 weeks GA w/o proteinuria + end organ dysfunction

What are the defining features of preeclampsia?

New onset HTN after 20 weeks GA >140/90 Proteinuria If no proteinuria: thrombocytopenia, renal failure, increase LFTs, pulmonary edema, cerebral/visual sx

Preeclampsia

New onset HTN in the second half of pregnancy - BP > 140/90 after 20 weeks GA AND Proteinuria - >300 mg/24 hrs - protein/creatinine ratio > 0.3 - 1+ protein on urine dipstick OR Evidence of end-organ dysfunction - Thrombocytopenia (plt < 100,000) - renal insufficiency (cr > 1.1 mg/dL) - impaired liver function (2x ULN) - pulmonary edema - cerebral or visual sx

What is presentation of GCA?

New onset headache (unilateral or bilateral) Scalp pain or tenderness along the temporal artery Jaw claudication Transient monocular blindness (amaurosis fugax) Elevated ESR

What medication is used to prevent vasospasm after a subarachnoid hemorrhage?

Nimodipine

A 66 year old man is undergoing an elective unilateral total knee replacement for osteoarthritis. He is strongly opposed to receiving blood products due to his religious beliefs and has a living will that confirms these wishes. He understood and acknowledge the risks of refusing blood transfusion when he signed the pre-operative consent form. During the procedure, a large artery is injured and a significant volume of blood is lost. He will not survivie without a transfusion. How should the situation be handled in the operating room with the patient under anesthesia?

No blood transfusion

A 42 year old woman has pulmonary sarcoidosis with no known extrapulmonary involvement. She denies any current resp sx, but her most recent chest CT reveals bilateral hilar adenopathy and reticular opacities in the upper lobes. What is the appropriate management for this patient?

No meds Monitor for resp sx Prednisone 4-6 weeks if worsening

A 51 year old man comes to the clinic for routine health eval. He has no complaints or chronic health problems. what tests does the USPSTF recommend for prostate cancer screening in this patient?

No recommended prostate CA screening

Lung cancer screening

No recommended screening for asymptomatic non-smokers Annual chest CT for patients ages 55-80 who have a 30-pack year history of smoking (still smoking or quit within 15 years)

What is the classic description of a nodular basal cell carcinoma lesion?

Nodular: pearly, translucent, telagniectasia Superficial Morpheaform

A 52 year old man with autosomal dominant polycystic kidney disease comes to the ED complaining of a sudden, severe headache that began 2 hours ago. He gets mild headaches from time to time, but has never had a headache this severe before. What is the most approrpriate dx test?

Non contrast CT of the head. If negative => LP

What are the main treatment options for primary dysmenorrhea?

Non pharmacologic interventions: heating pad, exercise NSAIDs Combined hormonal OCPs

Appendicitis treatment? - nonperforated - perforated

Non-perforated: Cefoxitin, ampicillin/sulbactam, or cefazolin plus metronidazole Perforated: pip/tazo, ticarcillin/clavalunate, or a third-generation cephalosporin plus metronidazole

What is the recommended screening for ovarian cancer?

None

Umbilical cord

Normal: 2 arteries and 1 vein One artery may indicate renal malformations

a 10 day infant has white papular lesions in the mouth on the palate and tongue. The patient is eating well and has no other symptoms. What is most appropriate treatment?

Nystatin oral suspension: 1 mL to each side of the mouth 4 times daily for 2 weeks fluconazole oral if unresponsive to nystatin 6 milligram/kilogram day 1 and then 3 milligram/kilogram x2 weeks

PID Tx

O/P - Cefotriaxone + 14 d doxycycline - Cefoxitin (IM) + Probenecid (oral) + doxy (oral) + metronidazole (oral) Inpatient - IV clindamycin + IV gentamicin - Cefoxitin or cefotetan (IV) + doxycycline (IV or oral) Avoid FQs due to resistance Counseling, STD testing, partner treatment

What abx are used in the treatment of PID?

O/P: Ceftriaxone + Doxycycline Cefoxitin + probenecid + doxy + metronidazole Inp: Cefoxitin/cefotetan + doxycycline Clindamycin + Gentamcyin

What agents can be used in the induction of labor?

OXYTOCIN Given IV Misoprostol/Dinoprostone - Ripen Cervix

A 58 year old man with chronic hepatitis C infection and cirrhosis comes to ED with severe upper GI bleeding. In addition to volume resuscitation, what drug could help reduce his bleeding while waiting to transport him to the endoscopy suite?

Octreotide

PCOS (2/3)

Oligo/amenorrhea Hyperandrogenism: acne, hirsutism, male pattern balding, increase testosterone Polycystic (string of pearls) on US

What is the treatment for a primary episode of genital herpes?

Oral acyclovir, valacyclovir, or famciclovir x 7-10 days

A 7 day old newborn has a purulent discharge from both eyes. Culture of the exudate reveals Chlamydia trachomatis. What is the most appropriate treatment?

Oral erythromcyin for 14 days

What exam features distinguish orbital cellulitis from periorbital cellulitis

Orbital cellulitis - could get vision loss, inside orbit, pain with eye movement, weakness or paralysis or extraocular muscles, proptosis Periorbital cellulitis - outside orbit, no pain with eye movement

What is the main risk factor for ovarian torsion?

Ovarian mass/cyst: > 5 cm in Diameter

Causes of acute pancreatitis

PANCREATITIS HyperParathyroid (hypercalcemia) Alcohol > 30% Neoplasm Cholelithiasis - 40% Rx drugs: Didanosine (HIV drugs), HCTZ, Lasix (certain diuretics), Valproic acid, azathioprine, estrogens ERCP Abdominal Surery Hypertriglyceridemia Idiopathic Trauma Infection (mumps) Scorpion sting (very rare)

What is the mngt of PCOS?

PCOS - oligo/amenorrhea, hyperandrogenism, polycystic ovaries on U/S, Infertility, Obesity, Insulin resistance, Endometrial hyperplasia Mngt - Diet and exercise, OCPs, Metformin, Spironolactone, Clomiphene - trying to get pregnant

Which type of pna are immunocompromised patients at greater risk of getting?

PCP - Diffuse bilateral interstitial infiltrates HIV + CXR

A 3 week old infant delivered at 26 weeks gestation develops tachypnea and tachycardia. The patient is dependent on mechanical ventilation. A continulous machine-like heart murmur is heard at the mid-left sternal border. What med is appropriate to correct this patient's condition?

PDA - Indomethacin Or Ibuprofen

What are the adverse effects of phenytoin?

PHenytoin Gives MDs Frustration Peripheral neuropathy Hirsutism Gingival hyperplasia Megaloblastic anemia Drug-induced lupus SJS Fetal Hydantoin syndrome

What abx used in the treatment of pelvic inflammatory disease?

PID: Infxn of upper reproductive tract: uterus, fallopian tubes, ovaries O/P: IM Rocephin + Doxycyline Cefoxitin + Probenecid + Doxycylcline Inpatient: Cefoxitin or Cefotetan + Doxy Clindamycin + Gentamicin

A 24 year old G2P1 woman at 30 weeks gestation is brought to the hospital because she thinks that her water broke. She denies contractions. Vital signs are within normal limits. Sterile speculum examination shows pooling of clear fluid in the vaginal vault, consistent with ruptured membranes. The fetal heart rate is reassuring. What is the most appropriate mngt?

PPROM (<37 weeks, no labor) Admit to hospital If signs of infection => Deliver IM bethamethasone tocolysis: Indomethacin, Nifedipine, Mg prophylaxis Abx: Ampicillin + Azithromycin Deliver @ 34 weeks

What is the treatment of Graves disease during pregnancy

PTU (1st Trimester) Methimazole (2nd or 3rd trimester) - 1st trimester would have scalp defect (however both could cause agranulocytosis) Thyroidectomy Beta blocker with atenolol, propanolol

What are the physical signs of appendicitis?

Pain at McBurney's point (1/3 distance form ASIS to umbilicus) Psoas sign (psoas pain with passive extension) Rovsing sign (RLQ pain with LLQ palpation) Obturator sign (RLQ pain with passive internal rotation of the hip)

What's the initial management of a patient with a 5 mm renal stone?

Pain meds (narcotics, NSAIDs) Tamsulosin

Sickle Cell Disease

Painful sickle crises due to dehydration, hypoxia, or infection. Functional asplenia => susceptible to encapsulated organism

Hodgkin Lymphoma Presentation?

Painless cervical lymphadenopathy Mediastinal lymphadenopathy Hepatosplenomegaly B symptoms (fever, night sweats, weight loss)

What is the classic scenario for bladder cancer?

Painless gross hematuria old man smoker Tx: Transurethral cystoscopic resection

A 24 year old man comes to the clinic after discovering a lump in his anterior neck while shaving. Physical examination of the neck reveals a solitary nontender thyroid nodule. what is the most appropriate mngt for this patient?

Palpable nodule => T4, TSH, US Hyperthryoid => Radioactive iodine uptake scan - cold: 10% cancer -> FNA - hot: Tx hypothyroid Euthyroid - FNA Hypothryoid - FNA

A 32 year old woman complains of severe epigastric pain that radiates to her back. She has no pmh and denies alcohol use. She has significant n/v. On exam, she has severe epigastric and RUQ tenderness. Vitals are normal other than mild tachycardia. Labs reveal WBC 22,000, ALT 142, and amylase 378. What is the likely cause of her symptoms, and what are the most appropriate next steps?

Pancreatitis from gallstones - NPO - IV pain meds - NGT - CT -U/S - ERCP - Then down the line, cholecystectomy

What are the current guidelines for cervical cancer screening?

Pap Test - Begin @ 21 - 21 - 29: every 3 years - >30: pap smear q3 years pap + HPV q5 years - stop @ 65 years

Cervical cancer screening

Pap smear every 3 years starting at age 21 In women ages 30-65, screening may be performed with Pap smear plus HPV every 5 years or pap smear every 3 years Cervical cancer screening is not recommended for most women over age 65

What are the clinical features and treatment of each of the stages of syphilis? Primary

Papule => Chancre (painless, clean based ulcer with indurated edges) IM Benzathine PCN Gx1

Hepatitis C Transmission

Parenteral, sexual (limited, transplacental Chronic form in 80-90% Carrier state Increased risk for Hepatocellular CA No vaccine

What is the difference between a simple partial and complex partial seizure?

Partial seizure - Aura - burning rubber - Focalized sensory or motor deficits - Localized paresthesias - Hallucinations - Repetitive/purposeless movements Simple - intact consciousness Complex - altered consciousness

What are the key difference between type 1 and type 2 DM?

Pathophysiology: Beta cell Destruction /// Insulin Resistance Body Habitus: Normal/Thin /// Overweight or obese Age of onset: Childhood/Adolescense /// Older patients Acute complication: DKA /// HHS

a 6-week-old male newborn is brought to the physician for scleral icterus. His mother says that he did not have jaundice at birth. She also noticed a lightening of the stool color and a darkening of the urine. Lab study show elevations in serum conjugated bilirubin and liver aminotransferase is. One is the most appropriate diagnostic imaging study for this condition?

Patient has biliary atresia so the best diagnostic imaging would be a cholangiogram

What patients are eligibile for the medical managemetn of ectopic pregnancy?

Patient must be stable Normal renal and liver functions hCG < 5000 mIU/mL No fetal cardiac activity on US Size < 3.5 cm Patient will be complaint with follow up

What measures should be employed to prevent ventilator-associated pna?

Patients Bed at 30-45 degrees ET cuff inflated to 20 cm H20 Minimize sedation Minimize days of intubation Good Hand Hygiene

sudden infant death syndrome

Peaks at 2-4 months Risk factors: Exposure to cigarette smoke, maternal age less than 20 years, prematurity, prone sleeping position, soft bending, and overheating Measures: Infant's placed on back to sleep; firm sleep surface; no pillows, soft objects, or loose bedding; avoid smoking during pregnancy and after delivery. Less effective measures: breast feeding and use of a pacifier when sleeping

Which antibiotic is used in the treatment of syphilis?

Pen G

What is the most common cause of upper GI bleeding?

Peptic ulcer Esophagitis Esophageal varices

AML Eval?

Peripheral smear (myeloperoxidase + myeloblasts, Auer rods) bone marrow bx

ALL Evaluation and Tx?

Peripheral smear and bone marrow biopsy Tx: 24-36 month of chemo (induction, consolidation, maintenance)

Aspirin MOA

Permanently inhibits COX1 and COX2 => blocks conversion of arachidonic acid to thromboxane A2 => inhibits plt aggregation

What are the treatment options for HPV genital warts?

Physical - surgical excision, laser therapy, cryotherapy Chemical - podophyllin, Tricholoracetic acid, 5-FU Immunologic - Imiquimod, IFN-alpha

What is the difference between physiologic jaundice and breast milk jaundice?

Physiologic: ~50% neonates, starts at 2-3 days, peaks at 3-5 days, bilirubin <10, due to inadequate UDPGT enzyme. Tx: phototherapy Breast Milk Jaundice: starts at 3-5 days, elevated for 3-12 weeks, bilirubin <10, due to an unknown substance in breast milk that increases internal absorption of bilirubin

Methods of contraception Progestin-only?

Pill (minipill), DMPA injection, Nexplanon, Mirena Notes: irregular bleeding, menstrual irregularities, DMPA - weight gain, mood changes, decrease bone mineral density

Methods of contraception Combined estrogen-progestin

Pills (OCP), Patch, vaginal ring Notes: Irregular bleeding, bloating, Nausea, breast tenderness, HA, HTN, MI and stroke, VTE

Inpatient diverticulitis treatment?

Pip/tazo Ticarcillin/clavulanate Ampicillin/sulbactam

What are the options available to provide emergency contraception?

Plan B- Levonorgestrel Yazpe Reigmen - Ethinyl Estradiol + Levonorgesterol - 72 hrs Ulipristal - 120 hrs Copper IUD - 5 days

Infants of mothers with pregestational diabetes are at increased risk for what complications?

Polyhydramnios Congenital malformation: caudal regression syndrome, situs inversus, NTD, cardiac abnormalities Preterm birth Macrosomia Unexplained still birth Hypocalcemia and Hypoglycemia

an 8-year-old boy is brought to the physician because of gross hematuria and generalized edema. His blood pressure is 150/88. Urinalysis shows red blood cells, RBC casts, and proteinuria. The patient had a significant throat infection 3 weeks ago. Was treatment for this patient?

Post streptococcal glomerulonephritis - check ASO titer - supportive care with sodium and H2O restriction and diuretic furosemide

What is the management of diabetes during pregnancy?

Pregestational - malformations, stillbirth, subQ insulin, fetal surveillance with NST at 32-34 weeks, Deliver at 38 weeks Gestational diabetes - fetal macrosomia, diabetic diet

What is ddx for secondary amenorrhea?

Pregnancy (most common cause) PCOS Prolactinoma - associated with galactorrhea, Headache, visual field defect Thyroid disorder premature ovarian failure Sheehan syndrome (postpartum pituitary infarction) Ashermann syndrome (intrauterine adhesions)

What are the primary risk factors associated w NEC

Prematurity Enteral feeding

What are the major categories of acute kidney injury?

Prerenal - due to underperfusion to kidney - dehydration, CHF Postrenal - Downstream to kidney obstruction - Prostate dz - BPH, CA Intrinsic - ATN, AIN, Renovascular, TTP-HUS, Malignant HTN

A 73 year old woman has metastatic breast cancer with a life expectancy of three months. She is in constant pain and is refilling her narcotic pain medication sooner than shoe should. There is a remote history of substance abuse and dependence. She asks if she can be prescribed more pain pills. What should be done for the patient?

Prescribe more Pain Pills

Acetaminophen overdose Presentation Treatment

Presentation - 1st 24 hrs: N/V, diaphoresis, normal labs - 24-48 hours: Hepatotoxicity, elevated ALT/AST - 72-96 hours: Liver failure (jaundice, encephalopathy, coagulopathy), renal failure Tx - Activated charcoal (within 4 hours) - N-acetylcysteine (within 8 hours)

Mitral regurgitation Presentation Physical exam Treatment

Presentation - Asymptomatic, Exertional dyspnea and LV failure, Pulmonary hypertension, A fib Physical Exam - Holosystolic mumur located at the apex and radiating to axilla - Murmur is louder int he left lateral decubitus position Tx - Valve replacement/repair - ACE-i or hydralazine can help manage sx short term

MVP Presentation Physical exam Tx

Presentation - Chest pain, palpitations, exertional dyspnea, dizziness, panic attacks, paresthesias/numbness, increased risk of arrhythmias, endocarditis, strokes, SCD Physical exam - Midsystoic click, Late systolic crescendo murmur Tx - Treat complications (mitral regurgitation, arrhythmias, etc.)

Aortic Stenosis Presentation Physical exmaination Treatmetn

Presentation - Exertional dsypnea - Exertional chest pain - Exertional dizziness or syncope - SCD Physical examination - Weak, delayed prolonged peripheral pulses - Systolic crescendo-decrescendo murmur located right 2nd intercostal space and radiating to the carotids Treatment - TAVR when symptomatic

Mitral stenosis Presentation Physical Examination Treatment

Presentation - SOB, Hemoptysis, Atrial fibrillation, Right sided heart failure Physical exam - Opening snap, Rumbling late diastolic murmur located at the apex, Mumur is louder in left lateral decubitus position Tx - Valve replacement/repair or balloon valvuloplasty - Diuretics and Beta blockers can help relieve sx while awaiting for sugery

What is the classic presentation of NPH and how is it treated?

Presentation - Wacky, Wet, and Wobbly Tx: Ventricular shunting

Aortic Regurgitation Presentation Physical exam Treatment

Presentation: - often asymptomatic - may cause palpitations or heart failure Physical exam: - Bounding pulses - Wide pulse pressure - early diastolic murmur located in left sternal border Tx - ACE-i or CCB to decrease afterload - Avoid Beta blockers (increase regurgitation) - Valve replacement/repair when symptomatic or when LVEF < 50%

Ethylene glycol and Methanol Presentation Treatment

Presentation: - Early: sedation and inebriation - Late: Metabolic acidosis, coma, hypotension, death - Methanol: damage to the retina and basal ganglia - Ethylene glycol: Renal failure, oxalate crystals in urine Treatment - Fomepizole: inhibits alcohol dehydrogenase - Sodium bicarbonate: corrects the acidosis - Supportive care with IVF and intubation - Hemodialysis for severe acidosis or severe toxicity

BZs OD What is presentation? What is the treatment?

Presentation: CNS depression, no resp depression or hypotension Tx: Supportive care, Flumazenil is generally not recommended because it may cause seizures, Activated Charcoal NOT INDICATED

Opioid overdose What is presentation? What is treatment?

Presentation: Lethargy, depressed mental status, resp depression, miosis, ileus, mild bradycardia and hypotension Treatment: Naloxone (opioid receptor antagonist)

ASA OD Presentation Treatment

Presentation: Tinnitus, Hyperthermia, Resp alkalosis followed by metabolic acidosis, Nausea/Vomiting, Dehydration, AMS Treatment: Activated Charcoal (within 1-2 hrs of ingestion), Sodium bicarbonate (alkalinize the urine - also tx for TCA overdose), Hemodialysis if it's really bad

A 56 year old man comes to the physician because of a 5 year history of worsening blurred vision when looking at near objects. He says that he now has difficult reading the newspaper, often needing to hold it at arm's length in order to make out the words. What is the most appropriate treatment for this patient's condition?

Presyopia Lens becomes less elastic => lose ability to focus on near objects Tx: Reading Glasses

Tuberculosis

Primary infection: rarely fatal Latent TB: asymptomatic Reactivation TB - Presentation: fever, night sweats, weight loss, cough, hemoptysis - CXR: infiltrate in apical-posterior segments - Dx: Sputum culture x3

Whats the difference between primary amenorrhea and secondary amenorrhea?

Primary: Age 13 if no secondary sex characteristic Age 15 if secondary sex characteristic present Secondary: Absence of menses for >3 cycles or 6 months in someone who has previously menstruated.

What is CJD

Prion disease that causes spongiform encephalopathy Rapidly progressive dementia plus muscle spasms and myoclonus Dx: Elevate 14-3-3 protein in CSF

What are the risk factors for ectopic pregnancy?

Prior ectopic pregnancy Hx of tubal surgery Hx of PID or other STDs Multiple lifetime sexual partners Smoking Infertility IUD in place

A 12 year old boy is brought to the ED by ambulance with sudden lower abdominal pain. He is diagnosed with appendicitis and appendiceal perforation and the surgical consultant recommends emergency surgery. The boy's parents are out of town for their anniversary adn they left his 17 year old sister in charge. What is the most appropriate next step in the care of this patient?

Proceed with surgery - Emergency, STIs, Pregnancy, Provision of contraception, drug or alcohol dependence

Open angle glaucoma Presentaion Dx Tx

Progressive vision loss (peripheral => central) Increase IOP on tonometry, Visual field testing, Cupping of optic disc Topical lantoanoprost, bimatoprost, tramadol, laser trabeculoplasty, surgery

What is the difference between nephrotic syndrome and nephritis?

Proteinuria > 3.5 g/d = proteinuria <3.5 g/d = hematuria = rbc casts

Opportunistic infections HIV Toxoplasmosis

Protozoan found in cat feces and undercooked meat CD4 < 100 Imaging: ring-enhanced lesion Ppx: TMP-SMX when CD4 < 100

A 17 year old girl at 21 weeks gestation comes to the clinic to discuss her pregnancy. She currently lives with her boyfriend and is planning on raising the child herself. The patient's parents have come with her on the visit. They believe she should put the baby up for adoption. They ask the physician to convince the patient that adoption is best for her future. What would be the most appropriate course of action?

Provide information about caring for the child and discuss all options!

When should a child with a febrile seizure have a LP to eval CSF cell count and culture?

Prsence of meningeal signs and symptoms Incompletely immunized patients aged 6-12 months Already on antibiotics

A 33 year old woman comes to the physician because of a arash that developed over the past 2 years. Examination shows thick, erythematous plaques with silver scales involving the scalp, elbows, and knees. What are the treatment options for the patient's condition?

Psoriasis - Emoillients - Topical corticosteroids: hydrocortisone, betamethasone, clobestal - Topical calcineurin: Tacrolimus - Topical Retinoids - Topical Vit D Severe: Phototherapy Systemic therapy with methotrexate, cyclosporine, retinoids, biologic agents (etanercept, infliximab)

What are the major risk factors for suicide?

Psychiatric disorder Feelings of hopelessness/worthlessness Impulsivity Increase age Male sex Access to weapons (firearms) Hx of suicide attempts

the patient comes to the emergency department after drinking a 48 oz bottle of fabric softener in an apparent suicide attempt. What are the major risk factors for suicide?

Psychiatric disorders, feelings of hopelessness or worthlessness, and pulse 70, increasing age, male sex, access to firearms, prior history of suicide attempts

An obese 42 year old Italian-American waste management consultant collapses at his New Jersey home. Evaluation does not reveal a medical cause, and it is believed that the syncope episode was precipitated by a panic attack. What are treatment options for a patient with panic disorder?

Psychotherapy - CBT Pharm -SSRIs -SNRIs -TCAs -BZs

What is the treatment for bulimia nervosa?

Psychotherapy SSRI: Fluoxetine TCAs: Desipramine, Imipramine, Amitryptiline Buspirone

What's the Brudzinski sign:?

Pt cannot flex the neck w/o bending the knees

What initial dx studies should be ordered in a patient with a suspected acute CHF exacerbation?

Pulse ox + ABG CXR EKG BNP: <100 Normal, <500 compensated CHF, 500-1000 Decompensated HF Echo CBC CMP

A 32 year old woman complains of dysuria, fever, and suprapubic pain. On examination, she has CVA tenderness. What is the most appropriate treatment?

Pyelonephritis - Ceftriaxone, Cefepime, Cipro, Levo

G6PD deficiency

RBCs are deficient in G6PD => susceptible to oxidative damage Hemolysis when exposed to oxidants: - Sulfonamides - Anti-malarial drugs (primaquine and chloroquine) - Dapsone - Isoniazid - Fava beans Peripheral smear: heniz bodies and degmacytes (bite cells) tx: Avoid oxidants

A 65 year old man comes to the physician with hematuria and vague back pain. The physical examination is normal. His leukocyte count is 5500/mm^3 and the serum hbg is 21.8 g/dL What is the most likely dx?

RCC

An infant born at 30 weeks gestational age is breathing 62/min and is displaying nasal flaring and grunting. CXR shows hazy interstitial infiltrates. What are the most appropriate next steps in this patient's management?

RDS CPAP FiO2 < 0.4 => continue if baby is breathing FiO2 > 0.4 => Intubate and exogenous surfactant

A 56 year old man is started on IV vancomycin for suspected MRSA endocarditis. Five minutes after infusion is started, he develops a pruritic erythematous rash involving his face, neck, and upper body. What is the most appropriate next step in mngt?

RED MAN SYNDROME: Direct mast cell activation - Stop vancomycin infusion - start Diphenhydramine and ranitidine - restart infusion at a slower rate

What tests are used to confirm the diagnosis of syphilis?

RPR, VDRL - screening + responsive to therapy If positive => FTA-ABS, MHA-TP Direct observation of darkfield microscopy also can dx

What is the common cause of bronchiolitis?

RSV Winter and spring, <2 yo, Premature, comorbid condition 1-3 days of mild URI sx, copious nasal secretion, wet cough, fever, wheezing/noisy breathing

What are the causes of a non-anion gap metabolic acidosis?

RTA Diarrhea Find the urine anion gap: Urine Na + K -Cl negative = diarrhea, positive = RTA

What physical defects comprise the Tetralogy of Fallot

RV outflow tract obstruction RVH VSD Overriding aorta

Antiretrovirals: NNRTIs Class side effect Efavirenz Nevirapine Rilpivirine

Rash 1. Teratogen - vivid dreams, depression 2. Hepatotoxicity 3. Avoid in high viral load or low CD4 count

A 2 month old infant boy is having frequent spitting up of his milk-based formula. The infant does not seem to be bothered by his spitting up, though it does occur after most feedings. His weight and height have been able at the 50th percentile. What is the most appropriate next step in the management of this patient?

Reassurance Brief trial of milk-free diet thicken feeds

What is the management of genital herpes?

Recommended: acyclovir, valacylovir, famiclovir Primary outbreak: treat all patients with oral antiviral for 7-10 days (initiate within 72 hrs of symptom onset) Recurrent outbreaks: - chronic suppressive therapy: decrease frequency of recurrence, decrease risk of transmission. - Episodic treatment: decrease severity and/or shorten duration

Differential diagnosis of syncope Reflex syncope

Reflex response causes vasodilation and bradycardia - Neurocardiogenic syncope - neural reflex increases parasympathetic tone - Situational syncope (posttussive, mictrution, post-exercise) - Carotid sinus hypersensitivity

Lead poisoning Tx?

Remove the source of lead exposure Chelating agents (EDTA, Succimer, Dimercaprol)

A 65 year old man presents with hematuria and vague back pain. His physical exam is normal. His serum leukocyte count is 5500/mm and hbg is 21.8. What is the most likely diagnosis?

Renal Cell Carcinoma - paraneoplastic EPO

Vitamin K

Required for clotting factors 2, 7, 9., 10, protein C, protein S Does not cross placenta and breast milk is low in Vitamin K Given to all babies IM to prevent hemorrhagic disease of the newborn

A female infant born at 30 weeks gestational age has difficulty breathing. On physical examination, she is displaying nasal flaring and grunting. Resp are 62/min. A CXR shows hazy interstitial infiltrates. What is the most appropriate next step at this time?

Resp distress syndrome Resp support - CPAP if FiO2 < 0.4 - Intubate + exogenous surfactant if FiO2 > 0.4

What is treatment for TB?

Rifampin, Isoniazid, Pyrazinamide, Ethambutol - For 2 months and then 7 months of Rifampin and Isoniazid

What are the major side effects to RIPE?

Rifampin: urine, sweat, tears turn orange-red, revs up cytochrome P450 Isoniazid: Hepatotoxicity, Peripheral neuropathy Pyrazinamide: Hepatotoxicity, Hyperuricemia Ethambutol: Optic neuritis

What are the risk factors of ischemic colitis?

Risk factors: DM, Atheroscerlosis, PVD, SLE

What is the management of a vigorous newborn with meconium-stained amniotic fluid immediately after delivery?

Routine care O2 PRN Intubate if resp distress DO NOT SUCTION VIGOROUS

Subdural Hemtoma: Px and Tx

Rupture of the bridging veins following trauma Slow progression CT: crescent shaped lesions that do not cross the midline Tx: if no neurological defects, then monitor

Etiologic agents of bacterial meningitis with empiric therapies 3 months - 50 years

S. pneumoniae, N. meningitidis Tx: Ceftriaxone/Cefotaxime + Vancomycin

Etiologic agents of bacterial meningitis with empiric therapies Over 50 years

S. pneumoniae, N. meningitidis, Listeria Tx: Ceftriaxone/Cefotaxxime + Vanc + Ampicillin

Which antidiabetic drug best fits each of the following descriptions? MOA: Increase loss of glucose int he urine

SGLT2-inhibitor

A 37 year old woman c/o fatigue, anorexia, and myalgias. Her serum sodium is 120 and her urine osmolarity is 110. What is the most appropriate treatment for this patient?

SIADH - Tx underlying problem - Fluid restriction - Na supplementation - Loop diuretics - Vasopressin receptor Antagonist: Conivaptan + Tolvaptan

A 66 year old woman Fractures or hip after fall from standing to observe punched-out lesions in the vertebral, hips, and femur. The patient says she has increasing pain, weakness, fatigue, she has triggered all to aging. Labs showed anemia hypercalcemia, and increased BUN and creatinine. Further studies to make diagnosis which expect to see?

SPEP - monoclonal antibody spike UPEP - Bence Jones protein Bone marrow biopsy - increased plasma cells

Treatment of MI Reperfusion

STEMI - PCI within 90 minutes of first medical contact - Fibrinolytic therapy if PCI is delayed >120 minutes w/in 1st 24 hours NSTEMI/unstable angina - Fibronlytic therapy is not indicated - Unstable patients may require urgent PCI - Stable patients can wait 12-24 hours for angiography Patients with severe CAD may be candidates for a CABG - 3 vessel or left main disease

Sickle Cell Disease Osteomyelitis typically due to what?

Salmonella

A 63 year old man comes to the physician because of a 2 week history of severe itching of his skin. He says "My whole body itches except for my head. It's so bad that I can't sleep at night." Examination of the skin shows multiple small, erythematous papules with the occasional hemorrhagic crust. A 1 cm, gray-colored, raised track is found on the webbing between his fingers. What is the best treatment for this patient?

Scabies Topical Permethrin Oral Ivermectin

What are the treatment options for actinic keratosis?

Scaly Lesions Tx: cryotherapy, curettage, topical 5-fu, topical imiquinods, topical ingenol nebutate, photodynamic therapy

A 12 year old boy is brought to the physician by his parents because of a sore throat and fever. Physical examination shows a diffuse erythematous rash on the trunk and extremities but spares the palms and soles. The rash blanches with pressure. The rash is rough to touch but is nontender. The patient also has a very red, nontender tongue. What is appropriate tx?

Scarlett fever or Group A strep Tx: PCN or Amoxicillin 10-14 days course

Other health screenings

Screen for high blood pressure - All patients > 18 Screen for type 2 DM - All patients w sustained BP > 135/80 Screen for hyperlipidemia - All men 35 year old and older (or age 20 with CHD risk) - All women 45 year old and older (or age 20 with CHD risk) Screening for AAA - One time abd u/s in men ages 65-75 who have ever smoked Screening for osteoporosis - DEXA every 2 years in all women over age 65 - Screen postmenopausal women under 65 if their fracture risk is equal or greater than the fracture risk of a 65 year old woman

Dx of HIV?

Screening assay of HIV-1, HIV-2 antibodies and p24 antigen If positive then use immunoassay of HIV-1, 2 If immunoassay negative, can check PCR of HIV RNA

A 64 year old man comes to the physician because of the development of brown skin lesions on his face and trunk over the past 5 years. Examination shows multiple greasy-appearing, warty, hyperpigmented plaques with a "stuck on" appearance. What is the most appropriate mngt?

Seborrheic Keratosis Reassurance - Cryotherapy after curettage

A 22 year old man comes to the physician because of 1 year history of "dandruff" Examination of the skin shows crusting and scaling of the scalp. There are erythematous, greasy-appearing scaly plaques involving the eyebrows and the nasolabial folds. What is the appropriate management?

Seborrheic dermatitis - antifungal shampoo: selenium sulfide, ketoconazole - topical corticosteroid - topical antifungal

Lacunar infarcts Pure sensory infarct

Sensory defect (numbness) of the face, arm, and leg of one side of the body No motor or cortical signs

DIC Causes: (STOP Making Thrombi)

Sepsis (gram negative) Trauma Obstetric complications (amniotic fluid embolism, abruptio placentae) Pancreatitis Malignancy Transfusion

A patient is diagnosed with placenta previa on routine U/S early in pregnancy. How should this patient be counseled?

Serial U/S No vaginal intercourse No exercise Return to hospital if contractions or bleeding C/S at 36-37 weeks Painless vaginal bleeding U/S before digital exam Unstable: IVF, blood, C/S

Symptoms of endometriosis?

Severe dysmenorrhea Chronic pelvic pain Deep dyspareunia Infertility Bowel and bladder symptoms: constipation/diarrhea, dyschezia (pain w defecation), urinary sx, hematuria or rectal bleeding

Closed-angle gluacoma Presentation Dx Tx

Severe eye pain, headache, N/V, Blurred visions, Halos, Conjunctival redness, poorly reactive pupil Gonioscopy Topical timolol, apraclonidine, pilocarpine, IV acetazolamide, mannitol, Laser peripheral iridotomy

Hepatitis B Transmission

Sexual, parenteral, and transplacental chronic form in 10% Carrier state Increased risk of hepatocellular carcinoma and cirrhosis Vaccine available

What is necessary for hepatitis D to successfully infect a patient?

Sexually, transplacentally or parenterally Usually only in concurrent hep B infection

What is the treatment for v fib or unstable ventricular tachycardia?

Shock and CPR 30:2 for 2 minutes Check Rhythm Shock and CPR for 2 minutes Epinephrine 1 mg q3-5 minutes Check Rhythm Shock and CPR for 2 minutes Amiodarone 300 mg IV bolus, 2nd dose = 150 mg Evaluate and treat reversible causes Check Rhythm => repeat

What are characteristics of fetal alcohol syndrome?

Short palpebral fissures Thin upper lip Smooth philtrum Flattened mid face Deficient brain growth: structural brain abnormalities, <10th percentile for head circumference, abnormal nerve exam, variable intellectual disability Growth retardation - <10th percentile for height and weight - FTT despite adequate intake - Disproportionate height to weight ratios

What are the absolute contraindications to combined Hormonal contraception use?

Smokers > 35 Uncontrolled HTN Hx of VTE or thrombotic disorder Hx of an Estrogen-dependent cancer Hx of stroke or CAD Migraines with aura Hepatic adenoma or active liver disease Abnormal vaginal bleeding of unknown etiology

COPD MANAGEMENT Category A Category B Category C Category D

Smoking cessation for all patients 1. SABA PRN (albuterol, ipratropium) 2. SABA PRN + Long acting bronchodilator (salmeterol, tiotropium) 3. SABA PRN, Long acting bronchodilator, Inhaled steroid (fluticasone, budenoside) 4. SABA PRN, long acting bronchodilator, inhaled steroid +/- theophylline, PDE-4 (roflumilast) +/- home o2 if : Pulse ox <88%, pulmonary htn, peripheral edema, polcythemia Yearly influenza vaccine Pneumococcal vaccine - All smokers age 19-65 - Single revaccination after the age of 65 if more than 5 years since initial vaccination

What features distinguish adjustment disorder with depressed mood from major depressive disorder?

Specific identifiable stressors due to emotional or behavioral symptoms symptoms began within 3 months of stressors symptoms resolved within 6 months of disappearance of stressors does not meet criteria for major depressive disorder

Outline the evaluation of a patient with suspected ectopic pregnancy

Stable => N => surgical management Stable => Y => TVUS and b-hcg. If hcg >1500 => TVUS should see but if neg -> ectopic <1500 => rpt hcg in 48-72 hrs If levels go up appropriately then no problem If levels fall then it is a failed pregnancy, follow b-hcg until 0 If levels rise inappropriately => D+C -> negative chorionic villi -> ectopic D+C -> positive chorionic villi -> failed IUP

Further dx eval of the above patient demonstrates a large aortic dissection extending from just proximal to the brachiocephalic artery to the celiac artery. How should this patient be managed?

Stanford A - Admit to ICU, IV Beta blocker, Emergent surgery

What is the current recommendation regarding the timing of HAART initiation?

Start antiretroviral therapy in EVERYONE with a diagnosis of HIV infection

What is first seen histologically in patients who drink alcohol chronically?

Steatosis

With what conditions would you expect to see a positive Nikolsky sign?

Stevens-Johnson Syndrome Toxic Epidermal Necrolysis Pemphigus Vulgaris SSSS

2 day old 30 week gestation age normal bone has bilious vomiting and lethargy. Abdominal x-ray reveals air in the intestines. What is most appropriate next step in this patient's management?

Stop Enteric feeds, start parental feeds Stabilize patient Abx: ampicillin, cefotaxime, + metronidazole Surgery

Etiologic agents of bacterial meningitis with empiric therapies CSF leak

Strep pneumoniae Ceftriaxone/cefotaxime + vancomycin

Foreign Body Aspiration

Stridor + airway obstruction 6 mo - 4 yo CXR Bronchoscopy Tx - Heimlich maneuver - Tracheostomy - Bronchoscopy

What are common findings with intussusception?

Sudden abd pain < 1 minute, episodic Pallor Sweating Vomiting Bloody mucus in stool (currant jelly stools) Abd tenderness Palpable sausage-like mass

How does the presentation of placenta previa differ from that of placental abruption?

Sudden painless bleeding

What medications have the highest risk of causing drug-induced lupus?

Sulfonamides Hydralazine Isoniazid Phenytoin Procainamide

Which antidiabetic drug best fits each of the following descriptions? MOA: increases insulin release

Sulfonylurea

Which antidiabetic drug best fits each of the following descriptions? Classes of drugs that may cause weight gain

Sulfonylurea, TZDs, Insulins

What's the kernig sign

Supine with knee and hip flexed to 90 degrees. Knee extension of more than 135 degrees causes increased resistance and pain

How should patients be counseled regarding nutrition during pregnancy?

Supp folic acid + iron 300-500 kcal/day BMI <18.5 28-40 lbs, BMI 18.5-24.9 25-35 lbs, BMI 25-29.9 15-25 lbs, BMI >30 11-20 lbs Meat fully cooked no unpasteurized dairy no deli meats no fish hgih in mercury like shark, swordfish, mackerel, king fish

A 76 year old man with a history of COPD is brought to the ED with sob and worsening cough for 2 days. He smoked one pack of cigarettes daily for 45 years but quit about 10 years ago. Physical examination shows normal temperature, pulse and blood pressure. His oxygen saturation is 84% on room air. The anterior-posterior diamter of his chest is increased. There are fine crackles in the lung bases bilaterally. The heart sounds are distant, but no murmur or gallop is present. There is trace pedal edema. What should your initial order set include?

Supp o2, CXR, CBC, BMP, ECG, ABG, neb albuterol or levalbuterol, Reanalyze in 30 minutes

A 2-year-old girl is brought to the clinic with low-grade fever, runny nose, a bark-like cough for the past 3 days. She is somewhat tired but has been playing regularly during the day, and her cough worsens during the nighttime. On exam, patient has coarse breath sounds but no wheezing or stridor. Patient's oxygen saturation is 99% on RA. What is the most appropriate next step in the managment of this patient?

Supportive care 1 dose of dexamethasone

What is the treatment for a parenchymal hemorrhage?

Supportive care Maintain normal ICP Anticonvulsants Surgical decompression - for large hemorrages to reduce risk of herniation

A 21 year old man comes to the physician for a non-productive cough, fever, and myalgias for the past four days. His sx began abruptly, but he denies any dyspnea or other severe resp sx. He has no other medical problems. His examination reveals rhinorrhea, but his lungs care clear. A rapid influenza screen is positive. What is the most appropriate tx for this patient?

Supportive care > 48 hrs Severe sx: consider antiviral usage

What is the treatment of bronchiolitis?

Supportive care with O2, high fluids, steroids are NOT effective, high flow o2 up to 15 L dpeending on weight, RSV vaccine

How is genital herpes simplex virus infection managed during pregnancy?

Suppresive treatment with acyclovir after 36 weeks C-section if active/prodromal symptoms.

What is the appropriate management of a necrotizing soft tissue infection?

Surgical debridement IV broad-spectrum antibiotics - Carbapenem or Beta-lactam/B-lactamiase-inhibitor - Clindamycin - MRSA coverage Supportive care - IVF/Vasopressors

A 2 month old infant is brought into the ER and is diagnosed with a left humerus fracture. The parents report that the baby rolled off the bed at home. The baby also has bruises on his legs. After stabilizing the fracture, what is the most appropriate management of this patient?

Suspicious - Eval for other injuries - PT, PTT, CBC, metabolic panel - Skeletal survey - CT of head if retinal hemorrhages - CPS

Aortic Rupture Sx Dx Tx

Sx: CP, Dyspnea, Difficulty swallowing, Usually has steering wheel or seat belt implant on chest Dx: Plain film xray will show widened mediastinum, enlarged aortic knob, hemothorax, displacement of left mainstem bronchus, TEE, CT Scan Tx: 2 large bore IVs, blood, IVF, surgery, If Heart rate and BP decrease, use IV esmolol or diltiazem

COPD Clinical features

Sx: exertional dyspnea, chronic cough, sputum production, +/- wheezing, recurrent resp infections PE: Barrel chest, diminished breath sounds, decreased diaphragm movement, +/- digital clubbing Dx testing: - CXR: lung hyperinflation, flattening of the diaphragm - PFTs: Increase TLC, Decrease FVC, Decrease FEV1, FEV1:FVC ratio <70%

What are the clinical features and treatment of each of the stages of syphilis? Tertiary

Syphilic aortitis -> aortic aneurysms, Gummas IM Benzathine PCN Gx3

38 yo man presents to the ER c/o 1 month hx of progressive sob and dry cough. He is now unable to walk unassisted into the ER because of his sob with exertion. His PaO2 is 65 mmHg, chest xray shows bilateral perihilar infiltrates and HIV screening test is positive. What treatment should be started for his pna?

TMP-SMX + steroids if PaO2 < 70

What is the first line treatment for PCP?

TMP-SMX for CD4 < 200

Which antidiabetic drug best fits each of the following descriptions? MOA: Increases insulin sensitivity

TZDs

Which antidiabetic drug best fits each of the following descriptions? MOA: agonist at PPARy receptors

TZDs

Which antidiabetic drug best fits each of the following descriptions? May cause edema or fluid retention

TZDs

What are the T causes?

Tamponade => pericardiocentesis Tension pneumo => needle decompression then chest tube Thrombosis (MI) => cardiac cath or thrombolytic Thromobsis (PE) => thrombolytic or thrombectomy Trauma => follow ATLS protocols Toxins

Causes of Macrocephaly

Tay-Sachs disease maple syrup urine disease Neurofibromatosis Tuberous Sclerosis Hydrocephalus Increased ICP Acromegaly Skeletal dysplasia Intracranial Hemorrhage

What are some causes of macrocephaly?

Tay-Sachs disease, maple syrup urine disease, neural cutaneous syndromes, hydrocephalus, skeletal dysplasia, acromegaly, ICH, increased intracranial pressure

What are the indications for a tetanus booster in an adult patient?

Td should be given to every adult every 10 years - A Tdap booster is recommended once in place of the Td between 19-64 years. Td should also be given to patients with wounds as below Nontetanus prone wound, LE, clean and minor - >3 prior tetanus immunizations: Td >10yrs since last dose - Uncertain or <3 prior tetanus: Td (Dt if <7 yo) => complete series Tetanus-prone wound (dirt, contaminated, puncture, crush injury) - >3 prior tetanus immunizations: Td > 5 yrs since last dose - Uncertain or <3 prior tetanus immunizations: Td (Dt if <7 yo) and tetanus immunoglobulin 250 units IM at other site other than Td => complete series

An 18 year old man comes to the clinic for a scrotal mass. He discovered a painless lump on his left testicle while showering two days ago. Physical examination confirms the presence of a non-tender nodule on the inferior pole of the testicle. 1. What is the most appropriate next step in the mgt? 2. What further testing should be performed before beginning treatment?

Testicular cancer. 1. U/S of scrotum 2. Orchiectomy - inguinal approach b4 Tx: b-HCG, AFP, LDH CXR/CT a/P - lymph node Bank sperm Advanced chemo: Etoponide, Bleomycin, Cisplatin

A 9 year old boy is seen in the ED for tachycardia and lightheadedness. He has a PMH of severe asthma for which he takes multiple medications. Lately, his asthma has been under good control. On exam, his lungs are clear and his oxygen is 99% on RA. His pulse is 150/min and bp is 72/40. What medication is likely causing his sx?

Theophylline

Candida spp. vaginal discharge?

Thick, white cottage cheese-like pH 4-4.5 Yeast forms Oral fluconazole, topical clotrimazole, miconazole, or nystatin

Vaginal discharge of gardnerella vaginalis

Thin gray white fish odor pH > 4.5 Clue cells, whiff test Metronidazole, Clindamycin

a 4-year-old boy has difficulty climbing stairs and frequent falls when running. The patient did not have these difficulties in the past. When asked to stand up from the floor, he needs to use his hands to push himself up. On physical exam, he has proximal muscle weakness in his lower extremities and his calf muscles appear to be hypertrophied. One is the most appropriate medication used to treat this condition?

This is Duchenne's muscular dystrophy and the first-line treatment is oral glucocorticoids prednisone 0.75 mgper kg per day orally

Flail Chest - Dx - Tx

Three or more adjacent rib fractured in two places Tx: Supp O2, close monitoring for early signs of resp compromise No weighted objects to "stabilize" the area BiPAP by mask or Endotracheal intubation with mechanical ventilation Analgesia

A 27 year old man with a hx of schizophrenia is brought to the emergency department with a severe neck spasm that forces his head to be maintained in an unusual position. What is the treatment for this condition?

Torticolis from High Potency Antipsychotic Tx: Benztropine, Diphenhydramine

What are the TORCH infections that can be acquired in pregnancy?

Toxoplasmosis Other (syphilis) Rubella CMV HSV

What is the most common bladder cancer in the US?

Transitional cell cancer 90% USA (tobacco use, aniline dye, male sex, schistosomiasis, chronic bladder irritation)

Hepatitis E Transmission

Transmission: Fecal-oral route More severe in pregnant women No chronic form or carrier state No vaccine available SElf limited, supportive care.

Hepatitis A Transmission

Transmission: Fecal-oral route No chronic form or carrier state Vaccine available - chronic liver disease Self limited, supportive treatment

MAO-inhibitors and SEs

Tranylcypromine, Phenelzine Drug-drug interactions, Hypertensive crisis

Which antidepressant best matches each of the following statements? Good choice for depressed patient with insomnia First line for most causes of unipolar depression Good choice for patient who discontinued an SSRI due to sexual dysfunction Good choice for depressed patient with appetite suppression and weight loss May help with smoking cessation Overdose causes sedation and life threatening arrhythmia Good choice for patients with chronic pain

Trazodone, Mirtazapine SSRIs Buproprion Mirtazapine Buproprion TCAs Duloxetine

Alport syndrome

Triad of eye diseases (lenticonus, cataracts), nephritis, high-frequency hearing loss Treatment: ACE-inhibitors to improve proteinuria

What is the treatment for GERD in infants?

Trial of milk-free diet Thicken formula/breast milk Acid suppressive therapy: PPIs, H2 receptor blockers

the child is brought in for a well-child checkup. The child appears anxious during the exam. According to his mother, he is able to stand unassisted, walk with assistance, say mama and dada and stack 2 blocks. What is developmental age of this child?

Twelve months

Malrotation and volvulus

Twisting of the small bowel often resulting in bowel obstruction Can twist around superior mesenteric artery Dx testing: upper GI series, barium enema, U/S Tx: Surgery

A 38 year old man is being treated for a spontaneous DVT and PE. HE had a previous DVT two years ago. Lab studies reveal a guanine to adenine mutation in the gene for prothrombin. Other than routine treatment for this patient's current thrombosis, what is the most appropriate long-term mngt of this patient?

Tx: Heparin => Transistion to Warfarin 3 months Long term warfarin INR 2-3

Opportunistic infections HIV - Thrush

Tx: oral fluconazole, topical nystatin - if in esophagus then oral fluconazole for 2-3 weeks

What is the treatment of each type of renal tubular acidosis?

Type 1 +2: Bicarb, K+, DEiuretics Type 4: Fludrocortisone, Dietary K+ restriction

Nephrolithiasis - Types? - sx? - dx? - tx?

Types: Calcium oxalate (most common), struvite, uric acid, and cysteine Sx: acute, severe colicky Flank pain with radiation to the lower abdomen, hematuria, nausea, vomiting, lower urinary sx (urgency, frequency, dysuria) Dx: plain film x-rays, CT scan w/o contrast Tx: - Stones < 8 mm have a 50% chance of passing - Pain control with narcotics and NSAIDs - Tamsulosin - Extracorporeal shock wave lithotripsy (ESWL) for stones in the renal collecting system or upper ureter - Ureteroscopy for stones in distal ureter - Percutaneous nephrolithotomy for very large stones (>3 cm)

Typical vs. Atypical pneumonia

Typical: - Fever, sob, cough, purulent sputum, malaise, chest pain - CXR: lobar infiltrate, "white out" - S. pneumoniae, S. aureus, H. influenzae, group A strep Atypical - Nonproductive cough, Headache, fatigue - CXR: diffuse patchy infiltrates - M. pneumoniae, Legionella pneumophila, Chlmaydophila pneumoniae

Fever without a source in the 3-36 month child (well appearing, immunized completely)

UA and urine cx for girls < 24 months and uncircumcised boys < 12 months No empiric antibiotics

Breast cancer screening

USPTSF: breast self examination not recommended, clinic breast exam insufficient evidence, mammogram q2 years 50-74

What are some transient causes of hematuria?

UTI, kidney stones, exercise, trauma, endometriosis, idiopathic

What is the preferred imaging modality for most gynecological problems?

Ultrasound

DVT Presentation?

Unilateral swelling of foot, ankle or calf - measure the circumference +/- pain in the extremity +/- palpable venous cord Homan's sign is neither specific or sensitive

Status Epilepticus

Uninterrupted seizures lasting greater than 5-10 minutes ABCs Benzos (IV Lorazepam and Diazepam) Phenytoin or fosphenytoin Refractory seizures: Intubate, Phenobarbital, Pentobarbital or propofol

HRT for menopause

Use estrogen + progesterone if uterus present - unopposed estrogen increases risk of endometrial cancer For relief of menopausal sx, not to prevent chronic disease Use for only shortest amount of time and at lowest dose needed Avoid if: - Coronary heart disease - Active liver disease - Hx of breast cancer, stroke, or VTE

A newborn has a right undescended testis. If this testis does not descend, when should surgical correction be performed?

Usually comes down around 3-4 months Surgery at 6 months

PID Begin empirical treatment if what is present?

Uterine tenderness or adnexal tenderness or cervical motion tenderness

Treatment of MM

VMP - Velcade - Melphalan - Prednisone Thalidomide Autologous BM transplant (after chemo) for selected patients

What is uterine rupture presentation?

Vaginal bleeding Abdominal pain Maternal tachycardia and/or hypotension Evidence of fetal distress (most commonly bradycardia) Loss of fetal station Cessation of contractions Change in shape of uterus

Differential diagnosis of GI bleedin Lower

Vascular malformation (angiodysplasia, AVM) Mesenteric ischemia Meckel's diverituclum Colon cancer Diverticular disease (diverticulosis, diverticulitis) UC/Crohn's Hemorrhoids

Opportunistic infections HIV Kaposi Sarcoma (HHV-8)

Vascular tumors of skin and internal organs

What is the treatment for polycystic kidney disease in an adult?

Vasopressin receptor antagonist Amiloride Drainage of large cysts Dialysis Renal transplant

What is the greatest risk for sudden cardiac death in a patient who has just suffered an acute MI in the last few hours?

Ventricular Arrhythmias

A 76 year old woman presents with memory problems. Her daughter says that she has lost interest in her usual activities and has a poor attention span. She is starting to move very slowly, and on several occassions she has had urinary incontinence. Physical exam reveals a broad-based gait with very short steps. There is no tremor or rigidity. What is the most effective treatment for this patient's condition?

Ventricular Shunting

What are diagnosis for genital herpes?

Viral cx, PCR of vesicular fluid, Tzanck smear, Serology cannot distinguish past infection from present infection

Posterior cerebral artery stroke

Visual problems Homonymous Hemianopsia with macular sparing on the contralateral side

What sx would you most likely see a patient suffering form a posterior cerebral artery stroke?

Visual sx (Homonymous Hemianopsia with Macular sparing)

Differential Diagnosis of syncope Orthostatic hypotension

Volume depletion (hemorrhage, vomiting, diarrhea, diuretics) Drugs (vasodilators, alpha1 blockers, clonidine, phenothiazines) Autonomic Failure (spinal cord injury, diabetic neuropathy, Parkison disease)

How are accidental exposures to HIV-contaminated blood or body fluids managed?

WASH OUT WOUND EVALUATE FOR RISK OF TRANSMISSION - blood, mucous membrane, percutaneous wounds - get baseline HIV Postexposure ppx within 72 hours - Tenofovir, Emtricitabine, Raltegravir - Continue for 4 weeks - HIV testing should be repeated 6, 12 weeks and 6 months

CSF study for viral meningitis

WBC count: 50-500 Differential: mostly lymphocytes Glucose: normal Protein: normal or slightly elevated Gram stain: negative Bacterial culture: negative PCR: Enterovirus, HSV

CSF study for bacterial meningitis

WBC count: > 1000 Differential: mostly neutrophils Glucose: Low Protein: High Gram stain: +/- Bacterial culture: + PCR: Not used

Lacunar infarcts Sensorimotor stroke

Weakness and numbness of the face, arm and leg of one side of the body No cortical signs

Physical growth Weight Height

Weight - At birth weight by 2 weeks - Child weights double the birth weight at 4 months - child weighs triple the birth weight at 12 months - Child weighs quadruple the birth weight at 24 months Height - Height increases 50% from birth length at about 1 year - Height is double the birth length at 4 years - Height is triple the birth length at 13 years

Failure to thrive

Weight below the 2nd or 3rd percentile, corrected for gestational age Weight less than 80% of ideal weight for age Weight crosses 2 major percentile curves downward on the standard growth curve over time Weight-to-length ratio less than the 10th percentile Rate of daily weight gain is less than that expected for age

What is the treatment for the NAFLD?

Weight loss - 8% Pioglitazone Hep A and B vaccination

Integrase inhibitors?

Well tolerated Raltegravir, Elvitegravir

A 16 year old girl comes to the physician because of a non-viable productive cough for two weeks. She has had more severe coughing fits that lead to vomiting. Culture of the posterior nasopharynx reveals Bordetella Pertussis. What is the most appropriate treatment?

Whooping cough Abx: Azithromycin or Clarithromycin

DIC Presentation?

Widespread intravascular clotting -> microangiopathic hemolysis Consumption of platelets and coag factors => Hypocoag + Hemorrhage Multi-organ failure

What are features of WAGR syndrome?

Wilms tumor: renal Aniridic: Absence of iris GU anomalies: crytochoridism, ambiguous genitalia, renal rupture Retardation (intellectual disability)

A 72 year old woman has a fever 4 days after ORIF to repair a hip fracture. How should this fever be evaluated?

Wind - CXR, CBC Water - UA and culture Walking - elevate legs, LE doppler, Chest CT Wound - look at hip Wonder drugs - look at drugs

Postop fever

Wind: Pna after postop day 3 Water: UTI post op day 3-5 Walking: DVT (any time postop) Wound: wound infections are postop 5-8 days Wonder drugs: medications (any time postop)

What are the characteristic features of IIH?

Young, obese females Headaches - daily, pulsatile worse at night or early AM, possible retroocular pain worsened by eye movements, possible N/V Papilledema Most worrisome sequela is Vision Loss CT Scan: Normal CSF pressure elevated (>200 mmH20 in non obese patients. >250 mmH20 in obese patients)

Relative Risk

[A/A+B]/[C/C+D]

Osteoporosis risk factors

age personal history of fracture or family hx of hip fracture Low body weight Long-term glucocorticoid use Smoking Excess alcohol use

Vision pathology

amblyopia: decreased vision due to misuse of high during critical visual development Strabismus: Abnormal eye alignment which impairs vision and depth perception. Retinoblastoma: Most common intra-ocular malignancy and children; most common presentation is leukokorea a white reflex treatment of retinoblastoma enucleation, radiation, bracytherpay, chemotherapy

What is the nonsurgical treatment for an Achilles tendon rupture?

casting in the equinus Position

A 70 year old man comes in for his 1st doctor's appointment in 25 years. He smoked 1 pack per day for 10 years but quit at age 30. He does not drink alcohol. He has no known health problems. What cancer screening tests would be recommended?

colon, discussed prostate cancer, no CT scan, abdominal ultrasound

What are the USPSTF recommendations for screening for each of the following? Colon Breast Cervix Prostate

colonoscopy, q10 years, 50-75 Mammogram, q2 years, 50-74 Pap smear, q 3 years, 21-65 None

Asthma severe persisent

continual daytime episodes or frequent nighttime sx <60% FEV1 PRN albuterol +high dose ICS +long acting B2 agonist +PO steroid +/- montelukast +/- theophylline

Treatment of alcohol abuse

counsel patient about health risks CBT inpatient rehab 12-step program Pharmacotherapy: - Naltrexone - Acamprosate - Disulfiram

Or other diagnostic criteria for hospitalization a psychiatric patient

danger to self or others unable to provide for themselves

was a classic presentation of an infant with Hirschsprung disease

delayed meconium passage and or chronic constipation bilious vomiting failure to thrive abdominal distension explosive stool discharge following rectal exam called a blast sign

lead poisoning

early symptoms: Anorexia, decreased activity, irritability and insomnia screen children who live in areas were over 27% of the housing was built before 1950 treatment of severe toxicity over 70: Dimercaprol and Cadissodium edetate

a 36-year-old auto mechanic is brought to the emergency department following an injury in which his arm was pinned under a car for 30 minutes. He complains of continued pain in the arm. On physical examination, the arm is tight and tender to palpation. There is severe pain with passive extension. What is the 1st step in management?

emergency fasciotomy for compartment syndrome

1-day-old male neonate experiences choking episodes whenever feeding is attempted. Physical examination shows a healthy-appearing Neonate however he does appear to be drooling excessively. what is the most likely diagnosis?

esophageal atresia diagnosis by NG tube treatment by surgery, but if delayed, gastrostomy

the indications for an emergent cholecystectomy

generalized peritonitis emphysematous cholecystitis

were the symptoms of alcohol withdrawal

headache, diaphoresis, palpitations, tremulousness, mild anxiety, insomnia, GI upset, anorexia, withdrawal seizures: myotonic seizures 8 hours after drink alcoholic hallucinations: 12-24 hours DTs: 2-4 days and lasts 5-7 days

immunization

hepatitis-B vaccine is given at birth to everyone. If maternal hepatitis B, also give hep B immune globulin do not delayed immunization for premature infants or low birth weight infants avoid live vaccinations like varicella, measles, mumps, rubella, rotavirus in patients with severe immunosuppression. Rotavirus is still recommended in patients with HIV. There is no connection between certain vaccinations like MMR are and autism there is a small risk of intussusception after rotavirus immunization, so avoid in patients with a history of intussusception.

A 3-week-old infant was brought to the clinic for frequent projectile vomiting. The patient is formula fed and vomits almost immediately after all meals. On inspection the growth chart, patient was at 50th percentile for weight and 1 week of age now is at 20th percentile. What metabolic abnormalities by the presence patient?

hypochloremic hypokalemic metabolic acidosis

TTN

increased respiratory rate that resolves in 24-48 hours due to retain lung fluid seen most often in infants born by C-section without labor chest x-ray showed increased lung volumes, flattened diaphragms, prominent and vascular markings from hilar region that has a sunburst pattern treatment is with supplemental oxygen

water the adverse effects of the stimulant medications used in the management of ADHD?

insomnia, appetite suppression, decreased growth velocity, tics, psychosis or mania

Which antidiabetic drug best fits each of the following descriptions? Should be stopped for at least 48 hours following administration of IV contrast

metformin

a 37-year-old woman complains that she cannot stop worrying about becoming contaminated with germs. Over the last several months she has developed a ritual process of washing her hands in order to alleviate these fears, but the urge to perform these ritual washings interfere with her work as a waitress, so she had to quit her job what is the most likely diagnosis? What treatment options are available?

obsessive-compulsive disorder 1st: SSRI, clompiramine, venlafaxine, CBT

Child abuse

presentation: Failure to thrive, multiple fractures of varying ages, bruises, burns, subdural hematoma, retinal hemorrhage, impossible injuries, general trauma or discharge may suggest sexual abuse. Evaluation: PT, PTT, LFTs, STDs testing, CBC, skeletal survey, CT of head always reports suspected child abuse to Child protective Services

what other treatment options for PTSD

psychotherapy with exposure and CBT pharmacological with SSRIs, TCAs, MOAi, mood stabilizers, alpha blockers, atypical antipsychotics.

NHL positive for CD20 can be treated w what biologic therapy?

rituximab

indications for electroconvulsive therapy

severe, debilitating depression refractory to antidepressants depression with psychotic features severe suicidality depression with food refusal leading to nutritional compromise depression with catatonia rapid response is required like pregnancy previous good response to ECT bipolar/schizophrenia / psychosis

Which antidiabetic drug best fits each of the following descriptions? Class of drugs that commonly causes hypoglycemia

sulfonylurea, insulin

was a treatment for RSV bronchiolitis

supportive care with oxygen and IV fluids

treatment for infectious mononucleosis

supportive with NSAIDs and/or acetaminophen and rest return to sports non contact 3 weeks after symptom onset, for contact sports 4 weeks after symptom onset

Why is it important to surgically correct an atrial septal defect

unrepaired atrial septal defects lead to right ventricular failure and heart failure, arrhythmias, Eisenemeger syndrome, paroxymal embolus and stroke


Ensembles d'études connexes

Microbio Chapt 6 Learning Objectives

View Set

Chapter 22: Management of Patients With Upper Respiratory Tract Disorders

View Set

Vprašanja za pisni del izpita iz predmeta Fotografija in računalniška grafika Academia 2020.

View Set

Client Centered Design Assessment

View Set

Fundamental Information Security Final Exam Review

View Set

60 - The Second World War, 1942-1945 (comprehensive)

View Set